var chapters = [ { title: 'Akutní', items: [28,29,30,31,32,33,34,35,36,37,38,39] }, { title: 'Interní', items: [0,1,2,3,4,5,6,7,8,9,10,11,12,13,14,15,16,17,18] }, { title: 'CNS', items: [19,20,21,22,23,24,25,26,27] }, { title: 'Ostatní', items: [40] } ] var items = [ // I {title: 'I1cz', cathegory: 1, keywords: ["hypertenze", "gravidita", "žloutenka", "enalapril", "methyldopa", "labetalol", "metoprolol", "nifedipin", "vitamin K"], parts: [ { text: '

30letá gravidní pacientka byla hospitalizována pro zjevnou žloutenku a začínající bolesti v pravé horní abdominální oblasti. U pacientky se jednalo o 8. těhotenství, kterému předcházely 4 spontánní vaginální porody a 3 porody císařským řezem. Pacientka poslední 3 roky trpí chronickou hypertenzí, která je dobře kontrolována léčbou Berliprilem.

Poslední tři těhotenství byla u pacientky pravděpodobně komplikována hypertenzí, ale bez nutnosti použití farmakoterapie.

', qs: [ { task: '

Jaká účinná látka je obsažena v Berliprilu a jaký je její mechanismus působení?

', answer: '

Berlipril obsahuje enalapril, který se řadí mezi inhibitory ACE. Jeho indikací není pouze léčba hypertenze, ale i léčba a prevence symptomatického srdečního selhání.

' }, { task: '

Která antihypertenziva jsou doporučována k terapii hypertenze v graviditě?

', answer: '

Pokud je hypertenze diagnostikována před graviditou, měla by být po otěhotnění léčena stejně jako před ním. Výjimku tvoří podávání inhibitorů ACE, blokátorů angiotenzinu II a kirenů, jejichž podávání je v těhotenství kontraindikováno kvůli riziku malformace plodu. Lékem volby u mírné hypertenze během těhotenství je methyldopa a labetalol (smíšený antagonista α/β receptorů). Dále lze použít selektivního antagonistu β1 receptorů metoprolol (zejména v pozdější fázi těhotenství), a blokátor kalciových kanálů, nifedipin (potenciální synergismus se síranem hořečnatým (tokolytikum) může vyvolat prudkou hypotenzi).

' } ] }, { text: '

Pacientce byla gravidita diagnostikována v 70. dni těhotenství a okamžitě jí lékař vyměnil Berlipril za Dopegyt (250 mg 2x denně). 2 týdny po nasazení Dopegytu byl tlak dobře kontrolován (TK 130/90).

', qs: [ { task: '

Proč byl pacientce zaměněn Berlipril za Dopegyt?

', answer: '

Berlipril byl zaměněn za Dopegyt z důvodu jeho kontraindikace v graviditě (především II. a III. trimestr). Náhodná aplikace ACEi v I. trimestru není důvodem k přerušení gravidity. Je známo, že expozice ACEi během II. a III. trimestru vyvolává u člověka fetotoxicitu (např. snížená funkce ledvin, oligohydramnion a retardace osifikace lebky) a neonatální toxicitu (selhání ledvin, hyperkalemie a hypotenze).

' }, { task: '

Jaká účinná látka je obsažena v Dopegytu a jaký je její mechanismus působení?

', answer: '

Dopegyt obsahuje methyldopu, která se řadí mezi centrálně působící antihypertenziva. Methyldopa působí antihypertenzivně, pravděpodobně prostřednictvím svého metabolitu α- methylnoradrenalinu, který působí agonisticky na α2 receptorech. Methyldopa také působí jako falešný prekurzor při syntéze katecholaminů (falešný neurotransmiter je méně účinný než noradrenalin) a snižuje plazmatickou hladinu reninu.

' } ] }, { text: '

92. den těhotenství si pacientka začala stěžovat na blíže nespecifikované problémy močového traktu a již týden trvající nevolnosti a bolesti v abdominální oblasti. Vyšetření moči odhalilo zvýšené hodnoty bilirubinu a u pacientky byl také pozorován sklerální ikterus. Změny vybraných biochemický parametrů, které byly kontrolovány v průběhu těhotenství, jsou zaznamenány v tabulce. Další biochemické parametry jako kyselina močová, alkalická fosfatáza, hemoglobin a krevní destičky byly v normě.

Parametr

Gestační čas (dny)

Fyziologická hodnota

92

95

101

116

266

274

AST (IU/L)

12-35

1800

2620

326

177

715

528

ALT (IU/L)

7-35

2415

2780

576

233

510

322

Bilirubin (μmol/L)

<17

113

220

378

326

127

118

aPTT (s)

22-36

60

42

37

33

PT (s)

11

19

13

14

13

95. den těhotenství byla pacientka hospitalizována s bolestmi v pravé horní abdominální oblasti, zjevným nažloutnutím kůže a sklerálním ikterem. Pacientce byl vysazen Dopegyt (TK 130/90) a byla provedena abdominální ultrasonografie, která prokázala normální žlučník, žlučovod, játra, pankreas i slezinu. Na základě koagulačních testů (aPTT a PT) byl pacientce nasazen i.m. vitamin K a to v dávce 10 mg po dobu 9 dnů.

Pacientka popřela i.v. užívání drog, větší počet sexuálních partnerů nebo předchozí transfúze krve. Screeningové vyšetření na jednotlivé typy hepatitid prokázalo prodělanou hepatitidu A, negativní hepatitidu B a C (výsledky na hepatitidu C nebyly při propuštění pacientky ještě známy). Pacientka byla 116. den těhotenství propuštěna se stálým tlakem (130/85) bez potřeby medikace.

', qs: [ { task: '

Jakou stanovíte pacientce z uvedených hodnot diagnózu a čím byla pravděpodobně způsobena?

', answer: '

Zvýšené hladiny jaterních transferáz (případně bilirubinu) poukazují na jaterní poškození, které je u pacientky pravděpodobně způsobeno podáváním methyldopy. Mechanismus hepatotoxického působení methyldopy není zcela objasněn, může být např. způsoben její abnormální biotransformací cytochromem P450. Dalším možným mechanismem je alergická reakce na její vznikající intermediální metabolity. Až u 5 % pacientů léčených methyldopou se objeví zvýšené hodnoty jaterních testů, ale hepatitida se objeví jen zřídka kdy (obvykle se příznaky rozvinou během prvních 3-4 týdnů). U predisponovaných pacientů je nutné během terapie methyldopou provádět kontrolní jaterní testy.

' }, { task: '

Proč byl pacientce podán vitamin K a za jak dlouho dobu můžeme očekávat nástup jeho účinku?

', answer: '

U pacientky byl diagnostikován deficit koagulačních faktorů, a proto byl pacientce podán vitamin K. Jedná se o další komplikaci, která může doprovázet sníženou funkci jater. Vitamin K je klíčový faktor pro syntézu některých koagulačních faktorů a je tedy nepostradatelný v procesu hemokoagulace. Jeho účinek nastupuje po 2-3 dnech aplikace.

' } ] }, { text: '

Pacientka byla opět hospitalizována 266. den těhotenství s pět dní trvající bolestí v epigastriu, sklerálním ikterem, tmavou močí a TK 130/80.

Pacientka měla 252. den zvýšený krevní tlak a tak jí její ošetřující lékař opět nasadil Dopegyt, protože špatně porozuměl výsledkům a myslel si, že pacientka je pozitivní na hepatitidu C. Těhotenství bylo ukončeno 274. den císařským řezem.

', qs: [ { task: '

V jakých dalších indikacích se používají látky strukturně podobné dopaminu, např. prekurzory dopaminu, a v jakých samotný dopamin?

', answer: '

Prekurzorem dopaminu je levodopa, která se používá k terapii Parkinsonismu. Další strukturně podobnou látkou je např. karbidopa, periferní inhibitor DOPA-dekarboxylázy, který se často kombinuje s levodopou pro dosažení jejich vyšších koncentrací v CNS. Dopamin patří mezi neselektivní sympatomimetika a je indikován např. jako součást komplexní terapie při akutním selhání perfuze v životně důležitých orgánech (především v ledvinách).

' } ] } ], biblio: ["

Smith GN, Piercy WN. Methyldopa hepatotoxicity in pregnancy: A case report. Am J Obstet Gynecol. 1995; 172(1): 222-224.

http://www.sciencedirect.com/science/article/pii/000293789590123X

"], editors: '04', index: 31 }, { title: 'I2cz', cathegory: 1, keywords: ["diabetes", "dyslipidemie", "hypertenze", "metformin", "glargin inzulín", "lispro inzulín", "ramipril", "hydrochlorothiazid", "kyselina acetylsalicylová", "amlodipin", "atorvastatin"], parts: [ { text: '

60-letý muž přišel na běžnou kontrolu ke svému praktickému lékaři. Už 20 let trpí diabetem mellitem II. typu, 10 let hypertenzí a 8 let dyslipidemií. V anamnéze má laserovou opakovanou fotokoagulaci z důvodů diabetické retinopatie a totální náhradu pravé kyčle pro závažnou osteoartritidu. Pacient přiznává častou nedůslednost v dodržování doporučené diety a nedostatek pohybu z důvodu bolestivosti v oblasti operovaného kloubu. Jako hlavní potíž udává otoky nohou, které se u něj objevují na konci dne. V posledních 18 měsících přibral 8 kg, což dává do souvislosti s inzulinoterapií. Snadno se unaví, ale neudává žádné potíže s dechem, bolesti na hrudi, palpitace nebo synkopy. Trávicí potíže nemá, ale často v noci vstává kvůli močení, které je obtížné.

Rodinná anamnéza: otec zemřel v 59 letech na infarkt myokardu, matka (83) má diabetes mellitus II. typu, dvě mladší sestry zdrávy, jedna trpí obezitou a poruchou glukozové tolerance.

Farmakologická anamnéza:

Siofor 1000, 1-0-1

Inzulin Lantus na noc, 18 j/hod

Inzulin Humalog 5 jednotek před jídlem

Amprilan H 5/25 mg, 1-0-0

Anopyrin 100, 1-0-1

Zorem 5 mg, 0-0-1

Atoris, 10 0-0-1

Udává 2-3 piva denně, nekouří

', qs: [ { task: '

Vyhledejte účinné látky v jednotlivých přípravcích, které pacient užívá. Vysvětlete mechanismus účinku jednotlivých léčiv.

', answer: '

Siofor obsahuje metformin, perorální antidiabetikum působící jako inzulinový senzitizér a zároveň tlumící glukoneogenezi v játrech.

Insulin Lantus je inzulin glargin, je dlouze působící analog inzulinu a používá se k náhradě bazální sekrece inzulinu.

Insulin Humalog je inzulin lispro, krátkodobý inzulin k zajištění potřeby při příjmu potravy.

Amprilan H obsahuje kombinaci účinných látek ramipril (ACE-inhibitor) a hydrochlorthiazid (thiazidové diuretikum, blokuje NA+ + Cl- kotransportér v distálním tubulu).

Anopyrin obsahuje kyselinu acetylsalicylovou, která má v této nízké dávce antiagregační účinky.

Zorem obsahuje amlodipin, kalciový blokátor dihydropyridinového typu, způsobuje především vasodilataci.

Atoris obsahuje atorvastatin, hypolipidemikum, které inhibuje aktivitu HMG-CoA reduktázy a tím snižuje tvorbu cholesterolu v játrech.

' }, { task: '

Přiřaďte jednotlivá léčiva k terapeutickým cílů, kterých chceme u tohoto pacienta dosáhnout.

', answer: '

Metformin a inzulinoterapie (glargin + lispro před jídlem) má snížit glykemii a zabránit následkům diabetu mellitu.

Ke zvládnutí dyslipidemie má pacient nasazenu jednu účinnou látku, atorvastatin.

Hypertenze je zaléčena kombinací látek – ramipril, hydrochlorthiazid a amlodipin.

Kyselina acetylsalicylová je u tohoto pacienta podávaná jako prevence trombembolické příhody.

' } ] }, { text: '

Fyzikální vyšetření: tělesná hmotnost 103 kg, výška 175 cm, TK 150/88 mmHg, TF 92/min, DF 17/min. Poslech odhalil systolický šelest nad levou hranou sterna s propagací do pravé karotidy. Břicho BPN. Prostata je mírně zvětšená. Je patrný otok na DKK distálně do poloviny lýtek a snížené čití na chodidlech obou nohou.

EKG záznam odpovídá hypertrofii levé komory.

Laboratorní výsledky z krve: kreatinin 97.2 µmol/L; draslík, 5.2 mmoI/L; bikarbonát 21 mmoI/L; glukóza na lačno 10,4 mmol/L, albumin, 37 g/L; celkový cholesterol, 5,4 mmol/L; HDL cholesterol, 0,73 mmol/L; triglyceridy, 2,8 mmol/L; LDL cholesterol, 3,99 mmol/L; hemoglobin A1c, 8.4 %; hemoglobin, 151 g/L; hematokrit, 45 %.

Laboratorní výsledky z moči: proteinurie (++), bez přítomnosti krve, buněk a dalších elementů. Kalkulovaná glomerulární filtrace je 74 mL/min.

', qs: [ { task: '

Které parametry nejsou u pacienta dostatečně zaléčeny? Jak byste upravili jeho farmakoterapii?

', answer: '

Tohoto pacienta vzhledem k jeho osobní i rodinné anamnéze můžeme považovat za rizikového.

Hodnoty z krevního vyšetření (glukóza na lačno 10,4 mmol/L, hemoglobin A1c, 8.4 %) ukazují na nedostatečnou kompenzaci diabetu. Nevíme, jak důkladně pacient provádí self-monitoring, vhodné by bylo zkontrolovat a případně upravit dávkování inzulinu nebo intenzivnější inzulinový režim s využitím inzulinové pumpy. Je potřeba pacienta motivovat k důslednějšímu dodržování dietních opatření a ke snížení tělesné hmotnosti, což úzce souvisí i s dyslipidemií, kterou také trpí.

Výsledky vyšetření lipidového spektra také nejsou zcela uspokojující, pacient má kombinovanou dyslipidemii. Celkový cholesterol je nad normou (5,4 mmol/L), ale důležitější jsou jednotlivé frakce. HDL cholesterol je nízký (0,73 mmol/L), naopak triglyceridy vysoké (2,8 mmol/L). Obě tyto frakce spolu většinou negativně korelují. LDL cholesterol, (3,99 mmol/L) je také je příliš vysoký. Velmi důležité je důslednější dodržování dietních opatření s omezením jak tuků, tak nadbytku sacharidů a alkoholu, a zvýšená fyzická zátěž. Je vhodné zvýšit dávku atorvastatinu a zvážit přidání některého z fibrátů do kombinace, zejména v případě, že pacient nebude schopen změnit svůj životní styl nebo by se jeho lipidové spektrum dokonce dále zhoršovalo.

Krevní tlak byl při vyšetření v pásmu mírné hypertenze. Cílem je dosáhnout u rizikového pacienta hodnot pod 135/80 mmHg. Řešením je zvýšit dávkování stávající terapie a případně přidat α1 antagonistu (pacient má také zvětšenou prostatu). V případě, že tato změna nebude mít dostatečný efekt, lze zaměnit thiazid za kličkové diuretikum i s ohledem na otoky nohou a mírně zvýšené hladiny draslíku.

' }, { task: '

Proteinurie bez výrazných změn v močovém sedimentu může být způsobena diabetickou nefropatiíí. Jaká léčiva zpomalují nástup a progresi tohoto ledvinného postižení?

', answer: '

ACE-inhibitory a sartany.

' }, { task: '

Pokud se bude postižení ledvin dále zhoršovat, které z používaných účinných látek mohou způsobit problémy?

', answer: '

Pokud se budou ledvinné funkce zhoršovat, problémem by mohl být metformin z důvodu zvýšeného rizika vzniku laktátové acidózy. Hladina draslíku (už nyní mírně zvýšená) by při poruše funkce ledvin zřejmě dále stoupala, což může být dále ovlivněno současně užívanými diuretiky anebo ACE inhibitory.

' } ] } ], biblio: ["

Tuttle K. Case study: A 60-year-old man with type 2 diabetes, hypertension, dyslipidemia and albuminuria. Advanced Studies in Medicine. 2005; 5(1A).

http://www.jhasim.com/files/articlefiles/pdf/asim_issue_5_1ap34-.pdf

"], editors: '03', index: 32 }, { title: 'I3cz', cathegory: 1, keywords: ["fibrilace síní", "TBC", "tuberkulóza", "INR", "warfarin", "rifampicin", "pyrazinamid", "infliximab", "cytochrom P450"], parts: [ { text: '

65-letá žena s fibrilací síní je léčená Lawarinem 10 mg/den (INR v rámci léčby v rozmezí 2,0-3,0).

Po zahájení léčby přípravkem Remicade se manifestovala latentní infekce Mycobacterium tuberculosis. Dále tedy byla pacientka léčena p.o. tabletami Benemicin (1 tbl 300 mg a 1 tbl 150 mg) a dále Pyrazinamidem (2 g/den).

Zpočátku terapie TBC byla antikoagulace nedostatečná, což bylo potvrzeno opakovaně stanovením INR. Po 3 měsících postupného zvyšování dávek warfarinu bylo dosaženo terapeutického INR. Díky častým návštěvám byla pacientka edukována co se týče antikoagulační léčby a změny dávek Lawarinu. Dávka Lawarinu musela být zdvojnásobena ve srovnání s počátečním stavem před započetím terapie TBC.

', qs: [ { task: '

Jaké jsou účinné látky přípravků Remicade, Benemicin, Lawarin a Pyrazinamid?

', answer: '

Remicade – infliximab

Benemicin - rifampicin

Lawarin -warfarin

Pyrazinamid – pyrazinamid

' }, { task: '

Jaký je mechanismus účinku těchto látek?

', answer: '

infliximab - chimerická lidská-myší monoklonální protilátka, která se váže s vysokou afinitou na rozpustné a transmembránové formy TNFα, ale ne na lymfotoxin α (TNFβ). U pacientů s revmatoidní polyartritidou byly v kloubech nalezeny zvýšené koncentrace TNFα. Po léčbě infliximabem došlo u pacientů k poklesu sérových hladin IL-6 a C-reaktivního proteinu.

warfarin – perorální antikoagulans. Brání syntéze koagulačních faktorů, závislé na vitaminu K. Z izomerů warfarinu je S-warfarin zhruba 5krát účinnější než R-warfarin. Účinek warfarinu je založen na jeho schopnosti zamezit redukci a působení vitaminu K při syntézách koagulačních faktorů II, VII, IX a X. V terapeutických dávkách warfarin brání syntéze koagulačních faktorů o 30 až 50 % a současně redukuje i jejich biologickou aktivitu.

rifampicin – baktericidní antibiotikum. Blokuje aktivitu DNA-dependentní RNA polymerázy. Působí silně baktericidně na mykobakteria tuberkulózy, atypická mykobakteria a mykobakteria lepry.

pyrazinamid – baktericidní chemoterapeutikum, inhibuje výstavbu buněčné stěny. Účinný pouze v kyselém pH.

' }, { task: '

Je aktivace latentní TBC očekávatelným nebo neočekávatelným nežádoucím účinkem při zahájení léčby přípravkem Remicade? Hlásili byste tento nežádoucí účinek státní autoritě?

', answer: '

Očekávatelný – viz SPC REMICADE - imunosupresivní účinek všech anti-TNF látek. Povinnost hlásit na SÚKL je u závažných očekávatelných a všech neočekávaných nežádoucích účinků. Závažné nežádoucí účinky definuje Zákon o léčivech (378/52007 Sb.) jako ty jež ohrozí pacienta na životě, vedou k hospitalizaci pacienta nebo jeho hospitalizaci prodlužují, vedou k trvalým následkům pro pacienta nebo jeho potomky.

' } ] }, { text: '

4 týdny po ukončení léčby rifampicinem se INR velmi zvýšilo (8,3), současně si pacientka stěžovala na podlitiny na rukou i nohou a pozorovala makroskopickou hematurii.

', qs: [ { task: '

Jaká je příčina této nežádoucí reakce?

', answer: '

Rifampicin je silným induktorem jaterních forem cytochromu P450 (CYP) a P-glykoproteinového transportéru. Mechanismus této interakce spočívá v indukci izoenzymů CYP2C9, CYP3A4, CYP1A2 a CYP2C19. Indukce CYP tedy zásadně ovlivní rychlost metabolizace mnoha léčiv, vč. warfarinu a sníží tak jeho terapeutické hladiny a potažmo i účinnost.

' }, { task: '

Jaký byste navrhli další terapeutický postup?

', answer: '

Přerušit podávání warfarinu na 1-2 dny, kontrolovat INR – v případě, že by INR hodnoty byly vyšší než 9, - vitamin K p.o./i.v.

' } ] }, { text: '

Terapeutické a stabilní INR bylo dosaženo po snížení dávky Lawarinu na 50 % (ve srovnání s dávkou užívanou v průběhu léčby TBC).

', qs: [ { task: '

Jaká preventivní opatření byste navrhovali provést před zahájení léčby kombinací rifampicin + pyrazinamid + warfarin?

', answer: '

Při zahájení i ukončení terapie rifampicinem častěji kontrolovat INR, měnit dávku warfarinu s ohledem na hodnoty INR.

' } ] } ], biblio: ["

Martins MA, Reis AM, Sales MF, Nobre V, Ribeiro DD, Rocha MO, Ribeiro AL. Rifampicin-warfarin interaction leading to macroscopic hematuria: a case report and review of the literature. BMC Pharmacol Toxicol. 2013;14(27).

http://www.biomedcentral.com/content/pdf/2050-6511-14-27.pdf

"], editors: '01', index: 33 }, { title: 'I4cz', cathegory: 1, keywords: ["hypoparatyreoidismus", "tyroidektomie", "parathormon", "kalciurie", "kalcitriol", "dihydrotachysterol"], parts: [ { text: '

Muž ve věku 74 let byl odeslán na neurologickou pohotovost se 7 dní progredující zmateností, letargií, dysartrií a slabostí. Stěžoval si také na zácpu, anorexii a bolesti břicha. Pacientův syn, který dorazil později, doplnil, že pacient je léčen pro hypertenzi a postoperační hypoparatyreoidismus po totální tyroidektomii v roce 1955. Užívá proto pravidelně přípravek Tachystin.

', qs: [ { task: '

Jaké jsou hlavní účinky parathormonu a jaké budou příznaky jeho nedostatku?

', answer: '

Parathormon zvyšuje kalcemii uvolňováním vápníku z kostí, zvýšením jeho reabsorpce v ledvinách a zvýšením jeho vstřebáváním v GIT (podrobněji viz učebnice fyziologie). Nedostatek se projeví hypokalcemií, kdy se rozvíjí typický tetanický syndrom, osteosklerózy, poškození zubů, kůže a adnex a očí.

' }, { task: '

Charakterizujte přípravek Tachystin, jakou účinnou látku obsahuje a jak působí?

', answer: '

Tachystin obsahuje dihydrotachysterol, syntetický analog vitamínu D. Díky své struktuře nemusí být aktivován v ledvinách za přítomnosti parathormonu. Zvyšuje kalcemii stejnými mechanismy jako kalcitriol.

' } ] }, { text: '

V anamnéze pacienta je také kalciurie zjištěná v roce 1995, pro kterou mu byla nasazena další látka.

', qs: [ { task: '

Jaká léčiva se mohou použít k terapii kalciurie? Jaký je mechanismus jejich účinku?

', answer: '

Při kalciurii se využívají thiazidová diuretika, která blokují Na+ - Cl- kontrasportér v distálním tubulu ledvin. Mechanismus jejich účinku při kalciurii je diskutován dvojí:

a) snížení intracelulárního Na+ způsobí zvýšenou výměnu sodíku za kalcium na bazolaterální membráně buněk tubulu

b) zvýšení koncentrace intracelulárního kalbindinu, což způsobí zvýšený vstup Ca2+ do tubulární buňky přes její apikální membránu a následuje už zmíněná výměna za sodík na bazolaterální straně tubulární buňky

' } ] }, { text: '

Při fyzikálním vyšetření byl pacient letargický, s pomalými pohyby a zastřenou řečí, TK 165/95 mmHg, dehydratovaný. Měl bradykinezi, snížené reflexy. Během hospitalizace byla zjevná zácpa a polyurie. CT mozku i EKG vyšetření, UTZ ledvin a RTG břicha byly bez patologie.

O něco později dorazily laboratorní výsledky:

Pacient

Referenční hodnoty

S-kalcium

3,70

2,1-2,6 mmol/l

S-fosfát

1,10

0,8-1,4 mmol/l

S-kreatinin

188,00

44-97 μmol/l

Exkrece kalcia močí za 24 hod

1,10

0,13-0,45 mmol/l kreatininu

Alkalická fosfatáza

1,08

< 1,5 μcat/l

Parathormon

< 3

12-72 ng/l

25 OH D3

46,00

> 75 nmol/l

1,25 di OH D3

14,00

40-155 pmol/l

PSA

1,36

< 3 μg/l

ACE

0,17

0,13 – 0,47 μcat/l

', qs: [ { task: '

Jak se mění plazmatické hladiny fosfátu při hypoparatyreóze?

', answer: '

Budou zvýšené z důvodu nižšího vylučování ledvinami.

' }, { task: '

Na jaký problém ukazují výsledky fyzikálního a laboratorního vyšetření?

', answer: '

Z laboratorních výsledků je významná především těžká hyperkalcemie, s čímž korelují neurologické známky (bardykineze, snížená výbavnost reflexů) a obstipace. Na úrovni ledvin je patrná porucha jejich funkce (snížená koncentrační schopnost), z čehož plyne polyurie a vysoké hladiny kreatininu. Zvýšená renální exkrece vápníku může vyústit až v nefrolitiázu a akutní renální selhání. Takto vysoká hladina vápníku v plazmě už může vyvolávat závažné arytmie, které se ale u tohoto pacienta neobjevily.

' }, { task: '

Stanovte diagnózu. Jak budete tohoto pacienta léčit?

', answer: '

Stav pacienta odpovídá předávkování vitamínem D. Terapie bude spočívat v hydrataci kontinuální infuzí, podávání furosemidu a metyprednisolonu. V některých případech je zkoušena také léčba bisfosfonáty, které zabraňují resorpci kostí. Samozřejmostí je omezení příjmu vitamínu D do srovnání stavu. Terapie se zahajuje vždy při hladině plazmatického kalcia nad 3,5 mmol/L, při hladině nad 3 mmol/L se řídíme klinickým stavem pacienta.

' } ] }, { text: '

Dva dny po přijetí a zahájení terapie se stav pacienta začal zlepšovat a komunikace s ním se zlepšila. Přiznal, že omylem užíval poslední 4 – 8 týdnů 75 kapek (2,5) ml dihydrotachysterolu místo své obvyklé dávky 50 kapek (1,7 ml). Příčina této změny v užívání nebyla vysvětlena. Během několik dnů se hladina vápníku vrátila do normálních mezí a s tím ustoupily i ostatní příznaky. Pacientovi byl před propuštěním dihydrotachysterol zaměněn za kalcitriol, který ale užíval pouze dva týdny a opět se vrátil ke staršímu léčivu.

', qs: [ ] } ], biblio: ["

Jensterle M, Pfeifer M, Sever M, Kocjan T. Dihydrotachysterol intoxication treated with pamidronate: a case report. Cases Journal. 2010;3(78).http://www.ncbi.nlm.nih.gov/pmc/articles/PMC2861028/

"], editors: '03', index: 35 }, { title: 'I5cz', cathegory: 1, keywords: ["CHOPN", "ipratropium", "salbutamol", "hydrokortizon", "ampicilin", "prednizolon"], parts: [ { text: '

71-letý pacient s chronickou limitací výdechového proudění vzduchu (chronical airflow limitation - charakteristická pro CHOPN, tj. chronickou obstrukční plicní nemoc) byl hospitalizován z důvodu zhoršující se dyspnoe. Úvodní vyšetření naznačilo infekční původ zhoršení jeho stavu. RTG vyšetření plic prokázalo přítomnost pravostranného zápalu plic.

Pacient byl léčen následujícími léky: ipratropium bromid (0.5 mg) a salbutamol (5 mg) 5x denně inhalačně, hydrokortizon 200 mg 4x denně intravenózně, ampicilin 500 mg 4x denně intravenózně a prednizolon 30 mg denně perorálně.

V anamnéze pacienta byla mimo jiné zaznamenána nykturie (dvakrát za noc po dobu zhruba 6 měsíců), ale byl bez historie obtíží při močení (např. slabého proudu moči či retence moči).

', qs: [ { task: '

Které z uvedených léčivých látek jsou používány jako bronchodilatancia? Jaké jsou jejich mechanismy účinku?

', answer: '

salbutamol – selektivní β2 adrenergní agonista, který působí na hladké svalstvo dýchacích cest a vede k jeho relaxaci a bronchodilataci.

ipratropium bromid - mechanismus účinku= anticholinergní působení, antagonizuje účinky acetylcholinu na muskarinových receptorech. Bronchodilatace po inhalaci je vyvolána místním (nikoli systémovým) působením takových koncentrací léčivé látky, které jsou schopny vyvolat anticholinergní účinek na hladkou svalovinu bronchů.

' }, { task: '

Kam řadíme ostatní uvedené léčivé látky a jaké jsou mechanismy jejich účinků?

', answer: '

ampicilin – antibiotikum penicilinové řady, působí prostřednictvím inhibice biosyntézy mukopeptidu buněčné stěny. Ampicilin má široké spektrum baktericidní účinnosti proti četným grampozitivním a gramnegativním aerobním a anaerobním bakteriím, ale je rozkládán beta-laktamázami.

hydrokortizon – patří mezi glukokortikoidy, jedná se o přirozený hormon kůry nadledvin, který má nejen glukokortikoidní, ale i mineralokortikoidní účinky. Mechanismus účinku spočívá v inhibici fosfolipázy A v metabolismu kyseliny arachidonové, výsledkem jsou protizánětlivé, antialergické a antiproliferativní účinky.

prednizolon – patří také mezi glukokortikoidy, má stejný mechanismus účinku jako hydrokortison.

' } ] }, { text: '

Po čtyřech dnech se u něj projevil dyskomfort v podbřišku a nebyl schopný se vymočit. Vyšetření odhalilo velký hmatatelný močový měchýř a středně zvětšenou prostatu. Po zavedení močového katetru odteklo 600 ml moči.

', qs: [ { task: '

Které z podaných léčiv mohlo retenci moči vyvolat a jaké byste navrhl řešení?

', answer: '

Ipratorpium bromid, obtíže při močení, případně až retence moči je typický anticholinergní účinek, vyplývající z blokády M-receptorů.

Ukončení léčby ipratropium bromidem.

' } ] }, { text: '

Léčba ipratropium bromidem byla ukončena, obtíže ustoupily a po 3 dnech mohl být močový katetr vyjmut.

', qs: [ { task: '

Jaké znáte další typické nežádoucí účinky anticholinergik?

', answer: '' }, { task: '

V jakých dalších indikacích jsou využívány anticholinergně působící látky?

', answer: '

Anticholinergní látky jsou využívány i:

' }, { task: '

Znáte nějaké další skupiny léčiv, které kromě hlavního mechanismu účinku vykazují i anticholinergní účinky?

', answer: '' } ] } ], biblio: ["

Lozewicz S. Bladder outflow obstruction induced by ipratropium bromide. Postgraduate Medical Journal. 1989; 65: 260 – 261.

http://pmj.bmj.com/content/65/762/260.full.pdf+html

"], editors: '05', index: 37 }, { title: 'I6cz', cathegory: 1, keywords: ["fibrilace", "heparin", "rhabdomyolýza", "simvastatin", "amiodaron", "furosemid", "ranitidin", "spironolakton", "valsartan", "kyselina acetylsalicylová", "bisoprolol", "pravastatin", "rosuvastatin", "fluvastatin"], parts: [ { text: '

Osmdesátiletý muž přišel na pohotovost s pocity bolesti a slabosti v proximální části dolních končetin. Anamnéza pacienta zahrnovala inzulínem nedostatečně kompenzovaný DM II s počínající diabetickou retinopatií, dále ischemickou kardiomyopatii, neléčený adenokarcinom prostaty, hypercholesterolémii, hypertenzi, chronické renální selhání na podkladě diabetické nefropatie a vaskulární onemocnění.

Po dobu posledních 3 let byl léčen Simgalem v dávce 40 mg/den bez zaznamenání nežádoucích účinků. Devět měsíců před přijetím byly naměřeny následující hodnoty: kreatinkináza (CK) 77 UI/L (Nl < 90 UI/L), sérový kreatinin 2,2 mg/dL. Čtyři dny před přijetím byla zjištěna fibrilace síní v souvislosti s otokem dolních končetin. U pacienta tak byla zahájena léčba Cordarone 200 mg/den, nízkomolekulárním heparinem a Furonem 80 mg/den.

FA: inzulín, Ranisan, Verospiron, Valsacor, Anopyrin, Bisocard

', qs: [ { task: '

Zjistěte, co obsahují uvedené léčivé přípravky za účinné látky, jaký je jejich hlavní mechanismus účinku a v jaké indikaci jsou u pacienta používány?

', answer: '' } ] }, { text: '

Vyšetření při přijetí: bez horečky, TK 130/70, bilaterální otok nohou, slabost proximálních svalů DK (Gowerův znak). Laboratorně byla zjištěna rhabdomyolýza (CK 29760 IU/L (Nl < 400 IU/L), laktát-dehydrogenáza 1093 IU/L (Nl 98-192 IU/L), AST 426 IU/L (Nl 6-33 IU/L), ALT 296 IU/L (Nl 14-63 IU/L), troponin 0,13 ng/ml (Nl < 0,08 ng/ ml), porušení renálních funkcí (kreatinin 2,97 mg/dL, urea 296 mg/dL), hyperkalémie (6,2 mEq/L), myoglobinurie (1393 ng/mL, Nl < 15 ng/mL). TSH v normě, neg. adenovirus, coxsackie virus, toxoplasma.

', qs: [ { task: '

Které z použitých léčiv, respektive jejich vzájemná kombinace, mohlo vyvolat bolest a slabost dolních končetin? Pokuste se s využitím dostupných farmakologických databází a vlastních znalostí zjistit příčinu uvedeného stavu.

', answer: '

Simvastatinem indukovaná rhabdomyolýza, která se rozvinula po přidání amiodaronu k dříve dobře tolerované chronické terapii statiny. Amiodaron je inhibitor CYP 3A4, což má za následek zvýšení plazmatických hladin léčiv, které se přes tuto formu enzymu metabolizují, tím pádem i simvastatinu.

' }, { task: '

Které léčivo by bylo vhodnější pro tohoto pacienta?

', answer: '

Kombinace simvastatinu a amiodaronu je možná pouze do dávky 20 mg simvastatinu denně. Pro pacienta by v tomhle případě byly vhodnější statiny, které nejsou metabolizovány přes CYP 3A4:

' }, { task: '

Vyberte konkrétní léčivou látku z této skupiny vhodnou pro řešení problému:

  1. generický název - zvažte vhodnost výběru z hlediska prospěšnosti pro pacienta, poměr prospěšnosti a rizika (“benefit-to-risk ratio”): nežádoucí účinky, kontraindikace, lékové interakce
  2. lékovou formu (zvažte rychlost nástupu a dobu trvání účinku, způsob podání)
  3. firemní název - zvažte dostupnost, cenu
  4. dávkování (zvažte intenzitu potíží, stav pacienta, ochotu a schopnost pacienta ke spolupráci “compliance”)
  5. předepište LP
  6. poučte pacienta (způsob užívání LP, možné NÚ)
', answer: '

a) Např. rosuvastatin- nemetabolizuje se výhradně přes CYP 3A4, ale též přes CYP 2C9

b) Per os – jiné LF nejsou v ČR na trhu

c) Např. Rosucard, Tintaros, Mertenil – všechny plně hrazené a obchodované v ČR

d) Doporučené dávkování je 1 x denně, počáteční dávka se volí dle hladiny cholesterolu, začíná se dávkou 5 mg nebo 10 mg, maximální denní dávka je 40 mg

e) Rx.

Tintaros 10 mg

POR TBL FLM 28x10 MG

Exp. Orig. No. I (unam)

D.S. 1x denně 1 tabletu

f) Rosuvastatin lze užívat kdykoliv v průběhu dne, s jídlem nebo nalačno. V průběhu terapie je nutné dodržovat nízkocholesterolovou dietu, protože je utlumena pouze endogenní produkce cholesterolu. Dále je doporučeno zvýšit přísun nenasycených mastných kyselin, omezit přísun nasycených, zvýšit přísun vlákniny, ovoce a zeleniny, zvážit konzultaci s nutričním terapeutem ohledně stravovacích návyků pacienta. Z dalších režimových opatření je vhodné doporučit zvýšení pohybové aktivity a zanechání kouření. Pacienty je třeba upozornit, aby okamžitě hlásili nevysvětlitelné bolesti svalů nebo svalovou slabost a křeče, zvláště pokud jsou spojeny se zvýšenou teplotou a nevolností.​

' }, { task: '

Co je to Gowerův znak?

', answer: '

Gowerův znak je popisován tak, že pacient k tomu, aby se postavil, musí použít i horní končetiny. Příčinou je nedostatečná svalová síla ve stehenní a kyčelní oblasti. Pro tento manévr je pacient umístěn na podlahu na všechny 4 končetiny (pozice kočky) a z jeho dosahu jsou odstraněny předměty, kterými by si mohl pomoci k tomu, aby se postavil.

Převzato z: Wikipedia

' } ] } ], biblio: ["

MAROT, Astrid, et al. Concomitant use of simvastatin and amiodarone resulting in severe rhabdomyolysis: a case report and review of the literature. Acta Clinica Belgica. 2011; 66 (2): 134-136.

http://search.proquest.com/docview/962427399?accountid=16531

"], editors: '037', index: 39 }, { title: 'I7cz', cathegory: 1, keywords: ["cisplatina", "ifosfamid", "mesna", "ondansetron", "dexametazon", "metoklopramid", "aprepitant"], parts: [ { text: '

67 letá žena s maligním smíšeným mezodermálním ovariálním tumorem podstoupila chirurgickou resekci a následnou cytostatickou léčbu. Laboratorní vyšetření před zahájením chemoterapie byla v normě včetně hladin albuminu (3,5 g/dL). Během prvních tří cyklů terapie byla pacientce podána cisplatina 20 mg/m2 spolu s ifosfamidem 1,5 g/m2, denně v 90 min. infuzi současně s aplikací mesny ve 4 po sobě jdoucích dnech.

', qs: [ { task: '

Jaké je fyziologické rozmezí plazmatických hladin albuminu?

', answer: '

Fyziologická plazmatická koncentrace albuminu: 3,4–5,4 g/dL

' }, { task: '

Jaké jsou mechanismy účinku a indikace cisplatiny a ifosfamidu?

', answer: '

Obě látky lze zařadit mezi alkylační látky. Jsou podávány ve formě proléčiv a v organizmu se mění enzymaticky (ifosfamid) nebo neenzymaticky (cisplatina) na aktivní metabolity. Konkrétně, ifosfamid je látka podobná cyklofosfamidu s oxizafosforinovou strukturou. Je metabolizován systémem cytochromu P450 na 4-hydroxyifosfamid a tautomer aldoifosfamid, který se samovolně rozpadá na akrolein (urotoxický) a alkylačně působící ifosfamid-yperit. Mechanismem účinku je alkylace bazí nukleových kyselin. Indikace ifosfamidu zahrnují různé typy maligních tumorů včetně nádorů varlat, vaječníků, prsu pankreatu aj.

' }, { task: '

Jaké jsou běžné nežádoucí účinky cytostatik způsobené jejich nízkou selektivní toxicitou?

', answer: '

Nízká selektivní toxicita „klasických“ cytostatik vede k negativnímu působení na všechny buňky lidského těla s rychlou proliferací. Mezi běžné nežádoucí účinky cytostatik, tak řadíme myelosupresi (leukocytopenie, trobocytopenie, anemie), GIT toxicitu (průjmy, malabsorpce, ulcerace, krvácení), kožní toxicitu, alopecii a reprodukční toxicitu. Jejich mutagenní mechanismus účinku je důvodem vzniku sekundárních tumorů.

' }, { task: '

Jaké jsou charakteristické nežádoucí účinky cisplatiny a ifosfamidu, kromě těch spadajících pod otázku č. 3?

', answer: '

Cisplatina je jedním z nejsilnějších emetogenů a působí nefrotoxicky. Ifosfamid, stejně jako cyklofosfamid, může vyvolat hemoragickou cystitidu a může vést k různým známkám neurotoxicity, jako jsou závratě, halucinace až koma.

' }, { task: '

Proč byla pacientce podána mesna?

', answer: '

Mesna je cytoprotektivní látka. Snižuje nefrotoxicitu oxizafosforinů inaktivací jejich 4-hydroxy- metabolitů a akroleinu.

' }, { task: '

Jaké jiné chemoprotektivní látky kromě mesny znáte?

', answer: '

Dexrazoxan chrání před kardiotoxicitou antracyklinových cytostatik, leukovorin (kys. folinová, citrovorum faktor) snižuje toxicitu metotrexátu a amifostin snižuje nefrotoxicitu cisplatiny.

' }, { task: '

Souvisí nějak plazmatická hladina albuminu s toxicitou některého z použitých léčiv?

', answer: '

Nízké hladiny albuminu jsou rizikovým faktorem pro neurotoxicitu ifosfamidu.

' } ] }, { text: '

V rámci premedikace byl pacientce aplikován ondansetron 32 mg a dexametazon 10 mg a po čtyři dny po ukončení každého z cyklů chemoterapie užívala pacientka 8 mg dexametazonu (ráno), metoklopramid 20 mg (každých 8 hodin) a ondansetron. Pacientka byla bez neurologických symptomů v průběhu prvních tří cyklů, ale rozvinula se neutropenická horečka a závažná nauzea se zvracením v průběhu třetího cyklu.

', qs: [ { task: '

Jaké jsou mechanismy účinku a indikace ondansetronu, dexametazonu a metoklopramidu?

', answer: '

dexametazon - syntetický glukokortikoid se silným antiflogistickým účinkem. Interaguje s intracelulárními receptory a ovlivňuje transkripci některých genů. Je bez mineralokortikoidní aktivity a patří mezi glukokortikoidy s dlouhým biol. poločasem. Mezi jeho endokrinní indikace patří exoftalmus. Z neendokrinních indikací se používá při terapii alergických reakcí, autoimunitních onemocnění, nefrotického syndromu, astmatu, kožních onemocnění, některých typů nádorů a jako adjuvantní terapie nevolností a zvracení při nádorové terapii.

ondansetron -antagonista serotoninových 5-HT3 receptorů na periferních i centrálních neuronech zapojených do procesů emeze. Je indikován k terapii a prevenci pooperační a cytostatiky indukované nevolnosti a zvracení.

metoklopramid - D2 antagonista v chemorecepční spouštěcí zóně a antagonista 5-HT3 receptorů. Mimo jeho centrálních účinků, metoklopramid zvyšuje GIT motilitu díky antidopaminergnímu efektu v kombinaci se zvýšenou aktivitou acetylcholinu v GIT. Výsledkem je zrychlené žaludeční vyprazdňování a zvýšení tonu dolního jícnového svěrače. Metoklopramid je indikován u zvracení indukovaného cytostatiky nebo radioterapií a používá se jako symptomatická terapie nevolností a zvracení různé etiologie.

' }, { task: '

Jsou dávky antiemetik v souladu s doporučeným dávkováním v SPC?

', answer: '

dexametazon – dávkování je značně variabilní bez jasně dané maximální dávky. Obvyklá úvodní dávka pro parenterální podání je v rozmezí 0,5 – 20 mg/den.

ondansetron – doporučená denní dávka dle SPC je v rozmezí od 8 do 32 mg.

metoklopramid – maximální doporučená denní dávka je 30 mg nebo 0.5 mg/kg tělesné hmotnosti.

' }, { task: '

Jakou změnu terapie byste volili pro omezení rizika neutropenické horečky a jak byste řešili nevolnost, která se objevila po 3. cyklu?

', answer: '

Nežádoucí účinky léčiv a jejich toxicita jsou většinou na dávce závislé. Proto snížení dávky by mohlo přinést i snížení rizika rozvoje těchto negativních jevů. Je však potřeba myslet na fakt, že se snížením dávky se může snížit i účinnost terapie. V tomto případě se zdá, že účinnost antiemetické terapie není dostatečná, a proto ani snížení dávky cytostatik pravděpodobně nepovede k omezení nevolností a emeze. Zvýšit dávku antiemetik již není možné (ondansetron na maximu doporučené dávky) proto připadá v úvahu záměna léčiv v antiemetickém režimu.

' } ] }, { text: '

Dávky cytostatik a antiemetik (premedikace i post-chemoterapeuticky) byly upraveny pro 4. cyklus chemoterapie následovně: cisplatina 20 mg/m2, ifosfamid 1 g/m2 denně se současnou aplikací mesny ve dnech 1-4. Plazmatická hladina albuminu byla u pacientky 3,6 g/dL. V premedikaci byl podán aprepitant 125 mg a dexametazon 10 mg. Post-chemoterapeutický režim se skládal z 80 mg aprepitantu společně s 8 mg dexametazonu aplikovaných ráno po dobu dvou dnů.

', qs: [ { task: '

Jaký je mechanismus účinku aprepitantu?

', answer: '

aprepitant - selektivním antagonistou substance P na neurokininových receptorech (NK1).

' } ] }, { text: '

Pacientka stále pociťuje nevolnost, ale nezvrací. Po dokončení aplikace mesny 3. den 4. cyklu se u pacientky náhle rozvíjí ifosfamidem indukované kóma. Infuze ifosfamidu byla okamžitě ukončena. Kóma přetrvává u pacientky méně než 24 h a infuze methylenové modři nebylo nutné aplikovat. Pacientka je bez trvalých neurologických nálezů. Zajímavé je, že analýza moči neodhalila žádné erytrocyty v průběhu celé hospitalizace.

', qs: [ { task: '

Proč došlo pravděpodobně k rozvoji ifosfamidem indukovaného komatu? Jaká interakce dvou léčiv může být příčinou?

', answer: '

S největší pravděpodobností se jedná o metabolickou interakci mezi ifosfamidem a aprepitantem na úrovni cytochromu P450. Aprepitant je středně silným inhibitorem CYP3A4 a ifosfamid je substrátem tohoto enzymu. Kombinace látek tedy vede ke zvýšení plazmatických hladin ifosfamidu a k rozvoji neurotoxických symptomů. Absence urotoxických symptomů je pravděpodobně díky dostatečnému množství mesny použité v prevenci tohoto typu nežádoucího účinku

' }, { task: '

Jak může metylenová modř snížit toxicitu ifosfamidu?

', answer: '

Mechanismus vzniku ifosfamidem indukované neuropatie je nejasný. Jedna z teorií předpokládá vznik metabolitů ifosfamidu blokujících mitochondriální dýchací řetězec. Metylenová modř je schopná obnovit aktivitu blokovaných mitochondrií. Současné znalosti o účinnosti metylenové modři v terapii ifosfamidem indukované neuropatie jsou rozporuplné.

' } ] } ], biblio: ["

Shindorf, ML, Manahan, KJ, Geisler, JP. The interaction of ifosfamide and aprepitant in gynecologic malignancies. Gynecologic Oncology Reports. 2013; 6: 34-35.

http://www.sciencedirect.com/science/article/pii/S2211338X13000458

"], editors: '0', index: 40 }, { title: 'I8cz', cathegory: 1, keywords: ["ramipril", "kyselina acetylsalicylová", "atorvastatin", "sertralin", "glycerol-trinitrát", "amlodipin", "metoprolol", "angioedém"], parts: [ { text: '

76letá žena byla přijata do nemocnice z důvodu bolesti v krku a levostranné bolesti obličeje trvající 24 hodin. Ty byly spojovány s otokem jazyka, který se zhoršil v průběhu posledních 4 hodin, což způsobilo neschopnost mluvit a zvýšenou salivaci. Pacientka popřela jakékoli pocity svědění nebo tepla v oblasti otoku a zamítla i jakýkoliv nedávný kontakt s alergeny. Relevantní osobní anamnéza zahrnovala akutní infarkt myokardu s následným koronárním bypassem, hypertenzi, hypercholesterolémii, depresi a lupénku. Užívala následující léky - ramipril, kyselinu acetylsalicylovou, atorvastatin, sertralin a glycerol-trinitrát ve spreji. Neměla žádnou známou alergii na léky.

', qs: [ { task: '

Jakým mechanismem účinku působí uvedená léčiva? Objasněte, v jaké indikaci jsou u dané pacientky použita.

', answer: '

ramipril – inhibitor ACE, enzymu, který v plazmě a v tkáních katalyzuje přeměnu angiotenzinu I na aktivní vazokonstrikční látku angiotenzin II a štěpí aktivní vazodilatátor bradykinin. Snížená tvorba angiotenzinu II a inhibice štěpení bradykininu vede k vazodilataci a poklesu krevního tlaku. Ramipril byl u pacientky použit jako antihypertenzivum a ke kardioprevenci/ sekundární prevenci po infarktu myokardu.

kyselina acetylsalicylová, patří do skupiny nesteroidních analgetik/antiflogistik, má analgetické, antipyretické a antiflogistické účinky. Jako mechanismus účinku se popisuje inhibice cyklooxygenázy, vedoucí k útlumu biosyntézy prostanoidů - prostaglandinu E2, prostacyklinu (prostaglandinu I2) a tromboxanu A2. Acetylsalicylová kyselina má ireverzibilní inhibiční účinek na agregaci trombocytů. V kazuistice není uvedená dávka, ale pravděpodobně byla podávána v nižší dávce jako antitrombotikum ze skupiny inhibitorů agregace trombocytů, podáváno z důvodu sekundární prevence AIM a po prodělaném koronárním bypassu.

atorvastatin – hypolipidemikum, k léčbě hypercholesterolémie, selektivní a kompetitivní inhibitor HMG-CoA reduktázy, tento enzym katalyzuje přeměnu z 3-hydroxy-3-methyl-glutaryl-koenzymu A na mevalonát, což je prekurzor sterolů včetně cholesterolu.

sertralin – antidepresivum, selektivní inhibitor zpětného vychytávání serotoninu (SSRI), k léčbě deprese.

glycerol-trinitrát ve spreji – vasodilatans, antianginózní účinek; v buňkách hladkého svalstva denitrován za vzniku oxidu dusnatého, který pravděpodobně reaguje s receptorem obsahujícím sulfhydrylovou skupinu na molekule enzymu guanylcyklázy s jeho následnou aktivací. Výsledným efektem je vzestup cGMP s následnou relaxací hladkého svalstva cévní stěny na všech úrovních krevního řečiště. Primárním následkem je dilatace venózního řečiště s následným zvýšením žilní kapacity, snížením venózního návratu (preload).

' } ] }, { text: '

Při vyšetření vitálních funkcí byly zjištěny následující hodnoty: teplota 38,2 °C, krevní tlak 161/75 mmHg, srdeční frekvence 108 tepů/min, frekvence dýchání 16 dechů/min a saturace kyslíkem 98 %. Pacientka měla levostranný unilaterální otok jazyka a hmatné, citlivé sublinguální a submandibulární žlázy na levé straně. Vzhledem k těmto zjištěním byl diagnostikován intraorální absces. Krevní testy ukázaly zvýšené hladiny zánětlivých markerů, což naznačovalo, bakteriální infekci. Byla zahájena antibiotická léčba a podán intravenózní hydrokortizon.

Respirační funkce pacientky se zhoršily, což si vyžádalo naléhavou intubaci. Po přezkoumání snímků z následného CT krku, které neprokázalo žádné známky infekčního ložiska, ale odhalilo závažné otoky dýchacích cest, byla veškerá antimikrobiální terapie ukončena. Druhý den hospitalizace se otok jazyka zvětšil tak, že jazyk vyčníval z úst a pacientka dostala dvě jednotky čerstvé mražené plazmy. Třetí den začal otok ustupovat. Pacientka byla extubována čtvrtý den a den poté byla propuštěna z JIP. Autoimunitní sérologická vyšetření byla negativní a hladiny komplementu, inhibitoru C1 esterázy (C1 EI) a tryptázy byly normální. Výsledky mikrobiologických vyšetření byly negativní.

', qs: [ { task: '

Stanovte diagnózu pacientky, které z podávaných léčiv ji coby nežádoucí účinek mohlo způsobit?

', answer: '

Byla stanovena následující diagnóza - angioedém způsobený sekundárně užíváním ACE inhibitorů (ACEl) .

V případě angioedému způsobeného ACEI, je diagnóza založena na přítomnosti následujících parametrů – současné podávání ACEI, neexistence alternativních diagnóz, včetně jiných příčin angioedému a rychlý ústup symptomů po vysazení ACEI.

U sledované pacientky byla provedena doporučená specifická biochemická vyšetření zahrnující měření hladiny složek komplementu a inhibitoru C1 esterázy (C1- EI) za účelem vyloučení jiných příčin angioedému (dědičný nebo získaný angioedém), které jsou spojené s nedostatkem nebo dysfunkcí C1- EI (proteázy, která se podílí na regulaci bradykininu).

Žádný z nálezů také nenaznačuje hypersenzitivní reakci - pacientka byla bez projevů svědění, kopřivky a popřela jakýkoliv nedávný kontakt s alergeny.

' }, { task: '

Je daný nežádoucí účinek očekávatelný?

', answer: '

Ano – viz SPC přípravků obsahujících ramipril (například SPC přípravku Tritace) - u pacientů léčených ACE inhibitory včetně ramiprilu byl hlášen výskyt angioedému, angioedém patří k závažným nežádoucím účinkům.

' }, { task: '

Vysvětlete, jakým mechanismem se ACEI podílí na vzniku angioedému.

', answer: '

ACEl inhibují odbourávání bradykininu a způsobují lokální zvýšení jeho hladiny a následně vazodilataci. To může vést k extravazaci tekutin do podkoží a vzniku angioedému.

' }, { task: '

Navrhněte další antihypertenzní léčbu – uveďte, která léčiva nejsou vhodná.

', answer: '

Pokračování v léčbě ACEI po epizodě angioedému je kontraindikováno, není ani žádný důvod pro nasazení jiného inhibitoru ACE, protože se jedná o class effect, tzn. že jde o nežádoucí účinek celé lékové skupiny. Změna medikace na blokátory receptoru angiotenzinu II (AIIRB) je také kontroverzní, protože i AIIRB jsou spojovány s výskytem angioedému. AIIRB by teoreticky měly představovat bezpečnou volbu, protože nezasahují do metabolismu bradykininu, ale data ze studií na zvířatech ukazují, že může existovat vztah mezi použitím AIIRB a nárůstem bradykininu v tkáních a byly popsány i případy u pacientů s opakovanými epizodami angioedému po změně léčby na AIIRB.

' } ] }, { text: '

Pacientce bylo doporučeno přerušit léčbu ACEI, a její antihypertenzivní terapie byla změněna na podávání amlodipinu a metoprololu.

', qs: [ { task: '

Do jaké skupiny patří uvedená léčiva a jakým mechanismem účinku působí?

', answer: '

amlodipin - je blokátor kalciového kanálu ze skupiny dihydropyridinu selektivní s hlavním vlivem na cévy a inhibuje tak membránový transport kalciových iontů do srdce a hladkého svalstva cévních stěn. Mechanismus antihypertenzivního účinku amlodipinu vyplývá z jeho přímého relaxačního účinku na hladké svalstvo cévní stěny.

metoprolol - je kardioselektivní betablokátor (tj. blokuje β1 receptory v myokardu při řádově nižších dávkách než je potřebné k blokádě β2-receptorů), bez částečně agonistických vlastností (tj. vnitřní sympatomimetické aktivity).

Je indikován i jako udržovací léčba po infarktu myokardu, u pacientů se suspektním nebo potvrzeným infarktem myokardu snižuje mortalitu, především snížením rizika náhlé smrti. Je to alespoň z části důsledek preventivního působení metoprololu na vznik komorové fibrilace.

' } ] } ], biblio: ["

Kloth N, Lane AS. ACE inhibitor-induced angioedema: a case report and review of current management. Crit Care Resusc. 2011;13(1):33-37.

http://www.cicm.org.au/CICM_Media/CICMSite/CICM-Website/Resources/Publications/CCR%20Journal/Previous%20Editions/March%202011/10_2011_Mar_CR-ACE.pdf

"], editors: '05', index: 41 }, { title: 'I9cz', cathegory: 1, keywords: ["omeprazol", "enoxaparin", "esomeprazol", "metamizol", "venlaflaxin", "hypertriglyceridemie", "gemfibrozil"], parts: [ { text: '

55-letá žena (výška 160 cm, váha 70 kg), byla přijata do nemocnice z důvodu hrudního traumatu a zlomeniny stehenní a hlezenní kosti v důsledku dopravní nehody. Pacientčina anamnéza byla běžná; užívala pouze omeprazol k léčbě žaludečního vředu. Popírá užívání alkoholu a nelegálních drog. Výsledky laboratorních testů, včetně jaterních testů, byly po přijetí do nemocnice normální. Ultrazvukové vyšetření břicha (Doppler), prováděné v době přijetí za účelem vyšetření traumatu, prokázalo mikrocholelitiázu bez rozšíření žlučových cest.

Přibližně 1 měsíc po zahájení pobytu v nemocnici, během něhož jí byl podáván enoxaparin, esomeprazol, metamizol a venlafaxin, u ní byla na základě rutinního krevního vyšetření diagnostikována hypertriglyceridemie (239 mg /dl) s normálními hladinami cholesterolu, AST a ALT.

', qs: [ { task: '

Do jakých farmakoterapeutických skupin patří uvedená léčiva? Popiště jejich mechanismy účinku.

', answer: '

enoxaparin – antikoagulans, nízkomolekulární heparin, je charakterizován vysokou anti-Xa aktivitou a nízkou anti-IIa (antitrombinovou) aktivitou.

esomeprazol – patří mezi tzv. antiulceróza/antiskretorika, specifický inhibitor protonové pumpy (H+/K+-ATPáza) v parietální buňce.

metamizol – patří mezi analgetika, antipyretika, tzv. deriváty pyrazolonu; mechanismus účinku podobně jako u jiných analgetik nebyl podrobně objasněn. Inhibuje cyklooxygenázu – zahrnuje inhibici syntézy prostaglandinů (PGE1 a PGE2) a reverzibilní inhibici agregace trombocytů.

venlafaxin- antidepresivum, SNRI – inhibitor zpětného vychytávání serotoninu a noradrenalinu.

' }, { task: '

Navrhněte léčivo/farmakoterapeutickou skupinu léčiv vhodnou k léčbě diagnostikované hypertriglyceridémie.

', answer: '

Léčiva snižující plazmatickou hladinu triacylglycerolů - fibráty, případně deriváty kyseliny nikotinové.

' } ] }, { text: '

Pacientka poté začala užívat gemfibrozil v dávce 600 mg denně.

', qs: [ { task: '

Do jaké farmakoterapeutické skupiny patří gemfibrozil? Popiště jeho mechanismus účinku.

', answer: '

Patří mezi hypolipidemika – fibráty.

Účinky na úpravu hladiny lipidů u lidí jsou zprostředkovány aktivací receptoru typu alfa aktivovaného peroxizomovým proliferátorem (PPAR) - agonisté jaderných PPAR-α receptorů.

Prostřednictvím aktivace PPAR zvyšuje lipolýzu a eliminaci aterogenních částic bohatých na triglyceridy z plazmy aktivací lipoprotein-lipázy a snížením tvorby apoproteinu CIII. Aktivace PPAR také vyvolává zvýšení syntézy apoproteinů AI a AII.

Výše zmíněné účinky vedou ke snížení frakcí o velmi nízké a nízké hustotě (VLDL a LDL) a ke zvýšení frakce o vysoké hustotě (HDL).

' }, { task: '

Jaké je standardní dávkování gemfibrozilu?

', answer: '

Podle SPC - standardní dávka je v rozsahu 900 mg až 1200 mg.

' }, { task: '

Zjistěte ATC kód skupiny fibrátů. Uveďte další zástupce ze skupiny fibrátů.

', answer: '

C10AB - například fenofibrát, ciprofibrát, bezafibrát, klofibrát.

' } ] }, { text: '

Jeden měsíc poté si pacientka stěžovala na nevolnost a malátnost. Byla provedena analýza krve, včetně vyšetření jaterních funkcí s následujícími výsledky: hladina alaninaminotransferázy (ALT) 1142 UI/L (normální < 40 UI/L), aspartátaminotransferázy (AST) 656 UI/L (normální < 40 UI/L), gama-glutamyltransferázy (GMT) 430 UI/L (normální < 50 UI/L), alkalické fosfatázy (ALK) 331 UI/L (normální < 126 UI/L), hladina celkového bilirubinu 2,70 mg/ml (fyziologické rozmezí 0.25-1 mg/ml) a přímého bilirubinu 2,21 mg/ml (fyziologické rozmezí 0–0,25 mg/ml). Ultrazvuk břicha prokázal perzistenci mikrocholelitiázy bez rozšíření žlučových cest a dalších anatomických abnormalit.

Sérologické testy na akutní virovou hepatitidu (HAV, HBV, HCV, CMV, a EBV) byly všechny negativní, což vyloučilo virovou etiologii.

', qs: [ { task: '

Co mohlo být příčinou náhlého zhoršení jaterních funkcí?

', answer: '

Pravděpodobně se jedná se o hepatotoxicitu po podání gemfibrozilu.

' } ] }, { text: '

Náhlá elevace jaterních enzymů ukazovala na možný nežádoucí účinek po podání léčiva. V daném okamžiku byla medikace pacientky následující: enoxaparin, esomeprazol, metamizol, gemfibrozil a venlafaxin. Jako nejpravděpodobnější byla stanovena diagnóza akutní toxická hepatitida – vyvolaná gemfibrozilem, protože gemfibrozil byl poslední lék přidaný do medikace pacienta.

Potenciální virová hepatitida byla vyloučena z důvodu negativních sérologických testů. S ohledem na pacientčinu anamnézu byla také vyloučena možnost alkoholové hepatitidy.

', qs: [ { task: '

Je daný nežádoucí účinek u gemfibrozilu očekávatelný?

', answer: '

Ano očekávatelný, dle informací o nežádoucích účincích v SPC.

' }, { task: '

Navrhněte vhodné řešení situace.

', answer: '

Ukončit terapii gemfibrozilem. V případě nutnosti další léčby hypertriglyceridémie, by tato léčba měla být v první řadě pomocí diety a cvičení, v případě nutnosti znovupodání fibrátů, je třeba pečlivě monitorovat ukazatele jaterních funkcí.

' } ] }, { text: '

Následně bylo zastaveno podávání gemfibrozilu. Nebyly provedeny žádné jiné terapeutické změny. Již druhý den po přerušení terapie gemfibrozilem se jaterní testy a klinické příznaky zlepšily. Ačkoli bylo zlepšení jaterních testů evidentní už druhý den, k plnému zotavení došlo až o 2 týdny později.

S ohledem na uvedená zjištění navrhujeme, aby byla zvýšená hladina triglyceridů v krvi řešena v první řadě prostřednictvím diety a cvičení a léčba fibráty přidávána až v případě, že dieta není dostatečně efektivní.

U dalších léků, které daná pacientka užívala (enoxaparin, esomeprazol a venlafaxin) byly také hlášeny případy hepatitidy, byť jen zřídka. V našem případě však okamžité zlepšení jaterních funkcí (jen 2 dny po ukončení léčby gemfibrozilem) podporuje hypotézu, že se jednalo o diagnózu gemfibrozilem indukované akutní toxické hepatitidy.

', qs: [ ] } ], biblio: ["

Domínguez Tordera P, Comellas Alabern JF, Ronda Rivero F. Gemfibrozil hepatotoxicity: a case report. Int J Clin Pharm. 2011;33(5):730-7322.

http://link.springer.com/article/10.1007%2Fs11096-011-9550-x#page-1

"], editors: '05', index: 42 }, { title: 'I10cz', cathegory: 1, keywords: ["kolchicin", "familiární středomořská horečka", "acebutolol", "ramipril", "spironolakton", "molsidomin", "kyselina acetylsalicylová", "amoxicilin", "klaritromycin", "omeprazol"], parts: [ { text: '

Muž (76 let) byl hospitalizován s horečkou, opakovaným zvracením, průjmem a abdominální bolestí. U pacienta byla v minulosti diagnostikována a genetickým vyšetřením potvrzena familiární středomořská horečka a již 6 let užívá kolchicin v dávce 1,5 mg denně, což výrazně snížilo četnost záchvatů nemoci. Muž má dále v osobní anamnéze esenciální arteriální hypertenzi, infarkt myokardu, chronické renální selhání (kreatinin 1,7 mg/dl, clearance 38 ml/min dle Cockrofta-Gaulta, proteinurie 0,1 g/24 h), cholecystektomii a intestinální obstrukci řešenou kolektomií. Kromě kolchicinu užívá dále acebutolol, ramipril, spironolakton, molsidomin a nízké dávky kyseliny acetylsalicylové.

', qs: [ { task: '

Jaké jsou mechanismy účinku výše uvedených léčiv? V jakých indikacích je pacient užívá?

', answer: '

acebutolol kardioselektivní (β1) betablokátor s mírnou vnitřní sympatomimetickou aktivitou a antiarytmickým působením. Užívá se v terapii hypertenze, srdečních arytmií, anginy pectoris a jako sekundární prevence po prodělaném infarktu myokardu, což je kromě hypertenze pravděpodobná indikace, pro kterou acebutolol užívá pacient z kazuistiky.

ramipril inhibitor ACE používaný v terapii hypertenze, srdečního selhání a onemocnění ledvin (např. diabetické nefropatie). Inhibitory ACE se využívají také v sekundární prevenci po infarktu myokardu. Pacient ramipril užíval pravděpodobně kvůli hypertenzi, renální insuficienci a stavu po infarktu myokardu.

spironolakton draslík šetřící diuretikum. Užívá se v léčbě srdečního selhání a arteriální hypertenze, při ascitu, edémech a při hypokalemii. Důvod užívání u našeho pacienta je nejasný, snad jako adjuvans terapie hypertenze nebo proti možným otokům způsobeným renálním postižením. Otázkou je hyperkalemie, která se při kombinaci ACE inhibitorů a K+‑šetřících diuretik může vyskytnout.

molsidomin v játrech se mění na metabolit, který uvolňuje oxid dusnatý. NO aktivuje guanylátcyklázu a vyvolává vazodilataci venózního řečiště a snižuje preload srdce, srdeční práci a spotřebu kyslíku. Užívá se v terapii anginy pectoris.

acetylsalicylová kyselina v nízkých dávkách má především antiagregační účinky. Je to způsobeno ireverzibilní inhibicí cyklooxygenázy trombocytů, která fyziologicky vytváří tromboxan A2 v procesu hemostázy. Je široce užívána jako sekundární prevence u ischemických aterosklerotických onemocnění (IM, CMP, angina pectoris apod.)

kolchicin inhibuje tvorbu tubulinových řetězců (dělicí vřeténko, cytoskelet), a tím tlumí dělení buněk a migraci buněk. Užívá se především u akutního dnavého záchvatu, u kterého snižuje fagocytózu urátových krystalů leukocyty a jejich migraci do postiženého kloubu. V ČR je momentálně registrován pouze pro toto použití, jinde ve světě má ještě další indikace, např. různá zánětlivá onemocnění (vaskulitidy, Behçetova choroba, familiární středomořská horečka apod.)

' } ] }, { text: '

Před časem byla pacientovi zjištěna erozivní gastritida s infekcí H. pylori. Byla zahájena sedmidenní léčba amoxicilinem (1000 mg po 12 hod), klaritromycinem (500 mg po 12 hod) a omeprazolem (20 mg 2x denně).

', qs: [ { task: '

Je eradikace H. pylori nastavena správně?

', answer: '

Zvolená kombinace léčiv je jednou z možných, je však vždy potřeba přihlédnout individuálně k pacientově další medikaci, což zde pravděpodobně nebylo provedeno. Z hlediska pacientovy medikace tedy léčba nebyla zvolena správně (podrobnosti viz Odpověď 4 a 5).

' }, { task: '

Jaké další kombinace léčiv se používají k eradikaci H. pylori?

', answer: '

Jiné kombinace léčiv mohou vypadat takto:

Dalším potenciálně využitelným léčivem s antimikrobiálním účinkem proti H. pylori je zásaditý salicylan bismutitý, který ale v této indikaci momentálně v ČR není registrován.

' } ] }, { text: '

Čtvrtý den eradikační léčby se u pacienta objevuje abdominální bolest, krvavý průjem a zvracení, pro něž je hospitalizován. Den po ukončení terapie gastritidy se objevuje horečka (38,5 °C) a dehydratace. Výsledky vyšetření: pancytopenie (hemoglobin 6,8 g/dl, trombocyty 88 000/mm3, leukocyty 2,4 × 103/mm3, neutrofily 1,3 × 103/mm3), zvýšený CRP (83 mg/l; normál < 5), mírná hepatická cholestáza, zvýšené lipázy (209 U/l; normál < 60), proteinurie (0,78 g/24 h), makroskopická hematurie. Dále je přítomna hyponatrémie, zvýšený sérový kreatinin a močovina. Vyšetření na hepatitidu B a C, HIV, legionelu a brucelu je negativní, krevní a močová kultivace je také negativní.

', qs: [ { task: '

Jaká je pravděpodobná příčina pacientova stavu? Které orgány jsou nejvíce poškozeny?

', answer: '

Příčinou je pravděpodobně intoxikace kolchicinem. Při této otravě jsou nejčastěji postiženy ledviny, játra, krvetvorba, slezina a GIT. Příznaky poškození všech těchto orgánů nacházíme i u našeho pacienta. Dále vykazuje mírné poškození pankreatu.

' } ] }, { text: '

Pacientovi byla snížena dávka kolchicinu na 0,5 mg/den. Dochází ke spontánnímu odeznění symptomů bez další medikace (kromě parenterální rehydratace). 14 dní po zahájení eradikační léčby se objevila u pacienta alopecie. Po zotavení z intoxikace je během 4 měsíců postupně dávka kolchicinu zvýšena opět na 1,5 mg denně.

', qs: [ { task: '

Které léčivo pravděpodobně způsobilo vznik intoxikace? Jaký je mechanismus této interakce?

', answer: '

Pravděpodobnou příčinou je léčba klaritromycinem. Toto antibiotikum je inhibitor CYP3A4 a P‑glykoproteinu. CYP metabolizuje kolchicin na demekolcin (demethylovaný kolchicin) a další metabolity, které jsou pomocí P-glykoproteinu secernovány z hepatocytů do žluče. Většina kolchicinu je vylučována biliárně, pouze malá část je vylučována močí (max. 15 %). Mírně mohl ke stavu pacienta přispět také jeho vyšší věk a preexistující renální postižení.

' }, { task: '

Která kombinace léčiv pro eradikaci H. pylori by pro pacienta byla vhodnější?

', answer: '

Vhodnější by byl režim s amoxicilinem a metronidazolem/tinidazolem. Tyto látky neovlivňují funkci CYP ani P‑glykoproteinu a lze je podávat u pacientů s renální dysfunkcí (u amoxicilinu při clearance kreatininu nad 30 ml/min není třeba úprava dávky).

' } ] } ], biblio: ["

Rollot, Florence, et al. Acute colchicine intoxication during clarithromycin administration. Annals of Pharmacotherapy. 2004; 38(12): 2074-2077.

http://aop.sagepub.com/content/38/12/2074.short

"], editors: '06', index: 43 }, { title: 'I11cz', cathegory: 1, keywords: ["leukémie", "prednizolon", "dexametazon", "metotrexát", "cyklofosfamid", "vinkristin", "daunorubicin", "asparagináza", "ranitidin", "leukovorin", "mesna", "dexrazoxan", "omeprazol"], parts: [ { text: '

U 15letého chlapce (42 kg, 1,55 m, povrch těla 1,36 m2) byla diagnostikována Philadelphia chromozom pozitivní ALL.

Po úvodní léčbě kortikoidy (prednizolon 60 mg/m2 celková denní dávka, dny 1–7) byla podána kombinace léčiv složená z dexametazonu per os (6 mg/m2) (dny 8–28), vysokodávkového metotrexátu (MTX) i.v. v průběhu 24 h (5 g/m2) (den 8), cyklofosfamidu i.v. (1 g/m2) (den 9), vinkristinu i.v. (1,5 mg/m2) (dny 8, 15, 22 a 29), daunorubicinu i.v. (40 mg/m2) (dny 15, 22, 29), L-asparaginázy i.v. (10,000 U/m2) (dny 12, 15, 18, 22, 25, 29, 32 a 35) a přípravku Ranisan 150 mg.

', qs: [ { task: '

Jaké jsou mechanismy účinku a indikace podávaných léčiv?

', answer: '

prednizolon - slabý syntetický glukokortikoid s nízkou mineralokortikoidní aktivitou. Mechanismus účinku látky spočívá ve vazbě na glukokortikoidní intracelulární receptory a ovlivnění transkripce příslušných genů po translokaci do jádra a dále v ovlivnění příslušných intracelulárních signálních drah. Prednizolon je určen především pro terapii zánětlivých, alergických a autoimunitních onemocnění (kolagenóza, lymfomy, revmatoidní artritida, ulcerózní kolitida aj.).

dexametazon - syntetický glukokortikoid se silným antiflogistickým účinkem bez mineralokortikoidní aktivity. Mechanismus účinku je shodný s prednizolonem, biologický poločas je ve srovnání s prednizolonem cca 3x delší. Mezi endokrinní indikace patří exoftalmus. Neendokrinní indikace zahrnují alergie a anafylaktické reakce, autoimunitní onemocnění, nefrotický syndrom, astma, kožní onemocnění, onkologické indikace a adjuvantní terapie nevolností a zvracení v průběhu chemoterapie.

metotrexát - cytostatikum ze skupiny antimetabolitů. Je to kompetitivní antagonista kyseliny listové. Blokuje přeměnu kys. listové na tetrahydrofolát inhibicí enzymu dihydrofolát reduktázy. Je indikován u různých typů malignit (akutní leukémie, lymfomy, karcinom prsu, hlavy a krku …) a v nízkých dávkách se používá v imunosupresivní terapii.

cyklofosfamid - alkylační látka ze skupiny oxazafosforinů. Jedná se o proléčivo, které je rychle konvertováno na aktivní metabolity jaterními enzymy CYP. Hlavním účinným metabolitem je 4-hydroxycyklofosfamid. Naproti tomu vzniká z cyklofosfamidu působením enzymů stejného systému i toxický metabolit akrolein. Aktivní metabolity se spojují pevnými kovalentními vazbami s nukleofilními atomy nukleových bazí a blokují tak fyziologickou roli nukleových kyselin. Indikace jsou podobné jako u MTX, zahrnují řadu nádorových a autoimunitních onemocnění.

vinkristin - přírodní alkaloid z barvínku, který interferuje s tvorbou dělícího vřeténka u proliferujících buněk. Váže se k tubulinovým dimerům, které jsou základním strukturním prvkem všech intracelulárních mikrotubulů. Mezi indikace náleží různé typy leukémií, myelomy, lymfomy, solidní tumory a idiopatická trombocytopenická purpura.

daunorubicininterkalační cytostatikum ze skupiny cytostatických antibiotik. Přesný mechanismus účinku daunorubicinu není znám. Předpokádá se interakalace do struktury nukleových kyselin, inhibice topoizomeráz a produkce volných kyslíkových radikálů. Daunorubicin se používá v terapii myelogenních a lymfocytárních leukémií.

asparagináza (krisantaspáza) - nádorové buňky lymfoblastické leukémie a non-Hodkinova lymfomu postrádají asparagin syntetázovou aktivitu a jsou závislé na přívodu asparaginu z extracelulárního prostředí. L- asparagináza rozkládá extracelulární asparagin a glutamin (alternativní substrát pro syntézu asparaginu) na kys. asparagovou a glutamovou. Asparagin je nepostradatelný pro proteosyntézu a buněčnou proliferaci.

ranitidin - selektivní H2 antihistaminikum indikované při pyróze, gastrických vředech, refluxní ezofagitidě a u Zollinger-Ellisonova syndromu.

' }, { task: '

Je dávkování léčiv v souladu s dávkami v SPC?

', answer: '

prednizolon – úvodní dávka by měla být v rozmezí 15-60 mg

dexametazon – dávkování je přísně individuální a velmi variabilní bez uvedené maximální dávky

metotrexát – u vysokodávkové terapie se používá 1 g/1m2 těl. povrchu

cyklofosfamid – pro vysokodávkovou intermitentní terapii hematologických malignit se používá dávka v rozmezí 800 -1600 mg/m2 těl. povrchu s intervaly mezi jednotlivými dávkami 21-28 dní

vinkristin – aplikovaná dávka by měla být v rozmezí od 1,4 do 2 mg/m2 těl. povrchu a týdenní dávka by neměla přesáhnout 2 mg

daunorubicin – maximální kumulativní dávka v průběhu celé terapie u dětí by neměla přesáhnout 300 mg/m2 těl. povrchu

L-asparagináza – obvyklá dávka je 6 000 jednotek/m2 těl. povrchu

ranitidin – není doporučené podávání dětem pod 16 let věku, maximální denní dávka pro dospělého je 150 mg

' }, { task: '

Co jsou chemoprotektivní látky a proč je používáme společně s cytostatiky. Jaká chemoterapeutika byla pravděpodobně pacientovi podána i když v anamnéze nejsou zmíněna?

', answer: '

Chemoprotektiva jsou látky, které snižují toxicitu cytostatik. V případě použitých cytostatik přichází v úvahu použití následujících chemoprotektiv:

leukovorin - (kys. folinová, citrovorum faktor) jedná se o aktivní formu kys. listové (5-formyl tetrahydrofolát). V případě použití vysokodávkového MTX je použití leukovorinu vysoce pravděpodobné, protože tento terapeutický přístup bez použití chemoprotektiva vede k významným symptomům toxicity MTX.

mesna – snižuje nefrotoxicitu cyklofosfamidu, respektive je urotoxického metabolitu akroleinu, který inaktivuje stejně jako 4-hydroxymetabolity cyklofosfamidu a jiných oxazafosforinů.

dexrazoxan – chrání před kardiotoxicitou antracyklinových cytostatik. Pravděpodobně vyvazuje volné atomy železa v myokardu a brání tak produkci volných kyslíkových radikálů.

' }, { task: '

Ranitidin byl pacientovi nasazen pravděpodobně pro omezení rozvoje nežádoucího účinku některého z léčiv. Kterého léčiva a nežádoucího účinku se to týká?

', answer: '

Vysoké dávky glukokortikoidů jsou spojeny s celou řadou nežádoucích účinků včetně indukce gastrických vředů. Pravděpodobným mechanismem zapojeným do provokace gastroduodenálních vředů je blok fosfolipázy A2, což následně vede k poklesu produkce mukoprotektivních prostaglandinů a převládnutí erozivních vlivů na sliznici GIT.

' }, { task: '

Jaké terapeutické postupy mohou vést ke snížení toxicity metotrexátu?

', answer: '

Alkalizace moči urychluje eliminaci metotrexátu ledvinami. Pro tyto účely lze použít hydrogenuhličitan sodný.

' } ] }, { text: '

V počátku terapie bylo podání MTX bez jakýchkoliv komplikací. Bylo provedeno rutinní stanovení MTX hladin 72h od aplikace s výsledkem 0,09 μmol/l. U pacienta nebyly zaznamenány žádné komplikace.

Před zahájením druhého cyklu terapie byl Ranisan (150 mg) zaměněn za Gasec (20 mg/den, p.o.) kvůli nedostatečné účinnosti (epigastrická bolest).

', qs: [ { task: '

Co je to TDM a kdy se používá?

', answer: '

TDM je metoda klinické farmakokinetiky založená na měření koncentrací léčiv v biologických vzorcích a jejich porovnávání se známými referenčními hodnotami. Metoda je použitelná pro optimalizaci dávkování u léčiv s úzkým terapeutickým rozmezím (antiarytmika, antiepileptika), stejně jako pro diferenciální diagnostiku u atypické reakce na léčivo nebo k ověření adherence pacienta k předepsané léčbě.

' }, { task: '

Jaké je účinná látka přípravku Gasec, jaký je její mechanismus účinku, a jaké jsou její výhody oproti ranitidinu?

', answer: '

Účinnou látkou přípravku Gasec je omeprazol, ireverzibilní inhibitor protonové pumpy. Trvalá blokáda H+/K+ ATPázové aktivity v parietálních buňkách vede ke snížení produkce kyseliny chlorovodíkové. Omeprazol je proléčivo aktivované v kanalikulech parietálních buněk. Na rozdíl od H2 antihistaminik inhibuje omeprazol produkci HCl bez ohledu na stimulační signál.

' } ] }, { text: '

Po druhém cyklu terapie vysokodávkovým MTX, byly hladiny MTX stanovovány denně pro monitoring jeho toxicity (viz. Tab. 1). Přestože byly podány vysoké dávky leukovorinu (až 40 mg/ 4x denně) zůstávaly hladiny MTX zvýšené po několik dní. U pacienta se objevily symptomy toxicity MTX (závažná mukozitida). Hodnoty sérového kreatininu a jaterní testy, až na laktátdehydrogenázu, byly v normě jak v průběhu terapie MTX, tak i po ní. V průběhu celého období bylo stanovováno opakovaně i pH moči, které se pohybovalo vždy nad hodnotami 7,0 (v rozmezí 7,0 – 8,5).

', qs: [ { task: '

Jaký je mechanismus interakce mezi MTX a omeprazolem?

', answer: '

Pravděpodobným mechanismem interakce mezi MTX a omeprazolem je blokáda exkrece MTX. MTX je vylučován v ledvinách mechanismem aktivního transportu. Pro transportní mechanismus jsou nezbytně nutné protony tvořené H+/K+ ATPázou, která může být inhibovaná omeprazolem. Z tohoto důvodu je prodloužen eliminační poločas MTX, snížena jeho clearance a mohou se rozvinout symptomy toxicity.

' }, { task: '

Proč byly sledovány také hladiny sérového kreatininu, jaterní testy a hodnoty pH moči?

', answer: '

Mezi běžné nežádoucí účinky MTX patří nefro- a hepatotoxicita, proto bylo provedeno stanovení markerů ledvinných funkcí a jaterního poškození. MTX je těžce rozpustný při nízkém pH, a proto s klesajícím pH moče hrozí riziko jeho precipitace a následného ledvinného selhání. Z tohoto důvodu bylo sledováno i pH moče.

' } ] }, { text: '

V okamžiku zastavení podávání omeprazolu začaly symptomy intoxikace MTX rychle ustupovat včetně mukozitidy. Tři týdny od kompletního vyloučení MTX byl podán další cyklus léčiva, tentokrát bez abnormalit v MTX clearance. Hodnoty plazmatický koncentrací MTX u všech tří cyklů terapie lze nalézt v Tabulce č. 1.

Tabulka 1: Plazmatické hladiny MTX (μmol/L) v průběhu vysokodávkové terapie

Čas v hodinách po aplikaci MTX

24487296120144168192216240
Cyklus 1-0.290.09-------
Cyklus 29110.263.41.270.981.170.970.440.260.15
Cyklus 335.40.290.16-------

červeně – plazmatické hladiny po ukončení podávání omeprazolu

', qs: [ ] } ], biblio: ["

Bauters, TGM, Verlooy, J, Robays, H, Laureys, G. Interaction between methotrexate and omeprazole in an adolescent with leukemia: a case report. Pharm World Sci. 2008; 30: 316-318.

http://link.springer.com/article/10.1007%2Fs11096-008-9204-9

"], editors: '0', index: 44 }, { title: 'I12cz', cathegory: 1, keywords: ["cyklosporin A", "metylprednizolon", "perindopril", "gliklazid", "digoxin", "simvastatin"], parts: [ { text: '

70-letý pacient (BMI 26) po transplantaci ledviny byl 3 roky léčen přípravkem Sandimmun Neoral 100 mg/ml, 1,75 ml/den (2,5 mg/kg /den), Medrol 4 mg/den, Prestarium Neo forte cps 1-0-0, Diaprel MR 1-0-0, Digoxin 0,125 1-0-0 a Zocor 40 mg 0-0-1.

', qs: [ { task: '

Jaké jsou účinné látky ve výše uvedených přípravcích a co je jejich hlavním mechanismem účinku?

', answer: '

Sandimmun Neoral 100 – cyklosporin A (CsA)– antagonista kalcineurinu (cytozolický protein řazený mezi fosfatázy, jehož aktivita je řízena hladinou intracelulárního kalcia prostřednictvím kalmodulinu). Kalcineurin aktivovaný kalmodulinem působí defosforylaci několika nukleárních faktorů, které jsou řazeny do skupiny nukleárních faktorů aktivovaných T-lymfocytů (NFAT), a tím jejich translokaci do jádra. Tam působením NFAT dochází k aktivaci transkripce a syntézy regulačních cytokinů v T-buňkách.

Mezi nejvýznamnější cytokiny, jejichž aktivaci CsA tlumí, patří interleukin-2 (IL-2), IL-3, IL-4, granulocyty/makrofágy stimulující faktor (GM-CSF), interferon-γ (IFN-γ), tumor nekrotizující faktor-α (TNF-α) a IL-17. Zdá se, že cyklosporin blokuje klidové lymfocyty ve fázi G0 nebo G1 buněčného cyklu a tlumí antigenem vyvolané uvolňování lymfokinů z aktivovaných T lymfocytů.

Medrol - metylprednizolon – syntetický glukokortikoid se silným protizánětlivým účinkem. Má větší protizánětlivý účinek než prednizolon a menší tendenci vyvolávat retenci sodíku a tekutin. Relativní účinek metylprednizolonu vůči hydrokortizonu je nejméně 4 ku 1.

Prestarium Neo forte tbl - perindopril - inhibitor angiotenzin konvertujícího enzymu (ACE). ACE je exopeptidáza, která umožňuje konverzi angiotenzinu I na vazokonstrikční angiotenzin II a zároveň způsobuje rozklad vazodilatační látky bradykininu na neúčinný heptapeptid. Inhibice ACE vede ke snížení angiotenzinu II v plazmě, což vede ke zvýšení aktivity reninu v plazmě (inhibicí negativní zpětné vazby uvolnění reninu) a snížení sekrece aldosteronu.

Diaprel MR – gliklazid - derivát sulfonylurey ze skupiny antidiabetik; snižuje hladiny glykemie stimulací sekrece inzulínu z β-buněk Langerhansových ostrůvků.

Digoxin - digoxin - kardiotonikum, inhibitor NA+/K+-ATPázy.

Zocor - simvastatin, inhibitor HMG-CoA reduktázy, inhibuje syntézu cholesterolu.

' }, { task: '

Vyhledejte možné interakce přípravku Sandimmun Neoral s ostatní medikací.

', answer: '

cyklosporin - simvastatinrhabdomyolýza, hepatotoxicita v důsledku inhibice CYP3A4 a OATP1B1.

cyklosporin – metylprednizolon – metabolizmus prostřednictvím CYP3A4, kompetitivní inhibice, hyperkorticismus.

cyklosporin – digoxin – inhibice P-glykoproteinu – hladiny digoxinu mohou být zvýšeny, toxicita digoxinu.

' }, { task: '

Které z ostatních interakcí mohou být potenciálně nebezpečné?

', answer: '

digoxin – metylprednizolon - hypokalemie po glukokortikoidech (případné zvýšení rizika, kdyby pacient bral diuretikum) - riziko toxicity dioxinu.

perindopril – digoxin – některé ACE inhibitory mohou snížit clearance digoxinu.

Antagonistický účinek glukokortikoidů a antihypertenziv.

' } ] }, { text: '

Z důvodu systémové aspergilové infekce byl ordinován Sporanox cps, 100 mg 2x denně. Biochemická vyšetření krve před nasazením Sporanoxu: kreatinin 1.39 mg/dL, urea 88 mg/dL, CK 19 IU/mL (norma: 25-90), LDH 245 IU/ml (70-240), ctrough hladina cyklosporinu 244 ug/L.

V den nasazení Sporanoxu byla dávka Sandimmun Neoral snížena na 125 mg/d z důvodu známé inhibice metabolismu cyklosporinu itrakonazolem. O 5 dní později byla dávka ještě snížena na 2 x 25 mg/d, protože hladina cyklosporinu byla 435 ug/L i při snížené dávce 125 mg/den.

O 2 týdny později byl pacient přijat pro malátnost a celkovou svalovou slabost. Pacient nebyl schopen samostatné chůze bez opory, delší stání působilo také problémy. Laboratorní vyšetření odhalilo: kreatinin 2,19 mg/dL, urea 178 mg/dL, kreatinkináza CK 554 IU/ml, LDH 386 IU/mL, cyclosporin A ctrough 144 ug/L.

', qs: [ { task: '

Tyto symptomy ukazují na nežádoucí účinek jednoho z léčiv, kterého?

', answer: '

Simvastatin – svalová slabost, rhabdomyolýza.

' }, { task: '

Co je příčinou tohoto náhlého nežádoucího účinku?

', answer: '

Interakce itrakonazol – simvastatin. Inhibice CYP3A4 podáním itrakonazolu.

' }, { task: '

Co byste mohli udělat pro prevenci tohoto nežádoucího účinku?

', answer: '

Snížit dávku statinu/vysadit jej.

' }, { task: '

Jaké je terapeutické referenční rozmezí cyklosporinu po orgánové transplantaci u pacientů 1 rok po transplantaci?

', answer: '

250-400 ng/mL

' } ] }, { text: '

Sporanox i Zocor byl vysazen. Během následujícího týdne se svalová slabost ještě zhoršila, CK se zvýšila až na 14000 IU/mL a následně během 10 dní klesla až na normální hodnoty. Svalová síla se začala pomalu zlepšovat, pacient rehabilitoval a po 2 týdnech byl schopen samostatné chůze s oporou.

', qs: [ ] } ], biblio: ["

Maxa JL1, Melton LB, Ogu CC, Sills MN, Limanni A. Rhabdomyolysis after concomitant use of cyclosporine, simvastatin, gemfibrozil, and itraconazole. Ann Pharmacother. 2002;36(5):820-823.

http://www.ncbi.nlm.nih.gov/pubmed/11978159

"], editors: '01', index: 45 }, { title: 'I13cz', cathegory: 1, keywords: ["cushingův syndrom", "mifepriston", "dexametazonový supresní test", "ketokonazol", "karbergolin", "bisoprolol", "amlodipin", "kosyntropin", "metyrapon", "etomidát", "enalapril", "mifepriston", "mitotan"], parts: [ { text: '

Žena 28 let byla hospitalizována z důvodu arteriální hypertenze a počínajícího diabetu. Přestože byla hypertenze léčena přípravky Afiten, Bisocard a Berlipril, kontrola jejího krevního tlaku nebyla dostatečná.

', qs: [ { task: '

Jaké jsou účinné látky obsažené v užívaných přípravcích, jaké jsou jejich mechanismy účinku a indikace?

', answer: '

Afiten - amlodipin – je blokátor vápníkových kanálů (L- typ) s dihydropyridinovou strukturou. Navozuje vazodilataci inhibicí transmembránového influxu vápníku v hladké svalovině cév a má antihypertenzivní a antiangiózní účinky (snižuje afterload a dilatuje koronáry). Používá se především v terapii esenciální hypertenze a angíny pectoris.

Bisocard - bisoprolol – je kardioselektivní β1 antagonista bez vnitřní sympatomimetické a membrány stabilizující aktivity. Používá se k terapii hypertenze, angíny pectoris a stabilního srdečního selhání (má jen mírný pozitivně inotropní efekt) se sníženou funkcí levé komory v kombinaci s diuretiky, inhibitory ACE a digoxinem.

Berlipril - enalapril – je proléčivo, které je v organizmu hydrolyzováno na enalaprilát s inhibičními účinky na ACE. Indikacemi jsou hypertenze a srdeční selhání.

' } ] }, { text: '

Pacientka neužívala žádná jiná léčiva. V průběhu posledních dvou let se její tělesná hmotnost zvýšila o 24 kg. Při příjmu byla aktuální tělesná hmotnost 92 kg, body mass index – 35,2 kg/m2 a krevní tlak 170/110 mmHg. Vyšetření ukazuje obezitu centrálního typu s „měsíčkovitým“ obličejem a býčí šíjí, proximální myopatii a „papírovou“ kůži s modřinami. Centrální obezita výrazně kontrastuje s tenkými končetinami, na břichu jsou patrné strie.

', qs: [ { task: '

O jakém onemocnění byste na základě popsaných symptomů uvažovali?

', answer: '

Jedná se o typické symptomy Cushingova syndromu.

' }, { task: '

Jaká biochemická vyšetření byste použili pro potvrzení Vaší hypotézy?

', answer: '

Pro ověření hypotézy Cushingova syndromu by měly být stanoveny hladiny ACTH, CRH a kortizolu.

' } ] }, { text: '

Plazmatické koncentrace kortizolu byly 5,3 μg/dL (referenční rozmezí: 8,0–25,0 μg/dL) ráno a 3,2 μg/dL v noci. Hodnoty ranních i nočních hladin adrenokortikotropního hormonu (ACTH) byly nedetekovatelné (< 5 pg/mL, referenční rozmezí: 10–80 pg/mL). Množství volného kortizolu v moči za 24 h bylo 12, respektive 14 μg/den u dvou nezávislých odběrů (referenční rozmezí: 20–90 μg/den). Plazmatické koncentrace dehydroepiandrosteronu byly sníženy (52 μg/dL; referenční rozmezí: 80–450 μg/dL). Jak noční plazmatické koncentrace kortizolu, tak i jeho koncentrace ve slinách byly velmi nízké (1,2 μg/dL, referenční rozmezí pod 7,5 μg/dL; 0,04 μg/dL a referenční rozmezí pod 0,27 μg/dL). Hodnoty kortikosteroidy vázajícího proteinu byly v normě (41 mg/L, referenční rozmezí 35–50 mg/L). Katecholaminy v moči v normě, a test na primární aldosteronizmus negativní. Vyšetření kostní denzity potvrzuje osteoporózu. CT abdomenu ukazuje malé nadledviny a MR hypofýzy je bez abnormalit.

', qs: [ { task: '

Co víte o dexametozonovém supresním testu a ACTH (kosintropinovém) stimulačním testu?

', answer: '

Dexametazonový supresní test je zkouškou činnosti negativní zpětné vazby systému hypotalamo-hypofyzární-nadledvinové regulační osy. Podáním syntetického glukokortikoidu dexametazonu by mělo být dosaženo suprese uvolnění endogenních glukokortikoidů z nadledvin, což se projeví poklesem plazmatických koncentrací kortizolu. Test lze provádět ve dvou odlišných uspořádáních. Při testu s nízkou dávkou dexametazonu je aplikován 1 mg látky, v případě vysokodávkového testu potom 8 mg. Dexametazon je aplikován buď v jednotlivé dávce večer a hladiny kortizolu jsou stanovovány ve vzorku krve odebraném následující den ráno nebo se dexametazon podává opakovaně (3 dny) v nižších dávkách (0,5 mg nebo 2 mg) a hladiny kortizolu jsou stanovovány v moči sbírané po celé tři dny testu. Pokud nedojde k poklesu hladin kortizolu (pod 50% bazální hodnoty) po 2 mg dexametazonu jedná se pravděpodobně o Cushingův syndrom. Rozdílné dávky dexametazonu umožňují diferenciální diagnostiku mezi nadprodukcí ACTH a hyperfunkcí kůry nadledvin/ektopického tumoru (nedojde k supresi po vysoké dávce).

ACTH (konsytropinový) stimulační test stanovuje citlivost nadledviny na hypotalamo-hypofyzární regulační osu. ACTH nebo jeho syntetický analog (tetrakosaktid, kosyntropin, alsaktid) jsou aplikovány pacientovi. Test se většinou používá pro diferenciální diagnostiku mezi primární a sekundární insuficiencí nadledvin. Látka je aplikována intravenózně v dávce 1 μg/kg (nízká dávka) nebo 250 μg (konvenční dávka) a odběr krve je proveden cca 60 min. od aplikace. Nárůst plazmatické koncentrace o více než 50 % je považován za pozitivní výsledek. V případě Addisonovy choroby není nárůst koncentrace kortizolu vyšší než 25 %. Naopak pokud je nárůst kortizolu dramatický (až 14ti násobek původních hodnot), lze uvažovat o sekundární adrenální insuficienci. V případě dlouhotrvající sekundární adrenální insuficience je však třeba pamatovat na možnou atrofii kůry nadledvin, v důsledku čehož je výsledek testu také negativní.

' } ] }, { text: '

Po aplikaci 250 μg kosyntropinu nedošlo k výraznějšímu nárůstu plazmatických hladin kortizolu (4,8 μg/dL, referenční rozmezí >19,6 μg/dL). Dexametazonový supresní test (jedna dávka 1 mg ) způsobil dramatický pokles ranních plazmatických hladin kortizolu na hodnotu 0,05 μg/dL.

Tyroidní dysfunkce (normální hladiny TSH a tyroidních hormonů), syndrom polycystických ovárií (normální hladiny testosteronu, androstendionu, absence klinických příznaků a změn na USG) a kongenitální adrenální hyperplázie (hodnoty 17-hydroprogesteronu v normě) byly vyloučeny. Vzhledem k tomu, že zmíněné symptomy naznačovaly hypersenzitivitu na glukokortikoidy, byl proveden dexametazonový supresní test s velmi nízkou dávkou (0,25 mg). Výsledkem bylo výrazné snížení hladin kortizolu (0,1 μg/dL), což potvrdilo diagnózu glukokortikoidní hypersenzitivity.

', qs: [ { task: '

Která léčiva by mohla být použita pro snížení produkce/uvolnění glukokortikoidů?

', answer: '

Inhibitory syntézy kortizolu

metyrapon – účinný krátkodobý inhibitor syntézy kortizolu. Inhibuje 11β-hydroxylaci a také tvorbu aldosteronu. Stimulace produkce kortizolu díky útlumu negativní zpětné vazby vede ke zvýšení syntézy a zvýšenému uvolňování kortizolových prekurzorů včetně 11-desoxykortizolu.

ketokonazol – inhibuje steroidogenezi na úrovni cytochromu P450 v kůře nadledvin.

mitotan – cytotoxická látka blokující štěpení bočního řetězce cholesterolu a 11β-hydroxylázu. Nástup účinku je pomalý a přetrvává několik měsíců po ukončení terapie.

etomidát – celkové anestetikum inhibující syntézu kortizolu. Působí rychle, nevýhodou je nutnost intravenózní aplikace. Vhodné pro použití u život ohrožující hyperkotizolémie.

Látky snižující aktivitu kortizolu

mifepriston – antagonista progesteronových a glukokortikoidních receptorů. Blokem negativní zpětné vazby zvyšuje hladiny ACTH a zároveň i hladiny kortizolu (proto nemůže být kortizol použit jako marker).

Na hypofýzu zaměřená terapie – ACTH, dopamin, somatostatin a agonisté PPARγ mohou modulovat syntézu a uvolnění kortizolu.

Dopaminergní agonisté – bromokriptin a kabergolin – jsou používány pro terapii hyperprolaktinémie a akromegalie.

PPARγ agonisté – u rosiglitazonu nebyl prokázán klinický efekt v porovnání s preklinickými daty.

Analoga somatostatinu – oktreotid, lanreotid – nedostatečná účinnost, nová látka pasireotid by mohl být účinnější podle dosavadních výsledků.

' } ] }, { text: '

I přestože je lékem volby u glukokortikoidní hypersenzitivity mifepriston, bylo rozhodnuto snížit sekreci kortizolu podáváním ketokonazolu v denní dávce 400 mg. Vzhledem k tomu, že čtyřměsíční terapie ketokonazolem nevedla ke snížení hmotnosti pacientky, bylo rozhodnuto přidat do terapie karbergolin (1 mg týdně).

', qs: [ { task: '

Jaké jsou indikace a mechanismy účinku mifepristonu, ketokonazolu a karbergolinu?

', answer: '

mifepriston – je syntetický steroid s kompetitivně antagonistickou aktivitou na progesteronovém a glukokortikoidním receptoru. Používá se k farmakologicky indukované interrupci (v kombinaci s prostaglandinovými analogy), k indukci maturace děložního cervixu před chirurgickou interrupcí a pro indukci porodu v případu smrti plodu in utero.

ketokonazol – je antimykotikum blokující syntézu ergosterolu. Používá se pro topickou aplikaci. Může být použit pro terapii endogenního Cushingova syndromu.

kabergolin – je dopaminergní látka ergolinové struktury. Je silným a dlouhodobě účinným D2 agonistou. Snižuje plazmatické koncentrace prolaktinu 7-28 dní po jednorázové aplikaci. Je indikován pro použití u Parkinsonovy nemoci, k zástavě laktace a k terapii hyperprolaktinémie.

' }, { task: '

Jak označujeme použití přípravku, u diagnózy neuvedené v SPC?

', answer: '

Off label použití je použití léčivého přípravku v rozporu s indikacemi uvedenými v SPC, tj. schválenými regulační autoritou. Předepisující lékař je zodpovědný za možné nežádoucí účinky léčiva, stejně jako za jeho možnou neúčinnost. Zároveň může být problém s úhradou takto použitých léčiv z veřejného zdravotního pojištění.

' }, { task: '

Jak mohou zmíněná léčiva snížit hladiny glukokortikoidů?

', answer: '

mifepriston – zvyšuje hladiny kortizolu na základě suprese negativní zpětné vazby, nicméně blokuje jeho účinky antagonistickým působením na glukokortikoidní receptory.

ketokonazol – inhibuje steroidogenezi na úrovni CYP450, 17α-hydroxylázy, ale i 11-hydroxylázy. Ketokonazol má i antiandrogenní účinky. Inhibuje aktivitu C17-20 lyázy v adrenálních a Leydigových buňkách.

kabergolin – podobně jako další dopaminergní agonisté (bromokryptin) přes D2 receptory regulují uvolňování ACTH z hypofýzy a ACTH produkujících tumorů. Částečně může snižovat nárůst ACTH způsobený aplikací mifepristonu.

' } ] }, { text: '

Po přidání karbergolinu ke ketokonazolu došlo u pacientky ke klinickému zlepšení stavu. Krevní tlak, i když byl pořád mírně zvýšený, se dařilo kontrolovat pomocí bisoprololu a amlodipinu. Hladiny glykémie a glukózová tolerance se vrátily k normálu. Také tělesná hmotnost poklesla o 11 kg, nicméně klinické známky Cushingova syndromu u pacientky byly stále patrné. Pacientka umírá po 8 letech terapie karbergolinem a ketokonazolem na následky dopravní nehody.

', qs: [ ] } ], biblio: ["

Krysiak, R., Okopien, B. Glucocorticoid hypersensitivity syndrome – a case report. West Indian Med J. 2012; 61(8):844-846.

http://caribbean.scielo.org/scielo.php?pid=S004331442012000800015&script=sci_arttext&tlng=en

"], editors: '0', index: 46 }, { title: 'I14cz', cathegory: 1, keywords: ["warfarin", "hluboká žilní trombóza", "hyperlipidémie", "atorvastatin", "fluvastatin"], parts: [ { text: '

Pacient, 67 let, s hlubokou žilní trombózou a hyperlipidémií v anamnéze užíval warfarin (4 mg/den) a jeho INR bylo v posledních 5 měsících udržováno v rozmezí 2-3. Další medikace zahrnovala atorvastatin (20 mg/den), vitamin B1 (100 mg/den) a multivitaminový přípravek. Pacient v posledních 6 měsících se stabilně nastavenou terapií, beze změny v potravních doplňcích a se stabilním příjmem vit. K. Pacient neguje konzumaci alkoholu a přiznává užívání žvýkacího tabáku. Pacient je adherentní k nastavené terapii, spolehlivý a dostatečně edukovaný o rizicích terapie warfarinem.

', qs: [ { task: '

Jaký je mechanismus účinku warfarinu a atorvastatinu?

', answer: '

warfarin – je perorální antikoagulans. Mechanismem účinku je kompetitivní antagonismus vitaminu K, který je kofaktorem gama glutamylkarboxylázy. Správná funkce tohoto enzymu je nezbytná pro tvorbu některých koagulačních faktorů (II, IV, IX, X).

' }, { task: '

Jsou podávané dávky léčiv v souladu s doporučeným dávkováním dle SPC?

', answer: '

warfarin – udržovací dávka se pohybuje obvykle v rozmezí mezi 3 a 9 mg. Dávkování je přísně individuální a řídí se protrombinovým časem, respektive hodnotami dosaženého INR.

atorvastatin – úvodní dávka je obvykle 10 mg a maximální 80 mg.

' }, { task: '

Může tabák ovlivnit metabolismus warfarinu?

', answer: '

Warfarin je racemická směs složená z R a S izomerů s odlišnou biotransformací. Oba izomery jsou však metabolizovány především jaterními cytochromy P450 (CYP). R-enantiomer je metabolizován preferenčně přes CYP1A2 a 3A4, kdežto S-enantiomer je substrátem pro CYP2C9. Tabákový kouř je známým induktorem CYP1A2 a 2B6. Nicméně, vliv žvýkání tabáku na metabolickou aktivitu CYP dosud nebyl popsán. Je pravděpodobné, že se bude lišit od kouření, kdy jsou za vysokých teplot z tabáku uvolňovány látky ze skupiny polycyklických aromatických uhlovodíků, které indukují CYP1A2.

' }, { task: '

Víte, jak rychle nastupuje účinek warfarinu?

', answer: '

Nástup účinku trvá přibližně 36 až 72 hodin a po vysazení účinek přetrvává 4 až 5 dní.

' }, { task: '

Jaké jsou hlavní mechanismy lékových interakcí warfarinu?

', answer: '

Hlavními mechanismy lékových interakcí warfarinu jsou vytěsnění z vazby na plazmatické bílkoviny a interakce na úrovni CYP.

' } ] }, { text: '

Pacientovi došel přípravek s atorvastatinem a kvůli jeho nedostupnosti byl pacientovi na 4 týdny nasazen fluvastatin s prodlouženým uvolňováním v dávce 80 mg/den. Po pěti týdnech se INR pohybovalo mírně nad cílovou hodnotou. Dávka warfarinu nebyla změněna a pacient pokračoval dále v léčbě fluvastatinem. Po dalších čtyřech týdnech bylo INR signifikantně zvýšeno na hodnotu 6,6. Pacient popírá symptomy krvácení nebo podlitiny a potvrzuje pravidelné užívání nastavené terapie. K žádným změnám v medikaci nebo v příjmu vit. K nedošlo.

', qs: [ { task: '

Jaká je cílová hodnota INR u warfarinizovaných pacientů?

', answer: '

Cílová hodnota INR pro většinu případů je 2,5 s tolerancí v rozmezí od 2,0 do 3,0.

' }, { task: '

Jaký je rozdíl mezi atorvastatinem a fluvastatinem, který mohl vést ke zvýšení INR?

', answer: '

Fluvastatin je biotransformován v játrech řadou enzymů CYP a je tedy relativně necitlivý ke změnám v aktivitě jednotlivých enzymů CYP, protože při inhibici jednoho enzymu jej může zastoupit jiný. Fluvastatin byl popsán jako inhibitor CYP 2C9.

Atorvastatin je metabolizován především cytochromem 3A4 a není známo, že by ovlivňoval aktivitu jiných enzymů CYP. Obě látky mají vysokou afinitu k plazmatickým bílkovinám (vázáno až 98 %), nicméně jejich vazba nekompetuje s vazebnými místy pro warfarin, a proto jeho plazmatické hladiny tímto způsobem neovlivňují.

' }, { task: '

Jaký je terapeutický postup v případě intoxikace warfarinem?

', answer: '

Přerušení podávání warfarinu, případně redukce jeho dávky, aplikace vitaminu K, transfuze krve nebo krevních derivátů obsahujících funkční hemokoagulační faktory.

' } ] }, { text: '

Warfarin byl vysazen na dobu 4 dnů a potom byla terapie zahájena nižší dávkou. Následně byl fluvastatin zaměněn za atorvastatin a hodnota INR začala klesat. Dávka warfarinu byla změněna jeden týden po záměně statinů. Hodnota INR se vrátila na požadované hodnoty.

', qs: [ { task: '

Jaká další léčiva ze skupiny statinů by mohla zvýšit antikoagulační účinek warfarinu?

', answer: '

Lovastatin, simvastatin a atorvastatin jsou substráty pro CYP 3A4, kdežto fluvastatin je metabolizován a zároveň inhibuje enzym 2C9. Pravastatin není metabolizován systémem cytochromu P450 a není o něm známo, ani že by nějakým způsobem ovlivňoval jeho aktivitu.

' } ] } ], biblio: ["

Andrus, MR. Oral anticoagulant drug interactions with statins: Case report of fluvastatine and review of the literature. Pharmacotherapy. 2004;24(2):285-290.

http://onlinelibrary.wiley.com/doi/10.1592/phco.24.2.285.33137/pdf

"], editors: '0', index: 47 }, { title: 'I15cz', cathegory: 1, keywords: ["diabetes", "enalapril", "isosorbid mononitrát", "digoxin", "furosemid", "spironolakton", "ranitidin", "hydrotalcid", "omeprazol", "glimepirid", "gabapentin", "tramadol", "metylprednizolon", "tiotropium", "teofylin", "formoterol", "budesonid", "ipratropium", "salbutamol", "bromhexin", "ambroxol", "sulodexid", "fenofibrát"], parts: [ { text: '

Sedmašedesátiletý pacient byl odeslán praktickou diabetoložkou k zahájení inzulinoterapie pro sekundární selhání PAD. Sám pacient udával asi měsíc zhoršení kompenzace diabetu, glykémie okolo 15 mmol/l, negoval změnu diety, medikace či režimu. Před několika týdny přechodné otoky nohou, dušnost, v noci spal s několika polštáři pod hlavou. Jiné potíže negoval, černou stolici nepozoroval. V osobní anamnéze udával kromě diabetu II. typu, těžkou CHOPN, pro kterou byl v invalidním důchodu, sledován na plicní klinice. Léčil se s hypertenzí a chronickou ICHS, ale bolesti na hrudi v té době neměl. Před 30 lety byl operován pro krvácející žaludeční vřed. Negoval alergie, byl abstinent, asi rok již nekouřil, dříve býval silný kuřák.

', qs: [ { task: '

Jaká léčiva byste předpokládali, že bude pacient užívat (stačí farmakoterapeutické skupiny)?

', answer: '

Vzhledem k diagnostikovanému diabetu II. typu lze předpokládat, že pacient užívá perorální antidiabetika. Pro zmiňované otoky a současně hypertenzi a ischemickou chorobu srdeční by mohl užívat diuretika a betablokátory (kontraindikace CHOPN), popřípadě nitrátová vazodilatans. Pro CHOPN se jeví pravděpodobné užívání bronchodilatačních látek, případně kortikoidů nebo mukolytik.

' } ] }, { text: '

Současná farmakoterapie se skládá z následujících léčiv:

Enap 5 mg 1-0-1

Sorbimon 20 mg 1-1-0

Digoxin 0,25 mg 1-0-0

Furon 40 mg 1-0-0

Verospiron 25 mg 1-1-1

Ranisan 150 mg 0-0-1

Talcid - dle potřeby

Omeprazol 20 mg 1-0-0

Amaryl 2 mg tbl 2-0-2

Kalium chloratum 1-0-0

Cardilan 1-0-1

Neurontin 100 mg 1-1-2

Tramal 150 1-0-1

Medrol 16 mg 1-0-1

Spiriva inh. 1-0-0

Euphyllin 300 1-0-1

Foradil 12 mg 1-0-1

Miflonid 400 1-0-1

Atrovent N 6 × 4 vdechy

Ventolin inhaler N - dle potřeby

Bromhexin 8 mg 1-0-1

Mucosolvan sol 1-1-1

', qs: [ { task: '

Jaká léčiva obsahují užívané přípravky, jaký je jejich mechanismus účinku?

', answer: '

Enap – enalapril – proléčivo, po absorbci se hydrolyzuje na enalaprilát, který je inhibitorem angiotenzin konvertujícího enzymu (ACE). Inhibice ACE vede k poklesu produkce angiotenzinu II a díky inhibici zpětné vazby v systému RAAS ke zvýšení hladin reninu a následně ke snížení sekrece aldosteronu.

Sorbimon – isosorbid mononitrátnitrátové vazodilatans – má přímý relaxační účinek na hladké svaly cév. Dilatuje především vény a snižuje tak preload, což u pacientů s ICHS vede k poklesu plnících tlaků v obou komorách a dochází ke snížení spotřeby kyslíku v srdci. Dilatace v arteriální části vede ke snížení afterloadu a dilatace koronár zlepšuje perfuzi myokardu.

Digoxin – digoxin – přírodní alkaloid inhibující Na+/K+-ATPázu v buněčné membráně, což vede k intracelulární akumulaci sodíkových iontů. Mechanismem antiportu jsou následně transportovány extracelulárně a opačným směrem je dopravován vápník. Vzestup intracelulární koncentrace Ca2+ iontů se projeví zvýšením inotropie. Nepřímým vagomimetickým účinkem (senzitizace baroreceptorů, facilitace cholinergní neurotransmise v myokardu působí negativně chronotropně a díky snížení vodivosti v AV uzlu má digoxin i antiarytmický účinek.

Furon – furosemid – je kličkové diuretikum blokující reabsorpci Na+, K+ a Cl- ve vzestupném raménku Henleovy kličky.

Verospiron – spironolaktondraslík šetřící diuretikum, kompetitivní antagonista aldosteronu, respektive inhibitor jeho intracelulárních receptorů. Přímým účinkem na tubulární transport zvyšuje i vylučování Ca2+.

Ranisan – ranitidin – reverzibilní antagonista histaminových H2 receptorů.

Talcid – hydrotalcidnesystémové antacidum obsahující soli Al3+ a Mg2+ vázané v mřížkovité vrstevnaté struktuře, ze které se postupně uvolňují a snižují kyselost žaludečního obsahu na hodnoty pH v rozmezí 3-5.

Omeprazol – omeprazol – ireverzibilní inhibitor protonové pumpy (H+/K+-ATPáza).

Amaryl – glimepiridPAD ze skupiny derivátů sulfonylmočoviny. Blokuje K+ kanály pankreatických β buněk, což vede k depolarizaci membrány a sekreci inzulínu. Z periferních účinků glimepirid zvyšuje senzitivitu periferních tkání k inzulínu, zvyšuje počty glukózových transportérů ve svalových a tukových buňkách a snižuje vychytávání inzulínu játry.

Kalium chloratum – draslík – hlavní intracelulární kationt, nezbytný pro správnou funkci téměř všech buněk. U pacienta hrozí hypokalémie díky použití furosemidu.

Cardilan – draslík a hořčík – přípravek je určen pro zlepšení tolerance terapie digitálisovými glykosidy, deplece draselných a hořečnatých iontů je rizikovým faktorem pro vznik srdečních arytmií. Ke snížení jejich plazmatických koncentrací může dojít také vlivem užívaných diuretik.

Neurontin – gabapentinstrukturní analog GABA, váže se na α2-δ podjednotky napěťově řízených Ca2+ kanálů, snižuje uvolňování monoaminů a zvyšuje koncentrace GABA v některých částech mozku.

Tramal – tramadolneselektivní agonista opioidních receptorů, inhibitor zpětného vychytávání noradrenalinu, zvyšuje uvolňování serotoninu, účinnost ve srovnání s morfinem je cca 1/10 - 1/6. Dále se uvádí že tramadol blokuje NMDA, 5-HT2C, nikotinové (α7)2 a muskarinové M1 a M3 receptory. Agonisticky působí na TRPV1 receptorech.

Medrol – metylprednizolonglukokortikoid, po vazbě na intracelulární receptor ovlivňuje expresi specifických genů a některé intracelulární signální dráhy.

Spiriva inh. – tiotropiumantagonista muskarinových receptorů s bronchodilatačním a inhibičním efektem na bronchiální sekreci, po inhalačním podání má díky kvarternímu dusíku lokální efekt v širokém rozmezí dávek než dojde k systémovému anticholinergnímu účinku.

Euphyllin – teofylininhibitor fosfodiesterázy – zvyšuje intracelulární koncentrace cAMP

Foradil – formoterolselektivní β2 agonista s bronchodilatačním efektem a minimálním vlivem na KVS. Inhibuje taktéž uvolnění histaminu a leukotrienů se senzibilizovaných plic.

Miflonid – budesonid - glukokortikoid, po vazbě na intracelulární receptor ovlivňuje expresi specifických genů a některé intracelulární signální dráhy.

Atrovent N – ipratropium - podobně jako tiotropium se jedná o antagonistu muskarinových receptorů s kvarterním dusíkem v molekule.

Ventolin inhaler N – salbutamol - selektivní β2 agonista s bronchodilatačním efektem, ve srovnání s formoterolem má kratší biol. poločas (účinek trvá 4-6 h).

Bromhexin – bromhexin – proléčivo ambroxolu sekretolytikum a sekretomotorikum.

Mucosolvan - ambroxol – zvyšuje sekreci hlenu v dýchacím traktu, zvyšuje tvorbu plicního surfaktantu a stimuluje činnost řasinek.

' }, { task: '

Odpovídá dávkování léčiv doporučeným dávkám?

', answer: '

Enap 5 mg 1-0-1 – pro terapii hypertenze je doporučená dávka v rozmezí od 5 do 40 mg denně.

Sorbimon 20 mg 1-1-0 – dávka 2x20 mg je určena pro terapii lehčích forem ICHS.

Digoxin 0,25 mg 1-0-0 - dávkování je přísně individuální s udržovací dávkou v rozmezí od 0,125 mg do 0,25 mg.

Furon 40 mg 1-0-0 – obvykle se podává 40 - 120 mg denně.

Verospiron 25 mg 1-1-1 – u terapie otoků se dávka pohybuje v rozmezí 25-200 mg u hypertenze 50 - 100 mg.

Ranisan 150 mg 0-0-1 – u většiny indikací je denní dávka 300 mg, pouze u pacientů se sníženou funkcí ledvin na polovinu.

Talcid dle potřeby – maximální denní dávka je 6 g.

Omeprazol 20 mg 1-0-0 – doporučená dávka 20 mg jednou denně.

Amaryl 2 mg tbl 2-0-2 – maximální dávka je 6 mg denně. Nad dávku 4 mg se dostavuje účinek pouze ve výjimečných případech.

Kalium chloratum 1-0-0 – dávkování je individuální dle hodnot kalémie, obvyklá dávka je 1-2 tablety 2-3x denně.

Cardilan 1-0-1 – obvyklá udržovací dávka je ½ až 1 tableta 3x denně.

Neurontin 100 mg 1-1-2 - zde zřejmě v indikaci periferní neuropatické bolesti, pacient pravděpodobně se sníženou funkcí ledvin, čemuž odpovídá i relativně nízká dávka (obvykle 900 mg a více až do max. 3600 mg).

Tramal 150 1-0-1 – 150 - 200 mg dvakrát denně.

Medrol 16 mg 1-0-1 – dávkování individuální.

Spiriva inh. 1-0-0 – 1x denně.

Euphyllin 300 mg 1-0-1 – pro věkovou kategorii 60-70 let je doporučeno rozmezí denních dávek 660 - 910 mg.

Foradil 12 mg 1-0-1 – obvykle 1 - 2 inhalační tobolky denně.

Miflonid 400 mcg 1-0-1 – obvykle 200 - 1600 mcg/den.

Atrovent N 6 × 4 vdechy – maximální dávka by měla být 12 vdechů, obvykle se užívají 2 - 4 vdechy.

Ventolin inhaler N dle potřeby.

Bromhexin 8 mg 1-0-1 – 1-2 tablety 3x denně.

Mucosolvan sol 1-1-1 – není uveden objem jednotlivých dávek = nelze ověřit správnost dávkování.

' }, { task: '

Přiřaďte léčiva k následujícím indikacím:

Hypertenze:

ICHS:

CHOPN:

DM:

Diabetická neuropatie:

Gastrický vřed:

', answer: '

Hypertenze: enalapril, spironolakton, furosemid

ICHS: isosorbid mononitrát

CHOPN: ipratropium, tiotropium, salbutamol, formoterol, teofylin, budesonid, metylprednizon, bromhexin, ambroxol

DM: glimepirid

Diabetická neuropatie: tramadol, gabapentin

Gastrický vřed: ranitidin, hydrotalcid, omeprazol

' }, { task: '

Jaká léčiva se Vám již nyní zdají nadbytečná?

', answer: '

Z hlediska racionální farmakoterapie se jeví jako zbytečné podávat kombinaci ranitidinu s omeprazolem; suplementovat draslík ve dvou přípravcích a navíc ještě podávat spironolakton; aplikovat tiotropium a ipratropium současně, stejně jako podávat ambroxol a bromhexin, který je jeho proléčivem; sporné je také současné použití formoterolu a salbutamolu, dávka glukokortikoidů (budesonid a metylprednizon) by pravděpodobně mohla být také redukována.

' }, { task: '

Existuje mezi používanými léčivy riziko lékových interakcí, jaké?

', answer: '

enalapril

A) nevhodná kombinace s draslík šetřícími diuretiky a draslíkovou suplementací, sám má kalium šetřící efekt

B) při kombinaci s PAD nebo inzulínem může vést ke zvýšenému riziku hypoglykémie

C) kombinace s diuretiky zvyšuje riziko rozvoje hypotenze

D) nevhodná kombinace s glukokortikoidy, které snižují jeho účinnost díky retenci tekutin

spironolakton

A) současné podání se suplementací draslíku může vést k hyperkalémii

B) může snižovat clearance digoxinu

furosemid

A) může snižovat účinnost perorálních antidiabetik

B) může potencovat účinek teofylinu

C) společně s glukokortikoidy může zvyšovat depleci draslíku

salbutamol

A) užívání současně s diuretiky nešetřícími draslík může vést k hypokalémii

B) snižuje plazmatické hladiny digoxinu

formoterol

A) kombinace s teofylinem vede k vzájemné potenciaci účinku

B) kombinace s xanthiny, kortikoidy a kličkovými a thiazidovými diuretiky může potencovat hypokalemický efekt

teofylin

A) riziko zpomalení biotransformace při kombinaci s ranitidinem

budesonid

A) hypokalemizující efekt může potencovat účinky digoxinu

tramadol

A) zvyšuje toxicitu digoxinu

gabapentin

A) snížení biologické dostupnosti až o 24 % při současném podání s antacidy obsahujícími hořčík a hliník

omeprazol

A) zvyšuje biol. dostupnost digoxinu o 10 %

' } ] }, { text: '

V objektivním vyšetření byly patrné známky kongesce v krčních žilách, nad průduškami byly oboustranně slyšitelné ojedinělé bronchitické fenomény, játra byla hmatná + 6 cm pod oblouk, na dolních končetinách oboustranně mírné otoky po bérce. Krevní tlak při přijetí byl 130/90 mmHg, pulz pravidelný 90/min.

Vybrané hodnoty biochemie séra:

Draslík: 5,15 mmol/l [3,6-5,4]

Hořčík: 1,16 mmol/l [0,80-1,05]

Urea: 9,3 mmol/l [2,8-8,3]

Kreatinin: 99 μmol/l [53,0-124,0]

Aminotransf. AST: 0,5 μkat/l [0,00-0,94]

Alkal. fosfatáza: 2,47 μkat/l [0,00-2,70]

TAG: 3,43 mmol/l [0–1,7]

Cholesterol: 7,89 mmol/l [0,00-5,00]

HDLc 1,13 mmol/l [1–1,6]

LDLc 5,17 mmol/l [0–3,0]

Glukóza: 10,29 mmol/l [3,00-5,60]

Glykovaný hemoglobin 9,2 % [2,8–6,0]

Hladiny léčiv: digoxin 0,56 mg/l [0,9–2,2], teofylin 5,57 mg/l [8–20]

', qs: [ { task: '

Co můžete odvodit z uvedených parametrů?

', answer: '

a) Hladiny draslíku a hořčíku jsou na horní hranici fyziol. rozmezí, proto by bylo vhodné suplementaci těchto látek zrušit nebo omezit (Kalium chloratum a Cardilan).

b) Zvýšené hladiny TAG, celkového a LDL cholesterolu jsou známkou dosud nediagnostikované smíšené hyperlipidemie.

c) Hodnoty glykémie a glykovaného hemoglobinu svědčí pro nekompenzovaný diabetes.

d) Plazmatická hladina teofylinu je pod minimální hodnotou terapeutického rozmezí a jeho dávka by tudíž měla být zvýšena.

' }, { task: '

Jaký je vztah plazmatických hladin draslíku k účinnosti digoxinu?

', answer: '

Při hypokalémii se zvyšuje frekvence výskytu a závažnost intoxikací. Vysoké hladiny kalémie vedou ke zpomalení vedení vzruchu síněmi a subnodálním systémem a snižují srdeční automaticitu.

' } ] }, { text: '

Samotné převedení pacienta na inzulín bylo nekomplikované. Dávka kortikoidů byla snížena na polovinu (indikace perorálních glukokortikoidů je u CHOPN sporná, bylo však respektováno rozhodnutí pneumologů). Při snaze redukovat polypragmázii jsme vycházeli z přání samotného pacienta. Při rozboru jeho medikace byla na první pohled patrná některá nedorozumění, kdy si ne zcela jasně vyložil doporučení při změnách medikace (např. Mucosolvan a Bromhexin, Spiriva a Atrovent). Vzhledem ke spíše vyšší hladině kalia a magnezia byla redukována substituce (Kalium chloratum a Cardilan). Během hospitalizace došlo při 40 mg furosemidu denně k ústupu otoků a známek městnání.

', qs: [ { task: '

Jaký může být důvod, že furosemid během hospitalizace snižuje otoky oproti ambulantní terapii u tohoto pacienta?

', answer: '

Pacient léčivo pravděpodobně neužívá pravidelně.

' } ] }, { text: '

Na cílený dotaz pacient přiznával, že měl problémy Furon užívat pravidelně, indikace mu navíc nebyla zcela jasná. Vzhledem k sinusovému rytmu a nedohledatelnému údaji o případné arytmii jsme se rozhodli ukončit léčbu digoxinem.

Pro anamnézu komplikovaného vředu a dlouhodobé užívání kortikoidů byla provedena kontrolní gastroskopie, která popsala ulceraci Forrest III.

', qs: [ { task: '

Jak byste změnili antiulcerózní terapii?

', answer: '

Ukončit terapii ranitidinem. Má oproti omeprazolu vyšší riziko lékových interakcí a nižší účinnost.

' }, { task: '

Jaké léčivo byste u tohoto pacienta použili pro antiagregační terapii?

', answer: '

Kyselina acetylsalicylová není vhodná jednak z důvodu rizika rozvoje gastroduodenálních vředů a jednak z důvodu možné indukce bronchokonstrikce. Přednost bychom dali jiné antiagregační látce např. dipyridamolu.

' }, { task: '

Pacient má minimální sklony ke kardiálním potížím, lze upravit antiangiózní terapii pouze pro použití v případě potřeby, jak?

', answer: '

Lze, nepodávat dlouhodobě působící nitráty, ale nasadit lékovou formu pro orální použití pouze u akutních ischemických záchvatů (např. sublingvální tablety nebo sprej).

' }, { task: '

Jaké léčivo byste použili pro nově diagnostikovanou hyperlipidémii?

', answer: '

Vzhledem ke smíšenému typu hyperlipidémie bychom volili pravděpodobně léčivo ze skupiny fibrátů.

' } ] }, { text: '

Gastroprotektivní léčba ranitidinem byla zaměněna za omeprazol. Vhledem k minimálním kardiálním potížím a sklonu k hypotenzi byla nakonec dočasně redukována i kardiální medikace, profylakticky podávané nitráty byly nahrazeny sprejem pro akutní použití. Vzhledem k nově zachycené dyslipidemii bylo do medikace naopak přidáno hypolipidemikum, pro kontraindikaci kys. acetylsalicylové či plné antikoagulace též sulodexid. Vzhledem k subterapeutické hladině teofylinu byla zvýšena jeho dávka s plánovanou kontrolou při další návštěvě na plicní klinice (s ohledem na kardiální diagnózy).

Pacient v dobrém stavu odcházel s polovinou původní medikace (metylprednizolon, omeprazol, sulodexid, fenofibrát, furosemid, KCl, formoterol, ambroxol, salmeterol, tiotropium, budesonid, teofylin a inzulín).

', qs: [ { task: '

Jaké jsou mechanismy účinku nově přidaných léčiv sulodexidu a fenofibrátu?

', answer: '

sulodexid - kombinovaná molekula složená ze středněmolekulárního heparinu (80 %) a dermatanové složky (20 %). Heparinová složka je zodpovědná za antikoagulační účinek spočívající zejména v inhibici aktivovaného faktoru X přes antitrombin III. Heparinová složka má pravděpodobně i fibrinolytické účinky, protože stimuluje uvolňování t-PA a inhibuje aktivitu PAI. Dermatanová složka pravděpodobně působí antiagregačně díky zvýšení uvolňování PGI2 a heparin-kofaktoru II. Dále má sulodexid lipolytické účinky díky aktivaci lipoproteinové lipázy. Společně s hypofibrinogenemickým účinkem (pouze u hyperfibrinogémie) jsou tyto dva mechanismy pravděpodobně zodpovědné za snížení viskozity krve. V neposlední řadě byl prokázán pozitivní vliv sulodexidu na stav cévního endotelu a sníženou adhezitivu krevních elementů k endotelu. Účinek sulodexidu na cévní systém tak lze hodnotit jako komplexní.

fenofibrát – aktivuje receptory PPARα. Tím dochází ke zvýšení lipolýzy a eliminaci lipoproteinů bohatých na triglyceridy. Zejména jde o pokles VLDL a LDL a naopak nárůst frakce HDL obsahující apoproteiny AI a AII, jejichž syntéza se vlivem fibrátů zvyšuje.

' } ] } ], biblio: ["

Strojil, J, Horák, F., Ürge, J. Polypragmázie u 67letého diabetika. Klinická farmakologie a farmacie. 2007;21(1):43-45.

http://www.klinickafarmakologie.cz/pdfs/far/2007/01/09.pdf

"], editors: '0', index: 48 }, { title: 'I16', cathegory: 1, keywords: ["benigní hyperplazie prostaty", "hyperurikemie", "infarkt myokardu", "metoprolol", "kyselina acetylsalicylová", "klopidogrel", "fentanyl", "atorvastatin", "perindopril", "tamsulosin", "molsidomin"], parts: [ { text: '

Pacientem je šedesátiletý muž s diagnózou benigní hypertrofie prostaty a hyperurikemie. Dostaví se k praktickému lékaři s náhle vzniklými bolestmi na hrudi vystřelujícími do krku. Vyšetřující lékař podal pacientovi Nitroglycerin 1 tabletu a odeslal ho sanitkou na vyšetření do spádové nemocnice. Na interní ambulanci vyšetřující lékař zaznamenal přetrvávající bolesti po nitrátu beze změny. EKG vyšetření odhalilo ST deprese v laterálních svodech, TF 80, TK 140/95. Laboratorní markery CK, MB a troponin T byly elevovány s nárůstem troponinu T v dalších 6 hodinách. Diagnóza byla uzavřena jako non-STEMI akutní infarkt myokardu.

Pacient byl akutně zaléčen:

Betaloc Zok 100 mg 1 tableta

Kardegic iv 1 ampule

Trombex 4 tablety

Fentanyl 4 ml 50 µg/ml i.v.

', qs: [ { task: '

Vyhledejte, o jaké účinné látky se jedná, zařaďte je do terapeutických skupin, a spočítejte celkové podané dávky v mg.

', answer: '

Betaloc Zok – metoprolol, betablokátor, antihypertenzivum, celková dávka 100 mg.

Kardegic – kyselina acetylsalicylová, antiagregans, celková dávka 500 mg.

Trombex – klopidogrel – antiagregans, celková dávka 4 x 75 mg = 300 mg.

Fentanyl – fentanyl - opioidní analgetikum, celková dávka 0,2 mg.

' } ] }, { text: '

Pacient byl odeslán na koronarografii, kde byla zjištěna stenóza koronární tepny a EF LK 60 %. Následně byla provedena implantace stentu. Po nekomplikovaném průběhu byl po týdnu na standardním oddělení propuštěn s následující medikací:

Anopyrin 100 mg 0-1-0

Plavix 75 mg 1-0-0 na tři měsíce

Betaloc Zok 50 mg 1-0-0

Atoris 20 mg 0-0-1

Prestarium Neo 1-0-0

', qs: [ { task: '

Vyhledejte, o jaké účinné látky se jedná, zařaďte je do terapeutických skupin a vysvětlete jejich mechanismus účinku.

', answer: '

Anopyrin – acetylsalicylová kyselinaantiagregans – kyselina acetylsalicylová je inhibitorem trombocytární aktivace. Ireverzibilní blokádou destičkové cyklooxygenázy zabraňuje syntéze tromboxanu A2, fyziologického aktivátoru, který je vylučován trombocyty.

Plavix – klopidogrelantiagregans – aktivní metabolit klopidogrelu, který vzniká po jeho metabolizaci CYP, selektivně inhibuje vazbu adenosindifosfátu (ADP) na jeho destičkový receptor P2Y12 a následně ADP-zprostředkovanou aktivaci glykoproteinového komplexu GPIIb/IIIa, čímž je inhibována agregace destiček.

Betaloc – metoprololantihypertenzivum – beta1-selektivní blokátor adrenergních receptorů bez vnitřní sympatomimetické aktivity.

Atoris – atorvastatinhypolipidemikum – léčivo ze skupiny statinů. Inhibicí klíčového enzymu HMG-CoA reduktázy blokují endogenní syntézu cholesterolu.

Prestarium – perindoprilantihypertenzivum – ACE inhibitor.

' }, { task: '

Jaké jsou kontraindikace pro podání kyseliny acetylsalicylové?

', answer: '

Kontraindikace pro podání kyseliny acetylsalicylové (ASA) jsou například: hypersenzitivita na ASA, akutní žaludeční nebo duodenální vředy, hemoragická diatéza, pacienti se stávající mastocytózou, u kterých užívání ASA může vést k závažným hypersenzitivním reakcím, předchozí výskyt astmatu, který byl indukován požitím salicylátů či látek s obdobným účinkem (NSAID), závažná jaterní nebo renální insuficience, kombinace s methotrexátem v dávkách > 15 mg/týden, dávkování > 100 mg/ den během třetího trimestru těhotenství.

' } ] }, { text: '

Po měsíci pacient přichází ke svému praktickému lékaři s perzistujícím suchým kašlem. Po vyloučení infekčního onemocnění nebo jiných organických příčin je jako příčina obtíží určen jeden z nasazených léků.

', qs: [ { task: '

Který lék vyvolal u pacienta kašel? Navrhněte úpravu terapie.

', answer: '

Jedná se o typický nežádoucí účinek inhibitorů ACE. Pokud se tento nežádoucí účinek vyskytne, je doporučeno zaměnit ACE-i za blokátor AT2 receptorů.

' } ] }, { text: '

Za rok přichází pacient s námahovou dušností a bolestmi na hrudi s akcelerací v posledním měsíci. Při vyšetření je pacientovi prováděna i kontrola lipidového spektra s následujícími výsledky: celkový cholesterol 4,5 mmol/l, HDL 0,7 mmol/l, LDL 2,8 mmol/l a TAG 4,1 mmol/l.

', qs: [ { task: '

Zhodnoťte typ hyperlipoproteinemie a navrhněte úpravu terapie. Vyjádřete se k možným rizikům interakcí mezi léčivy.

', answer: '

Jedná se o hypertriacylglycerolemii (optimální hladina TAG v séru u dospělého je do 1,7 mmol/l). Ke snížení TAG je možno přidat do terapie fibrát. Při téhle kombinaci je potřeba zvýšené pozornosti kvůli možné interakci se statinem, při které může dojít k poškození kosterního svalstva (rhabdomyolýza, sledovat myoglobin).

' }, { task: '

Na koronarografii byla zjištěna stenóza terminální větévky koronární tepny bez zhoršení EF neřešitelná invazivně. Navrhněte úpravu medikace pro zmírnění bolestí při námaze.

', answer: '

Nitrát v retardované formě. Pokud nestačí, tak je možné přidat molsidomin nebo kalciový blokátor.

' }, { task: '

Jakou terapii byste nasadili pacientovi v případě, že by se mu zhoršila benigní hypertrofie prostaty? Na jakou interakci musíme myslet?

', answer: '

Alfa-blokátor. Je však potřeba myslet na možnou interakci s betablokátorem, při které hrozí pacientovi synkopy. Nejvhodnějším by byl uroselektivní alfa-blokátor tamsulosin.

' } ] } ], biblio: ["

"], editors: '07', index: 53 }, { title: 'I17cz', cathegory: 1, keywords: ["antifosfolipidový syndrom", "warfarin", "guarana", "fenobarbital", "kofein"], parts: [ { text: '

Žena (27 let) prodělala 4 spontánní potraty a cévní mozkovou příhodu, po které jí byla potvrzena diagnóza antifosfolipidového syndromu. Léčbou je podání antikoagulancií – žena začala užívat warfarin.

', qs: [ { task: '

Jaký je mechanismus účinku warfarinu?

', answer: '

warfarin – perorální antikoagulans, brání syntéze koagulačních faktorů závislé na vitaminu K. Účinek warfarinu je založen na jeho schopnosti zamezit redukci a působení vitaminu K při syntézách koagulačních faktorů II, VII, IX a X. V terapeutických dávkách brání v syntéze koagulačních faktorů o 30 až 50 % a současně redukuje i jejich biologickou aktivitu. Warfarin dosahuje plného účinku po 2 až 7 dnech, během nichž dochází k eliminaci koagulačních faktorů již přítomných v cirkulaci.

' }, { task: '

Jak by měla být nastavena léčba, pokud by žena plánovala otěhotnění?

', answer: '

U žen s diagnostikovaným antifosfolipidovým syndromem se volí buď nízké dávky acetylsalicylové kyseliny p.o. prekoncepčně nebo i v průběhu těhotenství. Časté je v těhotenství také podání nízkomolekulárních heparinů (ev. nefrakcionovaného heparinu) s.c. U žen s nízkým titrem protilátek a bez předchozích potratů se volí pouze sledování bez medikace. Warfarin není možné v graviditě použít, má teratogenní účinky.

' } ] }, { text: '

Dávka byla postupně titrována až na 20 mg warfarinu denně, a přesto hodnota INR nedosahovala uspokojivých hodnot (opakovaně < 1,3). Podrobným rozhovorem se přišlo na to, že žena denně vypije více než 2 litry nealkoholického nápoje s obsahem extraktu z guarany. Bylo jí doporučeno se nápoji vyhnout. Díky tomu bylo možné titrovat dávku warfarinu směrem dolů až na 7,5 mg denně. Po opakovaném pokusném zařazení nápoje do pitného režimu se ženě INR opět snížilo na 1,2. Nápoj byl proto definitivně vyřazen ze stravy.

', qs: [ { task: '

Jaká látka obsažená v extraktu z guarany stav způsobila?

', answer: '

Pravděpodobně šlo o kofein.

' } ] }, { text: '

Muž (18 let) prodělal tři hluboké žilní trombózy a byl mu diagnostikován antifosfolipidový syndrom. Jeho předchozí anamnéza zahrnovala epilepsii léčenou dlouhodobě stabilní dávkou fenobarbitalu. Muž začal užívat warfarin.

', qs: [ { task: '

Jaký je mechanismus účinku fenobarbitalu? Má toto léčivo vliv na účinek jiných léčiv?

', answer: '

Fenobarbital je barbiturát. Tato léčiva se vážou na GABAA receptor spřažený s chloridovým kanálem (do jiného místa než benzodiazepiny) a způsobují útlum CNS. Fenobarbital je induktor CYP, může tedy snižovat plazmatické kontrakce léčiv.

' } ] }, { text: '

Dávka byla postupně titrována až na 20 mg warfarinu denně, a přesto hodnota INR nedosahovala uspokojivých hodnot (opakovaně < 1,5). Pohovorem bylo zjištěno, že pacient pije denně 1,5 l Coca-Coly. Lékař doporučil omezit konzumaci nápoje na 350 ml denně a INR se podařilo stabilizovat v požadované hodnotě při dávce 7,5 mg warfarinu denně. Za určitý čas se na pravidelné prohlídce ukázalo, že muž má opět INR 1,1. Nadměrné pití Coca-Coly popřel, připustil ale zvýšenou konzumaci kávy – více než 5 espress denně. Pacientovi bylo doporučeno pít pouze 1 espresso denně a INR se podařilo stabilizovat.

', qs: [ { task: '

Proč měl pacient takovou potřebu pít Coca-colu a kávu?

', answer: '

Pravděpodobně to bylo pro sedativní účinek fenobarbitalu. Sedace, spavost, únava a denní ospalost jsou popisovány jako velmi časté nežádoucí účinky tohoto léčiva.

' }, { task: '

Jakým mechanismem došlo v obou případech ke snížení účinnosti warfarinu?

', answer: '

Nejspíše došlo k interakci na úrovni metabolismu warfarinu. Warfarin je podáván jako směs S- a R-enantiomerů. Účinnější je S-warfarin, který je metabolizován CYP2C9. R-warfarin je metabolizován CYP3A4 a 1A2. Ovlivnění CYP2C9 tak bude výrazněji ovlivňovat účinnost warfarinu. Kofein je běžně jako planární molekula metabolizován CYP1A2, ale ve vysokých koncentracích se zapojuje také CYP2C9. Kofein indukuje metabolickou aktivitu obou výše uvedených CYP a dochází ke snížení plazmatické koncentrace warfarinu.

' } ] } ], biblio: ["

Clapauch, Silvia Hoirisch; Benchimol-Barbosa, Paulo R. Warfarin resistance and caffeine containing beverages. International journal of cardiology. 2012;156(1):e4-e5.

https://www.researchgate.net/profile/Paulo_Benchimol-Barbosa/publication/51581135_Warfarin_resistance_and_caffeine_containing_beverages/links/00b495284f7b33f1fd000000.pdf

"], editors: '06', index: 55 }, { title: 'I18cz', cathegory: 1, keywords: ["mikrocytární anemie", "diklofenak", "prednison", "kyselina alendronová", "perindopril", "amlodipin", "kolonopatie"], parts: [ { text: '

Žena, 69 let, séronegativní revmatoidní artritida, H. pylori negativní gastritis, colitis ulcerosa, hypertenze, osteoporóza, deprese. Přijata pro bolest břicha, krev ve stolici, průjem, intermitentní nauzeu a zvracení. Celkové tělesné vyšetření: afebrilní, orientovaná, spolupracuje, hypertenze kompenzovaná. Laboratorní vyšetření prokázalo mikrocytární anemii (MCV = 75), deficit Fe (Hb = 98 g/L).

FA: Fosamax 70 mg 1x týdně, Prednison 5 mg 1x denně, Diklofenak Duo 75 mg 2x denně, Prestance 5/5 1x denně, Cipralex 10 mg 1x denně.

Pro krev ve stolici byla pacientka indikována ke kolonoskopii. Kolonoskopie prokázala několik zánětlivých lézí v oblasti příčného tračníku. Biopsie z oblasti vzestupného tračníku ukazují na přítomnost ulcerativních lézí a fibrovaskulární granulační tkáně. Žádné známky dysplazie.

', qs: [ { task: '

Zjistěte, co obsahují uvedené léčivé přípravky za účinné látky, zařaďte je do farmakoterapeutické skupiny a popište jejich hlavní mechanismus účinku?

', answer: '

Fosamax = kyselina alendronová – terapie osteoporózy: inhibuje osteoklastickou resorpci kosti.

Prednison 5 mg = prednison – glukokortikoid: indikace antiflogistická a imunosupresivní.

Diklofenak Duo = diklofenak – nesteroidní antiflogistikum: neselektivní inhibice cyklooxygenázy.

Prestance 5/5= perindopril + amlodipin – antihypertenzivum: perindopril je ACE inhibitor a amlodipin je dihydropyridinový blokátor Ca2+ kanálů.

' }, { task: '

Pokuste se s využitím dostupných farmakologických databází a vlastních znalostí zjistit příčinu stavu a navrhněte řešení.

', answer: '

Příčina je pravděpodobně kolonopatie po NSAID a glukokortikoidech. Řešením by bylo přerušení léčby diklofenakem a glukokortikoidem, zvážení jiného druhu léčby či nasazení protektivních léčiv (antiulceróza).

' } ] }, { text: '

Pacientka přestává užívat diklofenak i prednison, po 7 měsících od vysazení diklofenaku i prednisonu je pacientka bez obtíží, hemoglobin 120 g/l.

', qs: [ { task: '

Specifikujte rizikové skupiny pacientů z pohledu terapie NSAID a gastropatie/kolonopatie.

', answer: '

Pacienti, kteří berou dlouhodobě glukokortikoidy, kuřáci, alkoholici, pozitivní nález H. pylori, dlouhodobě užívaná mukolytika. Riziko též roste u starší populace (60+), s délkou užívání a dávkou NSAID.

' }, { task: '

Navrhněte alternativní analgetickou terapii u revmatoidní artritidy

', answer: '

Alternativně lze krátkodobě použít analgetika: (intermitentně) nimesulid, meloxikam, paracetamol + kodein.

Adjuvantně některé léčivo ze skupiny DMARDS, antirevmatika třetí řady, biologická léčba. Konkrétní výběr je na revmatologovi.

' }, { task: '

Vyberte konkrétní léčivou látku (generický název, lékovou formu, firemní název, dávkování). Předepište LP a poučte pacienta (způsob užívání, možné NÚ).

', answer: '

Například meloxikam v léčivém přípravku Recoxa:

RECOXA

POR TBL NOB 30X15MG

Exp. Orig. No. I (unam)

D.S. 1 tabletu denně při bolesti

Pacient může začít užívat Recoxu až poté, co budou vyléčeny gastrické ulcerace. Lék se užívá 1x denně s jídlem nebo po jídle vždy ve stejnou denní dobu. Nesmí být překračována maximální denní dávka. Léčivý přípravek není vhodný pro akutní bolest, protože nástup účinku nastupuje za 1-2 hodiny, z toho důvodu se většinou užívá pravidelně pod dohledem lékaře.

' } ] } ], biblio: ["

Aftab, Abdur R., et al. NSAID-induced colopathy. A case series. J Gastrointestin Liver Dis. 2010;19(1): 89-91.

http://www.jgld.ro/2010/1/15.pdf

", "

El Hajj . et al. NSAID-induced Colopathy. CAS clinique/case report. Lebanese Medical Journal. 2009;57(4): 274-276.

http://lebanesemedicaljournal.org/articles/57-4/case3.pdf

"], editors: '07', index: 64 }, { title: 'I19cz', cathegory: 1, keywords: ["hypovolemický šok", "akutní renální selhání", "fibrilace síní", "digoxin", "amiodaron", "hypertyreóza", "pneumonitida", "thiamazol", "metoprolol"], parts: [ { text: '

56letý pacient byl hospitalizován pro rychle se rozvíjející poruchu řeči a chůze. Pacientova osobní anamnéza nezahrnovala žádnou chronickou medikaci, ale jednalo se o kuřáka s pravidelnou konzumací alkoholu. Při přijetí byl pacient ve středně závažném stavu a neurologické vyšetření prokázalo senzomotorickou afázii, hlubokou pravostrannou parézu a také pozitivní Babinského reflex na pravé straně. Počítačová tomografie mozku ukázala příznaky hemoragické mozkové příhody v levé hemisféře. Rentgen hrudníku neodhalil žádný nový nástup konsolidace a plicní cirkulace byla v normě.

Po iniciální stabilizaci se u pacienta objevily symptomy hypovolemického šoku a akutního renálního selhání (zvýšené hladiny urey, kreatininu, draslík 6,21 mmol/l a sodík 155,3 mmol/l) pramenící pravděpodobně z dehydratace pacienta. U pacienta byla zahájena i.v. rehydratace, dále byl podán dopamin v infúzi a forsírovaná diuréza. Důsledný monitoring na JIP objevil u pacienta fibrilaci síní.

', qs: [ { task: '

Jaké jsou normální hodnoty draslíku a sodíku v krvi a jak se provádí forsírovaná diuréza?

', answer: '

Běžně se hladiny draslíku pohybují mezi 3,6 – 5,0 mmol/l a hladiny sodíku mezi 137 – 148 mmol/l. Forsírovaná diuréza neboli zesílená diuréza je metoda pro urychlení eliminace látek exkretovaných do moči. V kombinaci s dostatečným zavodněním pacienta pomocí infuze se podávají nejčastěji diuretika (manitol, furosemid).

' }, { task: '

Které léčivo byste doporučili pacientovi podat k terapii fibrilaci síní?

', answer: '

K terapii fibrilaci síni lze použít digoxin, který má pozitivně inotropní a negativně chronotropní a dromotropní účinek. Jeho nevýhodou je pomalý nástup účinku. Dále lze použít b-blokátory (metoprolol a atenolol) a kalciové blokátory (verapamil a diltiazem) s negativně inotropním, chronotropním a dromotropním účinkem. Z antiarytmik III. skupiny lze použít amiodaron.

' } ] }, { text: '

Pacientovi byl podán i.v. Digoxin v dávce 0,5 mg, ale bohužel byl neúčinný, a proto byl i s ohledem na stav pacienta podán Sedacoron (iniciální bolus, následně v infuzi). Následující den došlo po celkové dávce Sedacoronu 1200 mg k poklesu srdečního tepu a úpravě sinusového rytmu.

', qs: [ { task: '

Jaké účinné látky jsou obsaženy v použitých léčivých přípravcích a jaké jsou jejich mechanismy účinku?

', answer: '

Digoxin obsahuje kardiotonikum digoxin. Digoxin blokuje Na+/K+ ATPázu v kardiocytech, což vede ke zvýšení intracelulární koncentrace sodíku, který je měněn za vápník z extracelulárního prostředí, což dále vede k nárůstu intracelulárních koncentrací vápníku a k pozitivně inotropnímu efektu.

Sedacoron obsahuje antiarytmikum amiodaron, který inhibicí draslíkového proudu prodlužuje repolarizaci ve fázi 3 a tím i akční potenciál a refrakterní periodu.

' } ] }, { text: '

Po dvou měsících se u pacienta objevila tachykardie neznámého původu a byly stanoveny hladiny tyreoidálních hormonů: TSH 0,001 mIU/ml (norm. 0,350 – 4,940), fT3 7,06 pmol/l (norm. 2,63 – 5,70) a fT4 37,57 pmol/l (norm. 9,0 – 19,0). Proto byla u pacienta zahájena léčba Thyrozolem 10 mg/den. Z důvodu nedostatečné odpovědi došlo ke zvýšení dávky na 30 mg/den. Po dalším měsíci byl pro perzistentní kašel a zvýšenou teplotu proveden rentgen plic s nálezem plicních lézí. Dalšími vyšetřeními byla u pacienta potvrzena intersticiální pneumonitida.

', qs: [ { task: '

Proč byly u pacienta kontrolovány hladiny hormonů štítné žlázy a proč se pravděpodobně u pacienta objevila pneumonitida?

', answer: '

Hladiny tyreoidálních hormonů byly kontrolovány, protože při terapii amiodaronem je potřeba myslet na možnou poruchu štítné žlázy. Jeho molekula obsahuje jód a při jeho běžně používaných dávkách pacient denně přijme až o několik řádů vyšší množství jódu, než je doporučováno. Toto může vést k nadprodukci tyroidálních hormonů (amiodaronem indukovaná thyreotoxikóza - AIT I. typu). Molekula amiodaronu se navíc strukturálně thyrodiálním hormonům podobá a může vyvolat zkříženou autoimunitní reakci s destrukcí folikulů štítné žlázy (AIT II. typu). V některých případech amiodaron může vyvolat naopak hypothyreózu. Platí, že v oblastech s vyšším příjmem jódu je vyšší výskyt hypotyreózy a v oblastech s jodovým deficitem je vyšší výskyt tyreotoxikózy.

Mezi častý nežádoucí účinek amiodaronu patří také plicní toxicita (intersticiální pneumotitida, pleuritida apod.).

' }, { task: '

Jaký je terapeutický postup u zjištěné hypertyreózy?

', answer: '

Základem terapie je léčba tyreostatiky. Za léčivo první volby dnes považujeme propylthiouracyl. S odstupem tří hodin od začátku podávání thionamidu pak přidáváme do terapie jodidy. Další terapie zahrnuje podávání betablokátorů ke zmírnění symptomů hypertyreózy (tachyarytmie, pocení, třes, neklid). Kortikoidy omezují konverzi T4 na T3 a jsou základem léčby autoimunitních onemocnění štítné žlázy, vyšší dávky kortikoidů se podávají u exoftlamu. Velmi důležitá je rehydratace a při malabsorpci také doplnění iontů a vitamínů. Další terapie je symptomatická, zahrnuje diuretika (prevence srdečního selhání), benzodiazepiny (zklidnění pacienta) a chlazení. Zajímavou možností je podávání lithia u jodem indukované tyreotoxikózy (nutnost TDM).

' } ] }, { text: '

I po 2měsíční léčbě Thyrozolem (dávka zvýšena na 60 mg/den) a 2týdenním podávání Betaloc Zoku (120 mg/den) přetrvávaly klinické i laboratorní příznaky hypertyreózy (TSH 0,0 mIU/ml, fT3 5,57 pmol/l, fT4 24,96 pmol/l). Ultrasonografie potvrdila zvětšenou štítnou žlázu a sérové vyšetření nezjistilo přítomnost antithyreiodálních protilátek (proti thyroperoxidáze, thyroglobulinu a TSH receptoru).

', qs: [ { task: '

Jak si vysvětluje dobu, po níž tyto symptomy/nežádoucí účinky přetrvávaly?

', answer: '

Biologický poločas amiodaronu je velmi dlouhý a individuálně variabilní: 20 – 100 dnů. Proto se u pacienta mohly příznaky hypertyreózy a intersticiální pneumonitidy objevit i po několika týdnech po ukončení terapie, i když byl amiodaron podáván jen několik dní.

' }, { task: '

Jaké účinné látky jsou obsaženy v použitých léčivých přípravcích a jaké jsou jejich mechanismy účinku?

', answer: '

Thyrozol obsahuje účinnou látku thiamazol, který inhibuje v závislosti na dávce ukládání jódu do tyrosinu a tím inhibuje syntézu hormonů štítné žlázy.

Betaloc Zoku obsahuje b-blokátor metoprolol.

' } ] }, { text: '

Pacientovi byl nasazen Prednison (20 mg/den), dávka Thyrozolu byla snížena na 30 mg/den a také dávka Betaloc Zoku byla postupně redukována. Tato terapie nakonec vedla k úpravě hladin tyreoidálních hormonů (fT3 4,22 pmol/l, fT4 16,30 pmol/l), k postupnému nárůstu TSH (0,002 μIU/ml) a úpravě nálezu na plicích.

Pacient byl po 2měsíční léčbě, rehabilitaci a po kontrole plicních funkcí propuštěn s doporučením pokračovat v zahájené léčbě (Thyrozol 10 mg/den, Prednison 10 mg/den, Betaloc Zok 10 mg/den, Furon, draslík a hořčík). Pacientovi byla diagnostikována amiodaronem indukovaná hypertyreóza a intersticiální pneumonitida.

', qs: [ { task: '

Proč byla dávka Betaloc Zoku redukována postupně?

', answer: '

Terapie b-blokátory by se neměla přerušovat náhle z důvodu rizika rebound fenoménu.

' } ] } ], biblio: ["

Grabczak, Elżbieta Magdalena, et al. Amiodarone induced pneumonitis and hyperthyroidism: case report. Pol Arch Med Wewn. 2008; 118(9): 524-529.

http://pamw.pl/en/issue/article/18846989

"], editors: '04', index: 65 }, //CNS { title: 'CNS1cz', cathegory: 2, keywords: ["schizofrenie", "klozapin", "alprazolam", "tabák", "cytochrom P450"], parts: [ { text: '

37 - letý muž s 8 - letou historii léčby paranoidní schizofrenie Denzapinem 200 mg p.o./den. Pacient byl v poslední době několikrát hospitalizován. Po hospitalizaci žije s matkou v bytě a dochází do komunitního terapeutického centra. Od počátku dospívání začal kouřit cigarety a od svých 23 let konopí, na které si rychle zvykl a kouřil jej denně. Jeho spotřeba byla pravidelně měřena objektivně prostřednictvím stanovení v moči v rámci screeningu v terapeutické komunitě. Screening byl vždy na jiné návykové látky negativní. Po poslední hospitalizaci byl pacient nastaven na léčbu klozapinem 600 mg/den. Vzhledem k zintenzivnění paranoidní představy se dávka zvýšila na 700 mg , poté se výrazně zmírnila paranoidní symptomatologie.

', qs: [ { task: '

Jaká je účinná látka Denzapinu ?

', answer: '

Klozapin.

' }, { task: '

Jaký je mechanismus účinku klozapinu a jaké je jeho terapeutické referenční rozmezí?

', answer: '

klozapin - vykazuje antipsychotický účinek lehce odlišný od klasických antipsychotik. Klozapin je antagonista serotoninu se silnou vazbou na 5-HT 2A/2C subtyp receptoru. Také vykazuje silnou afinitu k několika dopaminergním receptorům, ale vykazuje pouze slabý antagonismus na dopaminové D2 receptory, jejichž blokáda je považována za nositele hlavního antipsychotického účinku. Terapeutické rozmezí je od 50 do 450 ng/ml.

' } ] }, { text: '

Příležitostně byl k léčbě komedikován Xanax. Hladiny klozapinu byly pravidelně kontrolovány z důvodů ověření pacientovy adherence, potvrzení správné dávky a dosažení terapeutického rozmezí.

', qs: [ { task: '

Co je účinnou látkou přípravku Xanax? Jaký je jeho mechanismus účinku a pravděpodobné důvody jeho medikace?

', answer: '

alprazolam - stejně jako ostatní benzodiazepiny, má vysokou afinitu k benzodiazepinovým vazebným místům GABAA receptorového komplexu. Usnadňuje inhibiční neurotransmisi gama-aminomáselné kyseliny, která zprostředkovává jak pre- tak i postsynaptickou inhibici v centrálním nervovém systému (CNS).

' } ] }, { text: '

Tři měsíce po augmentaci dávky Denzapinu se pacient rozhodl přestat kouřit tabák i marihuanu (den 0). Abstinence konopí byla objektivně ověřena vyšetřením moči na obsahové látky konopí. Přibližně 1 měsíc po zanechání kouření se pacient dostavil do ambulance, rozrušený, stěžuje si na akustické halucinace a hlasy v hlavě. Symptomy se dále zhoršují během následujících 2 měsíců. Tři a půl měsíce po zanechání kouření byla vyšetřena hladina klozapinu. Naměřená hladina byla 1328 ng/ml a 715 ng/ml v případě jeho metabolitu norklozapinu.

', qs: [ { task: '

Co byste navrhovali udělat?

', answer: '

Přerušit podávání klozapinu, provést TDM.

' }, { task: '

Co by mohlo být příčinou tohoto stavu?

', answer: '

Odvykání kouření u pacientů léčených léky, které jsou substráty CYP1A2 může vést ke zvýšení plazmatických hladin těchto léčiv a v důsledku toho i k nežádoucím účinkům. Polycyklické aromatické uhlovodíky z tabákového kouře jsou induktorem CYP1A2, zejména v případě CYP1A2 * F polymorfních pacientů.

[[14_CZ.jpg]]

' } ] }, { text: '

Dávka Denzapinu byla snížena na 500 mg / den a psychotická symptomatologie zmizela. Během 1 týdne se koncentrace v plazmě vracejí na úroveň před odvykáním kouření.

', qs: [ { task: '

Byly tyto NÚ očekávatelné a preventabilní?

', answer: '

Ano – více v SPC Denzapin/klozapin.

' } ] } ], biblio: ["

Zullino DF, Delessert D, Eap CB, Preisig M, Baumann P. Tobacco and cannabis smoking cessation can lead to intoxication with clozapine or olanzapine. Int Clin Psychopharmacol. 2002;17(3):141-143.

http://www.ncbi.nlm.nih.gov/pubmed/11981356

"], editors: '01', index: 29 }, { title: 'CNS2cz', cathegory: 2, keywords: ["epilepsie", "erythromycin", "karbamazepin", "valproát"], parts: [ { text: '

U 21 letého pacienta byla diagnostikována idiopatická epilepsie. Léčba byla zahájena přípravkem Convulex 500 mg/den, s nedostatečnou odpovědí. Proto byl dále pacient léčen přípravkem Tegretol 1000 mg/den s dobrou odpovědí (bez záchvatů po dobu posledních 2,5 roku).

Následně pacient přichází na kontrolu pro přibližně 1 den trvající závratě, bolesti hlavy, zvracení, rozmazané vidění a nejistou chůzi. Při dotazu neguje horečku, záchvat, ztrátu vědomí, poranění hlavy. Také neguje nadužívání přípravku Tegretol nebo chybu v užívání přípravku. Na další dotaz na užívání jiných léčiv nebo doplňků stravy odpovídá, že dostal předepsaný Erythrocin 500 mg ( 2-2-2 ) pro atypickou pneumonii. Přípravek užívá poslední 2 dny.

', qs: [ { task: '

Co je účinnou látkou přípravku Convulex? Co je hlavním mechanismem účinku tohoto léčiva?

', answer: '

Convulex - valproát (kyselina valproová) –GABA enhancer - potenciace inhibičního účinku GABA prostřednictvím účinku na její syntézu a metabolismus.

' }, { task: '

Co je účinnou látkou přípravku Tegretol? Co je hlavním mechanismus účinku tohoto léčiva?

', answer: '

Tegretol - karbamazepin – mechanismus účinku není kompletně objasněn; pravděpodobně blokuje Na+ kanály, tím stabilizuje hyperexcitované nervové membrány, zabraňuje opakovaným neuronálním výbojům a snižuje synaptické rozšiřování excitačních impulsů.

' }, { task: '

Jaké je doporučené terapeutické rozmezí účinné látky přípravku Tegretol?

', answer: '

Doporučená terapeutická plazmatická koncentrace je v rozmezí 5- 12 ug/mL.

' } ] }, { text: '

Pacient byl afebrilní, při vědomí a dobře orientovaný. Jeho puls byl pravidelný 64/min, TK 115/65 mmHg, dechová frekvence 24/min, EKG bez výraznějších patologií. Dále byl přítomen oboustranný nystagmus a ataktická chůze, nicméně bez jiných výraznějších poruch koordinace, bez třesu. Stejně tak byly negativní meningeální příznaky i ostatní neurologické vyšetření bylo normální. Hemoglobin 12 g/dL, destičky 346000 / mm3, sérový Na+ 132 mmol/L, K+ 4,9 mmol/L, močovina 18,7 mg/dL, koncentrace kreatininu v séru 0,7 mg /dL, hladina glukózy v krvi 4,9 mmol /L, bilirubin v séru 0,27 mg/dL, alkalická fosfatáza 128 U/L. Hladina karbamazepinu v séru 21 ug/mL.

', qs: [ { task: '

Co může být příčinou těchto příznaků?

', answer: '

Pravděpodobně se jedná o interakci mezi erytromycinem a karbamazepinem. Erytromycin je makrolidové antibiotikum s inhibičním účinkem na CYP3A4 – hlavní biotransformační enzym karbamazepinu. Tímto mechanismem dochází při stejné dávce karbamazepinu ke zvýšení jeho plazmatických koncentrací a zvýšení rizika rozvoje toxicity.

' } ] }, { text: '

Pacient udává, že před rokem měl podobné příznaky poté, co omylem užil dvojitou dávku Tegretolu. Příznaky tehdy odezněly po vynechání dávky a opětovném správném užívání přípravku.

', qs: [ { task: '

Jaký byste navrhovali další postup léčby?

', answer: '

Vysadit Tegretol, podat antiemetika, (setrony, thiethylperazin), i.v. fyz. roztok, TDM karbamazepinu, opatrná titrace dávky.

' } ] }, { text: '

Erythromycin byl vysazen a karbamazepin byl vysazen na jeden den a další den byla podána snížená dávka 10 mg/kg/den. Bylo plánováno MRI vyšetření a lumbální punkce pro vyšetření cerebrospinálního moku, nicméně po vysazení léčby se příznaky zlepšily natolik, že bylo vyšetření zrušeno. Pacient byl po dohodě s rodiči propuštěn domů po 48 hodinách hospitalizace. Při následné kontrole byl pacient bez jakýchkoliv známek neurotoxicity, se sérovou hladinou karbamazepinu 7,6 ug/mL. Pacient byl dále sledován bez záchvatů nebo známek ataxie.

', qs: [ { task: '

Je tato interakce očekávatelná a preventabilní?

', answer: '

Ano, je očekávatelná - viz SPC. Je preventabilní, erytromycin, potažmo makrolidy jsou známy inhibičním účinkem na cytochrom P450.Pro léčbu pneumonie mohlo být použito jiné antibiotikum.

' }, { task: '

Co byste mohli udělat pro prevenci této nežádoucí interakce?

', answer: '

TDM v průběhu terapie erytromycinem, snížení dávky Tegretolu, změna antibiotické léčby (amoxicilin+klavulanát, cefalosporin).

' } ] } ], biblio: ["

Shrestha D, Dhakal AK, Singh KK. Erythromycin induced carbamazepine toxicity - a preventable drug interaction. Journal of Chitwan Medical College. 2013; 3(6):52-53.

http://www.nepjol.info/index.php/JCMC/article/view/9556

"], editors: '01', index: 34 }, { title: 'CNS3cz', cathegory: 2, keywords: ["Parkinsonova choroba", "zácpa", "levodopa", "karbidopa", "entakapon", "amantadin", "escitalopram", "klonazepam", "dokusát", "quetiapin"], parts: [ { text: '

Do ordinace přichází 72-letý pacient v doprovodu svého syna, ke kterému se nedávno přistěhoval. V anamnéze je nejdůležitější Parkinsonova choroba, která mu byla diagnostikována před 13 lety.

Pacient je zmatený a potřebuje pomoc s chůzí na delší vzdálenosti. Potřebuje stálý dohled, protože často padá. Jeho tvář má omezenou mimiku, hlas je slabý. Nápadný je klidový tremor v jeho levé paži. V poslední době má problémy se sám najíst a napít, nemůže používat příbor. Jeho největším problémem, jak udává jeho syn, je však těžká zácpa s velmi objemnou a tvrdou stolicí.

Pacient v současné době užívá následující léčiva:

Padovel 100 mg/25 mg/200 mg; 5x denně

PK-Merz; 1–½-1

Elicea Oro Tab 10 mg, tablety dispergovatelné v ústech; 1-0-0

Rivotril 0,5 mg; dle potřeby před spaním 1 tabletu

Dioctyl 100 mg kapsle; dle potřeby 1 kapsli

FERRO-FOLGAMMA; 0-1-0

Do medikace byl nedávno přidán také přípravek Derin 25 mg potahované tablety.

', qs: [ { task: '
Vyhledejte účinné látky přípravků a charakterizujte je. Jaký je mechanismus jejich účinku? (pozn. perorální přípravek Dioctyl není v ČR registrovaný, jeho SPC se dá vyhledat například na medicines.org.uk, nicméně stejnou účinnou látku obsahuje u nás registrovaný Yal rektální gel)
', answer: '

Padovel 100 mg/25 mg/200 mg obsahuje kombinaci látek levodopa, karbidopa a entakapon, jedná se o antiparkinsonika. Levodopa je prekursor syntézy dopaminu, karbidopa je inhibitor dopa-dekarboxylázy a entakapon je inhibitor COMT (catechol-O-metyl transferázy).

PK-Merz obsahuje antiparkinsonikum amantadin s nepřímým agonistickým účinkem na striatální dopaminový receptor a s anticholinergími účinky.

Elicea je antidepresivum, účinnou látkou je escitalopram (ze skupiny SSRI - selektivních inhibitorů reuptaku serotoninu).

Rivotril obsahuje klonazepam, pacient ho má užívat před spaním jako hypnotikum.

Dioctyl obsahuje dokusát sodný. Jedná se o povrchově aktivní látku. V kombinaci se sorbitolem se používá jako laxativum.

FERRO-FOLGAMMA obsahuje kombinaci železa, kyseliny listové a cyanokobalaminu (vitamin B12). Používá se při poruchách krvetvorby nebo hypovitaminózách.

Derin obsahuje quetiapin, atypické antipsychotikum. Quetiapin blokuje 5-HT2 receptory

více než D2, ale méně než ostatní antipsychotika II. generace. 5-HT2A/D2 antagonizmus zvyšuje výdej dopaminu do prefrontálního kortexu a striata a vede k redukci motorických nežádoucích účinků.

' }, { task: '

Jaká je další indikace amantadinu kromě terapie Parkinsonovy choroby?

', answer: '

Je to protichřipkové antivirotikum.

' }, { task: '

Pacient užívá kromě antiparkinsonik ještě látky z jiných lékových skupin. Zkuste odhadnout, v jaké indikaci je pacient užívá.

', answer: '

U Parkinsonovy choroby se často objevují poruchy nálady nebo deprese a demence, které přímo souvisejí se samotným základním onemocněním. Tento pacient má zřejmě depresivní symptomy a užívá proto antidepresivum escitalopram. Nežádoucím účinkem antiparkinsonik mohou být halucinace (které jsou o tohoto pacienta řešeny antipsychotikem quetiapinem) a poruchy spánku, pro které užívá benzodiazepin. Riziková jsou v tomto směru například antiparkinsonika s anticholinergním účinkem, jako je amantadin, který pacient užívá. Dokusát sodný užívá pacient ke zvládnutí zácpy. Dále má přípravek FERRO-FOLGAMMA k prevenci poruchy krvetvorby (možný nedostatečný příjem potravou a zároveň je neutropenie vzácným nežádoucím účinkem quetiapinu).

' }, { task: '

Znáte nějaká jiná antiparkinsonika než výše zmíněná?

', answer: '

Pro podrobný přehled viz učebnice farmakologie, pro potřeby tohoto textu jen vyjmenujeme agonisty různých typů dopaminových D receptorů bromokriptin, dihydroergokriptin, pergolid, ropinirol, pramipexol a rotigotin, inhibitory MAO B selegilin a rasagilin a anticholinergika biperiden a procyklidin.

' } ] }, { text: '

Zácpa je častý projev Parkinsonovy choroby a může se objevit mnoho let před vypuknutím motorických projevů. Je způsobena kombinací mechanismů: zpomalený průchod tráveniny tlustým střevem, slabý břišní lis a porucha motorické kontroly análního svěrače (může docházet k funkční obstrukci, kdy se sfinktery a m. puborectalis během defekace paradoxně kontrahují místo relaxace). Další přispívající komplikací bývá nevhodné složení stravy a nízký příjem tekutin nemocnými.

', qs: [ { task: '

Která léčiva užívaná pacientem mohou jeho zácpu zhoršovat?

', answer: '

Anticholinergní látky (amantadin), železo, quetiapin, escitalopram.

' }, { task: '

Vyjmenujte příklady dalších léčiv, která mohou způsobit zácpu.

', answer: '

Například antacida, opioidy, nesteroidní antiflogistika, vyšší příjem vápníku, anticholinergika, tricyklická antidepresiva, diuretika (zejména kvůli hypokalémii), vápníkové blokátory…

' }, { task: '

Zopakujte si, jaká znáte laxativa a doporučte nějaké pro tohoto pacienta.

', answer: '

Přehled laxativ viz učebnice farmakologie. Především na horní část trávicí trubice působí domepridon, uplatní se zejména u poruchy vyprazdňování žaludku. Při těžké zácpě je možné krátkodobě užívat pikosulfát nebo bisakodyl.

Bezpečné i pro dlouhodobější užívání jsou přípravky s obsahem laktulózy nebo makrogolu (polyetylenglykolu). Vyprazdňování může být významně usnadněno pomocí glycerinových čípků, nebo perorálních globulí s obsahem vazelíny. Při chronické zácpě je možné užívat také linaklotid (peptid, agonista receptoru guanylátcyklázy GC-C).

' }, { task: '

Doporučte režimová opatření ke zmírnění obstipace.

', answer: '

Pacientovi doporučíme dostatečný přísun tekutin a rozpustné vlákniny ve stravě. Někteří autoři doporučují probiotika. Prospěšný je také dostatek pohybu, což může zpomalit i progresi motorických příznaků základního onemocnění. Dysfunkce pánevního dna se řeší nácvikem, když toto řešení nepostačuje, lze ve vážných případech aplikovat botulotoxin do příslušných svalů nebo provést vyústění střeva stomií. Gastrokolický reflex usnadňuje defekaci během 90 minut po jídle, je vhodné tento reflex dále posilovat cílenou snahou o defekaci v tomto časovém okně.

' }, { task: '

Která léčiva mohou způsobit projevy podobné Parkinsonově chorobě?

', answer: '

Například klasická antipsychotika, některá H1 antihistaminika 1. generace (thiethylperazin, prometazin), metoklopramid, α-methyldopa, tricyklická AD, fenytoin…

' } ] } ], biblio: ["

Noll LM. Management of Constipation in Patients With Parkinson Disease. Journal of Hospice and Palliative Nursing. 2013;15(7):388-389.

http://www.medscape.com/viewarticle/810992

"], editors: '03', index: 36 }, { title: 'CNS4cz', cathegory: 2, keywords: ["midazolam", "fentanyl", "benzokain", "methylenová modř", "methemoglobinémie"], parts: [ { text: '

Tříletý chlapec prodělal tonsilektomii a v průběhu 3 týdnů po zákroku se u něj objevovaly epizody krvácení z horních částí GIT. Krvácení se projevovalo hematemezou a postupným poklesem hemoglobinu až na 46 g/l. Byla provedena přímá laryngoskopie bez nálezu krvácivého ložiska. Pacient dostal transfuzi plné krve a byl doporučen na oddělení pediatrické gastroenterologie k endoskopickému vyšetření za účelem nalezení krvácivého ložiska v GIT. Premedikace před vyšetřením zahrnovala podání Dormicum inj. sol. v dávce 0,1 mg/kg i.v., dále Fentanyl TORREX 50 mcg/ml inj. sol. v dávce 1 μg/kg i.v. a ošetření orofaryngu sprejem s obsahem 20 % benzokainu.

', qs: [ { task: '

Jaké jsou mechanismy účinku léčiv použitých v premedikaci? Do jakých lékových skupin patří a proč byly použity?

', answer: '

midazolam – látka ze skupiny benzodiazepinů. Jeho mechanismem účinku je vazba na GABAA receptor a otevření chloridového kanálu, což vede k inhibičnímu vlivu na CNS. Benzodiazepiny mají hypnosedativní, antikonvulzivní, anxiolytické, myorelaxační a amnestické účinky a jsou používány v široké paletě indikací.

fentanyl – agonista μ‑opioidních receptorů. Jde o velmi silné analgetikum. Používá se v léčbě bolesti, při neuroleptanalgezii (kombinace antipsychotik a opioidních analgetik) a ataranalgezii (kombinace benzodiazepinů a opioidních analgetik).

benzokain – lokální anestetikum esterového typu. Jeho mechanismem účinku je blokáda Na+ kanálů v neuronech senzitivního nervstva. Používá se především k anestezii kůže a sliznic.

Fentanyl a midazolam byly použity v režimu ataranalgezie ke zklidnění pacienta a proti bolesti při vyšetření. Benzokain byl aplikován ke zmírnění nepříjemných a bolestivých pocitů při zavádění sondy do GIT.

' }, { task: '

Proč byla ze skupiny benzodiazepinů zvolena právě účinná látka přípravku Dormicum?

', answer: '

Midazolam má vystupňované hypnosedativní účinky a krátký biologický poločas. Je tedy nejvhodnější k zabezpečení zklidnění pacienta po krátkou dobu trvání vyšetření a nezatěžuje jej protrahovanou sedací.

' }, { task: '

Jaké přípravky (lékové formy, indikace) s obsahem benzokainu jsou registrovány v ČR?

', answer: '

Orální pastilky Neoseptolette Duo, Olynhexo, Hexoral, Benzocain LIFE PHARMA, masážní roztok na dásně Herbadent a orální sprej Neoseptolette Duo. Přípravky se používají při infekcích dutiny ústní a hltanu, ev. při onemocněních dásní.

' } ] }, { text: '

Endoskopie proběhla bez komplikací, zdroj krvácení však lokalizován nebyl. Po dokončení vyšetření se u chlapce objevuje progresivní cyanóza. Dítě je neklidné, podrážděné. Saturace kyslíkem zjištěná pulzní oxymetrií klesá z 97 % na 80 % i přesto, že je pacient napojen na zdroj O2 pomocí nosních sond. Pacient má dobrou krevní cirkulaci, teplé končetiny, dýchání je volné, bez obstrukce. Má normální krevní tlak. Postupně se však rozvíjí tachykardie (140–170 tepů/min) a tachypnoe (40–50 dechů/min). Také cyanóza progreduje a dítě začíná být zmatené a agitované. Rentgen hrudníku je bez nálezu. Při odběru krve pro přesnější stanovení krevních plynů je zjištěno, že krev má čokoládovou barvu. Vyšetření potvrzuje výraznou hypoxii, hypokapnii a kompenzovanou metabolickou acidózu.

', qs: [ { task: '

Pro jakou diagnózu svědčí symptomatologie a výsledky vyšetření?

', answer: '

Jde o získanou methemoglobinémii.

' }, { task: '

Které léčivo pravděpodobně tento stav způsobilo a jak?

', answer: '

Methemoglobinémie je vzácným nežádoucím účinkem lokálních anestetik (LA). Amidová LA se v organismu metabolizují na aminy, které oxidují hemové Fe2+ na Fe3+. Benzokain je sice esterového typu, ale obsahuje aminoskupinu přímo ve své struktuře.

' } ] }, { text: '

Pacientovi podáno léčivo X v dávce 1 mg/kg i.v. a rychle došlo k návratu stavu do normálu. Ustoupila cyanóza, normalizoval se stav krevních plynů. Chlapec byl propuštěn druhý den ráno bez následků. Léčivo X má charakteristickou modrou barvu a při externím podání vykazuje mírné antiseptické účinky. Dříve se například používalo jako složka nosních kapek. Látka se dále běžně využívá v textilním průmyslu a uplatnění nachází také v akvaristice.

', qs: [ { task: '

Jaké léčivo bylo dítěti podáno? Jaký je pravděpodobný mechanismus jeho účinku jako „antidota“?

', answer: '

Lékem volby je methylenová modř (methylthioniniumchlorid), která má redukční schopnosti a urychluje tedy přeměnu methemoglobinu zpět na hemoglobin (Fe3+ na Fe2+).

' }, { task: '

Jaký vitamin by se také mohl k terapii použít?

', answer: '

Redukční vlastnosti využitelné v terapii methemoglobinémie má také vitamin C.

' } ] } ], biblio: ["

Dashan, Ahmed; Donovan, G. Kevin. Severe methemoglobinemia complicating topical benzocaine use during endoscopy in a toddler: a case report and review of the literature. Pediatrics. 2006;117(4):e806-e809.

http://pediatrics.aappublications.org/content/117/4/e806.short

"], editors: '06', index: 52 }, { title: 'CNS5cz', cathegory: 2, keywords: ["Parkinsonova choroba", "angina pectoris", "cévní mozková příhoda", "levodopa", "entakapon", "karbidopa", "pramipexol"], parts: [ { text: '

Muž (77 let, 95 kg, 178 cm) byl přijat do nemocnice s poruchou řeči a slabostí levé strany těla. Jeho anamnéza zahrnuje anginu pectoris a Parkinsonovu nemoc (PN). Pravidelně užívá tato antiparkinsonika: Isicom 100 (1,5 tbl 4× denně), Oprymea 0,52 (3 tbl 3× denně), Comtan (1 tbl 4× denně) a do hospitalizace byl jeho stav kompenzovaný, pouze se slabým klidovým tremorem rukou.

', qs: [ { task: '

Jaké účinné látky obsahují přípravky, které pacient užívá? Jaké jsou jejich mechanismy účinku?

', answer: '

levodopa (L-DOPA, 3,4-dihydroxy-L-fenylalanin) – prekurzor v biosyntéze dopaminu. Je užívána jako proléčivo ke zvýšení hladin dopaminu, protože je schopná procházet hematoencefalickou bariérou, což dopamin nedokáže. Jakmile levodopa projde do CNS, je metabolizována pomocí DOPA-dekarboxylázy na dopamin. Dopamin ale vzniká z levodopy i na periferii, což vyvolává gastrointestinální a kardiovaskulární nežádoucí účinky. Současně podávaná karbidopa inhibuje DOPA-dekarboxylázu pouze na periferii. Tím lze jednak redukovat dávku levodopy a jednak snížit nežádoucí účinky léčby.

pramipexol – non-ergolinový agonista dopaminových receptorů (D1 a D2). Využívá se v monoterapii i v kombinované léčbě Parkinsonovy nemoci.

entakapon – inhibitor COMT (katechol-O-methyltransferázy). Tento enzym na periferii odbourává levodopu a snižuje tak množství, které je k dispozici pro absorpci do CNS. Použití entakaponu umožňuje snížit dávky levodopy.

' } ] }, { text: '

Diagnostikováno bylo intracerebrální krvácení v oblasti bazálních ganglií a rozšíření mozkových komor (hemoragická CMP). Pacient musel být zaintubován a byla provedena ventrikulostomie. K zachování enterální výživy byla orální cestou zavedena do žaludku sonda. Pacientovi byl podáván přípravek enterální výživy Nutren 1,5 (Nestlé Nutrition, Glendale, USA) kontinuálně 70 mL/h (proteiny 0,9 g/kg/den), zvýšená potřeba proteinů je kryta 2 sáčky přípravku Prosource (Medtrition Inc., Lancaster, USA) denně (proteiny 1,4 g/kg/den). Do sondy byla také podávána nadrcená léčiva spolu s vodou, po jejich aplikaci následovalo vždy ještě podání čisté vody do sondy k proplachu.

', qs: [ { task: '

Jaká jsou doporučení pro denní příjem proteinů pro zdravého dospělého člověka, zdravého seniora a pro polymorbidní pacienty?

', answer: '

Zdravý dospělý: 0,6‒1,0 g/kg denně

Zdravý senior: 0,9‒1,1 g/kg denně

Polymorbidní pacient: až 1,5 g/kg denně

U polymorbidních nebo kriticky nemocných záleží na charakteru onemocnění. Jiná doporučení budou pro onkologického pacienta s rysy malnutrice (vyšší dávky) a jiná u pacienta s chronickým onemocněním ledvin (nízkoproteinová dieta).

' } ] }, { text: '

V průběhu dvou týdnů se pacientův stav z hlediska hemoragické CMP zlepšil, zhoršily se ale příznaky PN (tremor, výrazná rigidita). Dávky antiparkinsonik po dobu hospitalizace zůstaly nezměněny. Neurolog vyslovil podezření na snížení absorpce levodopy a upravil léčebný režim pacienta. Během dne následujícího po změně režimu se rigidita a tremor výrazně zlepšily.

', qs: [ { task: '

Jakým mechanismem je levodopa absorbována do krevního oběhu a do mozku?

', answer: '

Levodopa (derivát fenylalaninu) je absorbována jak ze střeva do krevního oběhu, tak z oběhu do CNS prostřednictvím přenašeče pro velké neutrální aminokyseliny (LNAA transporter). Tento přenašeč funguje v souladu s kinetikou dle Michaelise-Mentenové – jde tedy o saturabilní mechanismus. Malé množství levodopy proniká také pasivní difúzí.

' }, { task: '

Jaké látky mohou kompetovat s absorpcí levodopy?

', answer: '

Levodopa kompetuje o přenašeč právě s velkými neutrálními AMK jako jsou fenylalanin, leucin, izoleucin, tryptofan a tyrosin. Strava bohatá na proteiny podávaná zároveň s levodopou snižuje její absorpci a tedy i účinnost.

' }, { task: '

Jak byste změnili terapeutický režim u pacienta, aby byla zmírněna tato interakce?

', answer: '
  1. Nepodávat zároveň stravu bohatou na bílkoviny a levodopu – v případě pacienta z kazuistiky tedy změnit kontinuální podávání enterální výživy na bolusové dávky, mezi kterými je možné podat levodopu.
  2. Snížit celkový příjem proteinů ve stravě – v případě našeho pacienta zcela vyřadit přípravek Prosource. Doporučení pro parkinsoniky je přijímat 0,8 g/kg proteinů denně, což je u starších a polymorbidních pacientů de facto nízkoproteinová dieta. Pro domácí ošetřování se potom doporučuje dieta s redistribucí proteinů – pokrmy bohaté na proteiny jsou konzumovány hlavně na noc, kdy tolik nevadí případné snížení účinku levodopy.
  3. Zvýšit dávku levodopy, a tedy zvýšenou koncentrací zvrátit kompetici s aminokyselinami o přenašeč ve prospěch levodopy.
' } ] } ], biblio: ["

Cooper, Mandelin K.; Brock, David G.; McDaniel, Cara M. Interaction between levodopa and enteral nutrition. Annals of Pharmacotherapy. 2008;42(3):439-442.

http://aop.sagepub.com/content/42/3/439.short

", "

Chocenská, Eva. Parkinsonova choroba a vliv bílkovin ve stravě. Sestra. 2010, 6.

http://zdravi.e15.cz/clanek/sestra/parkinsonova-choroba-a-vliv-bilkovin-ve-strave-452682

"], editors: '06', index: 56 }, { title: 'CNS6cz', cathegory: 2, keywords: ["dimenhydrinát", "závislost", "escitalopram", "mirtazapin"], parts: [ { text: '

33 letá vdaná žena se čtyřmi dětmi byla odeslána do psychiatrické ambulance lékařem z pohotovosti kvůli abstinenčním příznakům, 3 dny po vysazení dimenhydrinátu. Dimenhydrinát užívala 12 let v denní dávce 300 mg. Žena nebyla schopná si látku opatřit kvůli změnám v legislativě, podle nichž bylo možné získat dimenhydrinát pouze na lékařský předpis. Žena si stěžovala na nevolnost, zvracení (20 x denně), třes rukou, podrážděnost, pocení, ospalost, bolest hlavy, špatnou chuť k jídlu, hypersomnii. Somatické vyšetření ukázalo mydriázu a mírný třes rukou. Pacientka splňovala kritéria MKN-10 pro “závislost na sedativech, hypnotikách nebo anxiolytikách s vysazením” (F13.23). Kritéria byla pozitivní pro látkovou závislost, včetně tolerance, nadměrného užívání, neúspěšných pokusů ukončit užívání a pokračování navzdory vědomí o tom, že látka působí závažné fyzické problémy. EKG ukazovalo sinusový rytmus se srdeční frekvencí 88/min., krevní tlak byl 130/80 mmHg, tělesná teplota byla 36,5 °C. Byly provedeny laboratorní testy včetně krevního obrazu, testů renálních a jaterních funkcí, vyšetření funkce štítné žlázy, hladin sérového feritinu, kyseliny listové a vitaminu B12. Všechny hodnoty byly v normě s výjimkou nízké hladiny vitamin B12 (83 pg/ml) - laboratorní referenční rozmezí bylo 120-883 pg/ml. Hladina dimenhydrinátu v krvi nebyla stanovena kvůli nedostatečnému vybavení pracoviště.

', qs: [ { task: '

Co je to dimenhydrinát? Popište mechanismus jeho účinku.

', answer: '

dimenhydrinát - antagonista převážně H1 receptorů, ačkoliv také interaguje s jinými neurotransmitery, zejména s acetylcholinem (antagonista muskarinových receptorů). Jeho halucinogenní a euforizující účinky vedou závislé osoby ke zneužívání dimenhydrinátu a podobných antihistaminik v kombinaci s jinými látkami. Na ulicích se prodává taková kombinace dimenhydrinátu s heroinem pod názvem “cheese”. Na druhé straně mohou sedativní a anxiolytické účinky vést k užívání a zneužíváni antihistaminik psychiatrické pacienty. Dimenhydrinát se používá hlavně jako antiemetikum a k prevenci a terapii nevolnosti při cestování, nevolnosti z ozařování, pooperačního zvracení, nevolnosti a zvracení vyvolaným léčivy a symptomatické terapii nevolnosti a zvracení v důsledku Meniérovy choroby a jiných vestibulárních poruch.

' }, { task: '

Jaké jsou nejčastější nežádoucí účinky dimenhydrinátu?

', answer: '

Nejčastější nežádoucí účinek je sedace, která se může lišit od slabé ospalosti až ke spánku. Podávání látky může být spojeno s neschopností soustředění, únavou, závratěmi, hypotenzí, svalovou slabostí a inkoordinací. Pokud se sedativní účinek objeví, zmenší se po několika dnech.

' } ] }, { text: '

Po motivačním rozhovoru s pacientkou jí byl opět podán dimenhydrinát ve snížené dávce (250 mg/den) kvůli závažnému abstinenčnímu syndromu. Tři dny po první návštěvě její abstinenční příznaky zcela ustoupily. Byla odebrána podrobná psychiatrická anamnéza: administrace Beckova inventáře deprese (BDI) a Addiction Profile Index (API). Celkové skóre API bylo 6,7 (vážná závislost) a skóre BDI 39, což indikuje těžkou depresi. Pacientka nikdy neužívala nikotin, alkohol, ilegální drogy, netrpěla žádnou chronickou chorobou. Vdala se v šestnácti letech a první dítě měla v devatenácti. Dva roky později, když byla dva týdny těhotná s druhým dítětem, navštívila praktického lékaře kvůli zvracení, ten jí předepsal dimenhydrinát 50 mg/denně a ona si během těhotenství a kojení zvýšila dávku až na 150 mg/denně. Dítě bylo kojeno 3 měsíce a jednou se u něj ve stáří dvou let vyskytnuly febrilní křeče. V tomto roce pacientka znovu otěhotněla a zvýšila si dávku ve třetím trimestru na 300 mg/denně. Když její muž odcestoval za pracovními povinnostmi, rozvinula se u ní depresivní symptomatika jako nespavost a ztráta zájmu. Kvůli žádoucímu sedativnímu účinku látky nemohla ukončit její užívání a porodila zdravého chlapce, kterého kojila dva roky. Když bylo dítěti 9 měsíců, vrátil se její manžel domů, ale jejich manželské problémy zůstaly nevyřešeny. Ve věku 28 let, když byla opět těhotná, navštívila lékařku kvůli kontrole těhotenství, ale o své závislosti jí neřekla. V tomto období musel její manžel opět služebně odcestovat a to spustilo novou epizodu deprese. U nenarozeného dítěte byla zjištěna brániční kýla a ihned po porodu podstoupilo chirurgický zákrok bez komplikací. Tři roky později se pacientka rozhodla skončit s užíváním léku, ale u její matky se rozvinulo nádorové onemocnění a zemřela, takže pacientka užívání ukončit nemohla. Když přišla na kliniku, užívala 300 mg dimenhydrinátu denně bez přerušení 12 let s anamnézou tří nebo čtyř neúspěšných pokusů o ukončení užívání, které byly doprovázeny podobnými abstinenčními příznaky, bez hospitalizace. Kvůli depresi byl předepsán escitalopram 10 mg/denně, mirtazapin 15 mg/denně a vitamin B12, kvůli jeho nedostatku. Dávka dimenhydrinátu byla postupně snižována (50 mg/denně za týden) a látka byla zcela vysazena do pěti týdnů, přičemž jediným abstinenčním příznakem byla mírná nevolnost. Po kontrole za 3 měsíce se skóre BDI u pacientky snížilo a dimenhydrinát neužívala.

', qs: [ { task: '

Co je to escitalopram? Popište jeho mechanismus účinku a hlavní indikace.

', answer: '

escitalopram - selektivní inhibitor zpětného vychytávání serotoninu (SSRI) s vysokou afinitou pro primární vazebné místo. Váže se také s tisíckrát nižší afinitou na alosterické místo serotoninového přenašeče. Escitalopram nemá žádnou nebo má pouze nízkou afinitu pro celou řadu receptorů včetně 5-HT1A, 5-HT2, dopaminové D1 a D2 receptory, α1-, α2-, β-adrenoceptory, histaminový H1 receptor, muskarinový cholinergní, benzodiazepinový a opioidní receptory. Inhibice zpětného vychytávání serotoninu je jediný pravděpodobný mechanismus účinku vysvětlující farmakologické a klinické účinky escitalopramu. Hlavní indikace zahrnují: epizoda velké deprese, panická porucha s agorafobií nebo bez ní, sociální úzkostná porucha, generalizovaná úzkostná porucha, obsesivně-kompulzivní porucha.

' }, { task: '

Jaké jsou hlavní nežádoucí účinky escitalopramu?

', answer: '

Běžné nebo velmi běžné vedlejší účinky zahrnují: zvýšenou chuť k jídlu, zvýšení hmotnosti, úzkost, neklid, abnormální sny, snížené libido, u žen anorgasmie, bolest hlavy, nespavost, somnolence, závrať, parestézie, třes, průjem, zácpa, zvracení, sucho v ústech, u mužů poruchy ejakulace a impotence.

' }, { task: '

Co je to mirtazapin? Popište jeho mechanismus účinku a hlavní indikace.

', answer: '

mirtazapin - určen k terapii epizod velké deprese. Mirtazapin je centrálně aktivní presynaptický α2-antagonista, který zvyšuje centrální noradrenergní a serotonergní neurotransmisi. Zvýšení serotonergní neurotransmise je specificky zprostředkováno cestou 5-HT1 receptorů, protože 5-HT2 a 5-HT3 receptory mirtazapin blokuje. Antagonistická aktivita mirtazapinu na histaminových H1 receptorech souvisí s jeho sedativními vlastnostmi.

' }, { task: '

Jaké jsou nejčastější nežádoucí účinky mirtazapinu?

', answer: '

Velmi běžné nežádoucí účinky zahrnují: zvýšení hmotnosti, zvýšená chuť k jídlu, somnolence, sedace, bolest hlavy, sucho v ústech.

' }, { task: '

Co je to serotoninový syndrom? Popište riziko serotoninového syndromu v souvislosti s escitalopramem a mirtazapinem.

', answer: '

Symptomy serotoninového syndromu mohou být hypertermie, rigidita, myoklonus, nestabilita vegetativního nervového systému s možnými rychlými fluktuacemi vitálních příznaků, změny psychického stavu, které zahrnují zmatenost, podrážděnost, extrémní agitaci prohlubující se až k deliriu a kómatu. Jestliže jsou serotonin aktivní látky kombinovány s mirtazapinem, je zapotřebí dbát zvýšené opatrnosti a pacienta klinicky monitorovat. Terapie mirtazapinem by měla být přerušena, jestliže se taková událost vyskytne a měla by být zahájena podpůrná symptomatická léčba. Z postmarketingové zkušenosti je zřejmé, že se serotoninový syndrom objevuje velmi zřídka u pacientů léčených pouze samotným mirtazapinem.

Kvůli riziku serotoninového syndromu je kontraindikována kombinace escitalopramu s inhibitory MAO-A jako je moklobemid. V kombinaci se selegilinem (ireverzibilní inhibitor MAO-B) je kvůli riziku rozvoje serotoninového syndromu zapotřebí dbát zvýšené opatrnosti.

' } ] } ], biblio: ["

Kaya FD. A Patient With Dimenhydrinate Dependence: A Case Report. Bulletin of Clinical Psychopharmacology. 2014; 24: 184-187.

http://www.scopemed.org/fulltextpdf.php?mno=42988

"], editors: '02', index: 59 }, { title: 'CNS7', cathegory: 2, keywords: ["tramadol", "torpidní lumboischiadický syndrom", "meloxikam", "fluoxetin", "morfin", "metoklopramid", "fentanyl"], parts: [ { text: '

50 letý pacient s bolestmi zad byl odeslán rehabilitačním pracovištěm do zařízení specializovaného na léčbu bolesti kvůli nastavení analgetické medikace v září 2001. Dosavadní neurologická ani rehabilitační terapie neměla významnější efekt. Pacient pracuje jako pomocný stavební dělník a je 3 měsíce v pracovní neschopnosti. Jiným onemocněním netrpí, před 10 lety byl sledován kvůli opakované gastritidě. Stěžuje si na tupé bolesti v dolní části zad, které převažují na pravé straně. Bolesti trvají cca 1 rok a stále se zhoršují. Mají klidový charakter, který výrazně zhoršuje fyzická námaha, občas vystřelují po zevních stranách do dolních končetin. Trápí pacienta v podstatě nepřetržitě, na vizuální analogové škále (VAS 0-10) dosahují stupně 6-7 v klidu a 8-9 při námaze. V noci bolesti pacienta budí. Zobrazovací metody ukázaly degenerativně deformativní změny charakteru spondylartrózy, mírné snížení meziobratlových prostorů L4/5, L5/S1. Šíře páteřního kanálu je normální, minimální protruze ploténky L5/S1. Operační řešení není indikováno. Dg.: torpidní lumboischiadický syndrom s minimálně vyznačenou iritačně zánikovou symptomatologií L5 bilaterálně. Pacient užívá tablety tramadolu 2 × 100 mg.

', qs: [ { task: '

Co je to tramadol a jaký je mechanismus jeho účinku?

', answer: '

tramadol - centrálně působící opioidní analgetikum. Je to neselektivní parciální agonista opioidních μ, δ a κ receptorů s vyšší afinitou k μ-receptorům. Další mechanismy, které přispívají k analgetickému účinku, jsou inhibice zpětného vychytávání noradrenalinu a serotoninu na synapsích.

' }, { task: '

Jaké jsou hlavní nežádoucí účinky opioidních analgetik?

', answer: '

Nauzea, zvracení, zácpa, sucho v ústech, únava, závratě.

' } ] }, { text: '

Jako záchranná medikace při průlomové bolesti byl přidán tramadol ve formě kapek (20-40 kapek při bolesti, možno opakovat po 5 hodinách). Pacient dále dostává meloxikam a současně fluoxetin jako koanalgetikum a pomocné léčivo. Po 2 měsících dochází pouze k mírnému zlepšení obtíží (VAS v klidu 5-6, při námaze 7-8). Z tohoto důvodu je terapie upravena (tramadol ve formě tablet s řízeným uvolňováním 2 x 150 mg a tramadol ve formě kapek dále jako záchranná medikace). Ostatní medikace zůstává zachována.

', qs: [ { task: '

Jaké jsou mechanismy účinku meloxikamu a fluoxetinu a do jakých farmakoterapeutických skupin patří?

', answer: '

meloxikam - nesteroidní protizánětlivé léčivo (NSAID) ze skupiny oxikamů s protizánětlivými, analgetickými a antipyretickými vlastnostmi. Inhibuje COX (enzym odpovědný za přeměnu kyseliny arachidonové na prostaglandin H2). Má vyšší relativní afinitu ke COX-2 než ke COX-1. Rozdíl však není dostatečný k tomu, aby chyběly nežádoucí účinky na GIT.

fluoxetin - antidepresivum ze skupiny SSRI (selektivní inhibitory zpětného vychytávání serotoninu). Zabraňuje zpětnému vychytávání serotoninu z nervové synaptické štěrbiny.

' }, { task: '

Jaké jsou hlavní nežádoucí účinky meloxikamu a nesteroidních antiflogistik obecně?

', answer: '

Hlavní nežádoucí účinky se vyskytují v GIT (peptické vředy, perforace nebo krvácení do GI traktu, někdy fatální, zejména u starších pacientů). Dále se popisuje nevolnost, zvracení, průjem, nadýmání, zácpa, dyspepsie, bolest břicha, meléna, hematemeze, ulcerózní stomatitida, exacerbace kolitidy a Crohnovy choroby. Dlouhodobé podávání vyvolává insuficienci ledvin (intersticiální nefritidu), kyselina acetylsalicylová může způsobit reakci přecitlivělosti (astma z analgetik) zejména u alergiků.

' }, { task: '

Jaká jiná koanalgetika znáte?

', answer: '

Jako koanalgetika mohou být kromě SSRI použity látky ze skupiny tricyklických antidepresiv (amitriptylin, nortriptylin, dosulepin), které mají vlastní analgetický účinek. Nežádoucí účinky nejsou tak časté, protože pro analgetické působení je nutná výrazně nižší dávka než pro působení antidepresivní. Dále mohou být použita antikonvulziva (karbamazepin, gabapentin, valproát, lamotrigin, topiramát)). Pomáhají snižovat dávky ostatních analgetik a zvyšují efekt léčby.

' } ] }, { text: '

Po úpravě terapie byl pacient 10 měsíců relativně stabilizován, postupně se však objevilo zhoršování bolesti. Celková denní dávka tramadolu se pohybovala v rozmezí 400-500 mg bez výraznějšího účinku. Pacient popisuje nevolnost, malátnost a ospalost.

', qs: [ { task: '

Jaká léčiva můžete pacientovi podat pro prevenci nežádoucích účinků opiodních analgetik a v případě intoxikace?

', answer: '

Pro prevenci nežádoucích účinků opioidních analgetik je možné aplikovat laxativa, prokinetika, antiemetika. V případě intoxikace je vhodné podat opioidní antagonisty - naloxon nebo naltrexon.

' } ] }, { text: '

Kvůli nedostatečné kontrole bolesti a nežádoucím účinkům jsou v lednu 2003 nasazeny silné opioidy (morfin ve formě tablet s řízeným uvolňováním 2 x 30mg). Tramadol ve formě kapek zůstal zachován jako záchranná medikace. Současně byl nasazen metoklopramid jako prokinetikum. Ostatní medikace zůstává beze změn. Tato léčba přinesla výraznou úlevu od bolesti (VAS v klidu 1-2, při námaze 5). Terapii však provází silný útlum, nevolnost a zvracení i přes podávání prokinetika. Pacient udává úpornou obstipaci.

', qs: [ { task: '

Jaká laxativa je možné použít při obstipaci po podání opioidů?

', answer: '

Nejvhodnější jsou osmotická laxativa – laktulóza 3 x 1-2 lžíce. Dále je možné použít stimulační laxativa (např. sena, parafinové projímadlo, glycerinový čípek).

' }, { task: '

Co je to analgetický žebříček WHO?

', answer: '

Analgetický žebříček WHO je postup při terapii bolesti. První stupeň žebříčku představuje léčbu mírné až středně silné bolesti a používají se zde neopioidní analgetika (NSAIDs) v monoterapii. Jestliže se po aplikaci maximální dávky těchto léčiv nedosáhne zmírnění bolesti na snesitelnou míru, je nutné přejít na látky druhého stupně žebříčku WHO. Kombinace více látek ze skupiny NSAIDs v tomto případě nemá význam a zvyšuje riziko nežádoucích účinků. Druhý stupeň žebříčku představuje terapii středně silné a silné bolesti. Používají se zde slabé opioidy, přičemž je vhodné jejich podání v kombinaci s NSAIDs, případně s pomocnými analgetiky (koanalgetiky). Jestliže se po aplikaci maximální dávky těchto léčiv (kombinovaných s NSAIDs) nedosáhne zmírnění bolesti na snesitelnou míru, je nutné přejít k látkám třetího stupně žebříčku WHO. Na tomto stupni se používají silné opioidy. Tyto opioidy je možné kombinovat s látkami ze skupiny NSAIDs a koanalgetiky. Není vhodná kombinace silných a slabých opioidů.

' } ] }, { text: '

Kvůli nežádoucím účinkům byla v březnu 2003 indikována rotace opioidu za TTS fentanyl 25 μg (ostatní medikace byla zachována). Pacient udával stejnou úlevu od bolesti jako po aplikaci retardované formy morfinu, ale bez výskytu nevolnosti, zvracení a zácpy. Rovněž útlum byl podstatně nižší. Při námaze nebyla úleva od bolesti dostatečná, proto byla zvýšena dávka TTS fentanylu na 50 μg. Zvýšení dávky se obešlo bez problémů, byl vysazen metoklopramid. Pacient mohl opět pracovat a pravidelně rehabilitovat. Po roční pravidelné rehabilitaci je plánováno snížení, případně vysazení opioidní medikace.

', qs: [ { task: '

Co je to TTS?

', answer: '

TTS (transdermální terapeutické systémy) se používají pro systémovou aplikaci látek. Nejčastěji se používají transdermální náplasti (emplastra transcutanea), které zabezpečují řízené uvolňování léčivých látek. TTS předávají zdravou kůží definované a přesné dávky léku za definovanou jednotku času. Léčivo obchází GIT a je chráněno před metabolizací v játrech v rámci first pass efektu. Výskyt nežádoucích účinků je snížen.

' }, { task: '

Jaká jiná léčiva se podávají ve formě TSS?

', answer: '

Ve formě TTS je dále možné podat např. nikotin (léčba závislosti na tabáku, pro zmírnění touhy po kouření a pro zmírnění abstinenčních příznaků), hormonální přípravky s obsahem estrogenů (hormonální substituční terapie klimakterických obtíží po vymizení měsíčního krvácení) nebo rotigotin (antiparkinsonikum).

' } ] } ], biblio: ["

Cirmanová V. Transdermální systém Neupro v léčbě Parkinsonovy nemoci. Medical Tribune. 2009;28.

http://www.tribune.cz/clanek/15205

", "

Doležal T, Hakl T, Kozák J, Kršiak M, Lejčko J, Skála B, Sláma O, Ševčík P, Vorlíček J. Metodické pokyny pro farmakoterapii bolesti. Bolest. Supplementum, 2009; 2: 1-27.

http://chronickabolest.cz/admin/contentarticle/edit/id/4/pub/content/4/MP_Bolest_2009.pdf

", "

Hřib R, Hakl M. Transdermální fentanyl v léčbě chronické bolesti zad. Neurologie pro praxi. 2005; 6: 39-41.

http://www.neurologiepropraxi.cz/pdfs/neu/2005/01/09.pdf

"], editors: '02', index: 61 }, { title: 'CNS8cz', cathegory: 2, keywords: ["Alzeheimerova choroba", "diabetes mellitus", "donepezil", "morfin", "propofol", "rokuronium", "izofluran", "vekuronium", "neostigmin", "glykopyrolát", "inzulin", "suxamethonium", "metformin", "glimepirid"], parts: [ { text: '

Žena 74 let, 60 kg přijata pro plánovanou oboustrannou mastektomii. Před rokem jí byla diagnostikovaná Alzheimerova choroba a před měsícem diabetes mellitus. Farmakologická anamnéza: Glucophage 500 mg 2x denně, Amaryl 2 mg 1x denně, Aricept 5 mg 1x denně. Při vyšetření pacientka vykazuje zhoršené mentální funkce, strnulý výraz obličeje a zpomalenou motoriku. Všechny laboratorní testy v normálu. Ráno před operací byly pacientce vysazeny všechny léky.

', qs: [ { task: '

Popište mechanismy účinků léků, které pacientka užívá a zařaďte je do farmakoterapeutické skupiny.

', answer: '

Glucophage – metformin – perorální biguanidové antidiabetikum, MÚ: zvyšuje transport glukózy do buněk, snižuje jaterní glukoneogenezi

Amaryl – glimepirid – perorální antidiabetikum, derivát sulfonylurey, MÚ: zvyšuje uvolňování inzulinu z pankreatických beta buněk

Aricept – donepezilcholinomimetikum, MÚ: specifický a reverzibilní inhibitor acetylcholinesterázy

' }, { task: '

Znáte jiné látky ze skupiny inhibitorů acetylcholinesterázy? Čím se odlišují od donepezilu?

', answer: '

Například galantamin, který se liší od donepezilu biologickým poločasem (donepezil 70 h a galantamin 5-7 h) a tím, že galantamin též zvyšuje vnitřní aktivitu acetylcholinu na nikotinových receptorech. Jedná se pravděpodobně o alosterickou modifikaci vazebného místa na receptoru.

' } ] }, { text: '

Pacientka byla uvedena do anestezie i.v. podáním 3 mg morfinu a 100 mg propofolu, načež následovala aplikace 0,6 mg/kg rokuronia. Pacientka byla uměle ventilována 3 minuty. Při pokusu o endotracheální intubaci se objevily spontánní respirační pokusy, z toho důvodu byl přidán bolus 20 mg propofolu a 10 mg rokuronia pro usnadnění intubace. Anestezie byla dále vedena izofluranem, O2, a N2O.

', qs: [ { task: '

Popište medikaci, která byla použita pro celkovou anestezii u pacientky (mechanismus účinku, zařazení do farmakoterapeutické skupiny).

', answer: '

morfin – opioidní analgetikum, MÚ: agonista opioidních receptorů

propofol – intravenózní celkové anestetikum, MÚ: není zcela objasněn, pravděpodobně zvyšuje vazbu inhibičního neurotransmiteru GABA na GABAA receptory

rokuronium – nedepolarizující periferní svalové myorelaxans, MÚ: kompetitivní antagonista na NN-receptorech na nervosvalové ploténce

vekuronium – nedepolarizující periferní svalové myorelaxans, MÚ: kompetitivní antagonista na NN-receptorech na nervosvalové ploténce

izofluran – inhalační celkové anestetikum, MÚ: není zcela objasněn, ovlivňuje fluiditu membrán, interaguje s inhibičními neurotransmitery

' } ] }, { text: '

V 15. a 25. minutě po tracheální intubaci se opět objevily spontánní respirační pokusy. Bylo usouzeno, že rokuronium nemá dostačující myorelaxační efekt a z toho důvodu bylo zaměněno za vekuronium. I tak nedošlo k dostačující svalové relaxaci. Po dokončení 2 hodinové operace u pacientky přetrvávala neuromuskulární blokáda, kvůli které byl podán neostigmin 60 μb/kg a glykopyrolát 10 μg/kg i.v., po kterém stále není pozorováno klinicky adekvátní respirační úsilí. Po dalších 15 minutách je pacientce podán opět neostigmin 10 μb/kg a glykopyrolát 5 μb/kg i.v. Následně dochází k vzestupu krevního tlaku, srdečního rytmu a zvýšení glykémie. Pacientce byl podán inzulin a byla katetrizována, kvůli monitorování renálních funkcí.

', qs: [ { task: '

Porovnejte podané dávky léčiv s dávkami uvedenými v SPC.

', answer: '

morfin – počáteční dávka 10 – 20 mg

propofol – počáteční dávka 1,5 – 2,5 mg/kg à 90 – 150 mg

rokuronium – počáteční dávka 0,6 mg/kg – 2 mg/kg à 36 mg – 120 mg

neostigmin – s.c. a i.m. podání: 0,25 – 0,5 mg, i.v. podání: 0,05 – 0,07 mg/kg à 3 – 4,2 mg

glykopyrolát – 0,2 mg i.v. na 1 mg neostigminu, případně 0,01 – 0,015 mg/kg i.v. při dávce 0,05 mg/kg neostigminu à 0,06 – 0,9 mg při dávce 3 mg neostigminu

' }, { task: '

Jaký je mechanismus antagonismu neostigminu a glykopyrolátu u neuromuskulární blokády?

', answer: '

Glykopyrolát inhibuje účinek acetylcholinu na strukturách inervovaných postgangliovými cholinergními nervy a na hladkých svalech, které odpovídají na acetylcholin, ale nemají cholinergní inervaci. Výsledkem je snížení gastrické, faryngeální, tracheální a bronchiální sekrece. Glykopyrolát antagonizuje muskarinové symptomy (bronchospasmus, bradykardie a intestinální hypermobilita) vyvolané inhibitory cholinesteráz. Neostigmin je inhibitor acetylcholinesteráz, zvyšuje a intenzifikuje muskarinové a nikotinové účinky acetylcholinu. Má i přímé agonistické účinky na nikotinové receptory kosterního svalstva.

' } ] }, { text: '

Pooperační vyšetření krevních plynů ukázalo postupně se zlepšující respirační acidózu za současné manuální respirační asistence. Jakmile byla pacientka při vědomí, měla dostačující respirační úsilí a po výzvě byla schopna pohnout končetinami, byla extubována. 5-10 min po extubaci u pacientky nastala tachypnoe, saturace O2 klesla pod 85 % a pacientka musela být znovu intubována (aplikován propofol 40 mg a suxamethonium 12,5 mg i.v.). Pacientka byla převezena na jednotku intenzivní péče. Další den byla pacientka při vědomí, orientovaná, spontánní dechová aktivita byla adekvátní. Pacientka byla úspěšně extubována a za týden propuštěna do domácího ošetřování.

', qs: [ { task: '

Jak byste vysvětlili pacientčiny problémy?

', answer: '

Prolongovaná neuromuskulární blokáda, která byla způsobena lékovou interakcí mezi donepezilem, jakožto inhibitorem acetylcholinesterázy a nedepolarizujícími svalovými myorelaxancii. Tato situace nastala, protože donepezil byl před operací, vzhledem k jeho dlouhému biologickému poločasu (70 h), vysazen pozdě.

' } ] } ], biblio: ["

Bhardwaj A. et al. Donepezil: A cause of inadequate muscle relaxation and delayed neuromuscular recovery. J Anaesthesiol Clin Pharmacol. 2011;27(2): 247–248

http://www.ncbi.nlm.nih.gov/pmc/articles/PMC3127310/

"], editors: '07', index: 67 }, { title: 'CNS9cz', cathegory: 2, keywords: ["cefalexin", "levofloxacin", "morfin", "laktulóza", "fentanyl", "myoklonus", "asterixis", "delirium", "fyzostigmin", "glykopyrolát", "trazodon", "alprazolam", "klonazepam", "donepezil"], parts: [ { text: '

55letá pacientka s rekurentním ovariálním karcinomem a peritoneálními metastázami (komplikovanými navíc parciální chronickou obstrukcí tenkého střeva) byla hospitalizována pro narůstající abdominální bolest, horečku spojenou s abscesem břišní stěny a dehydrataci. Dosavadní medikace pacientky byla Durogesic 125 MCG/H jednou za 3 dny, Vendal Retard 30 mg dle potřeby, Neurontin 300 mg (1-1-1), Prozac (2-0-2), Trittico AC 150 (0-0-1), Neurol 0,5 dle potřeby.

Vstupní biochemické vyšetření bylo v pořádku až na albumin (1,9 g/dl), draslík (2,2 mmol/l) a hořčík (1,4 mg/dl).

', qs: [ { task: '

Jaké účinné látky jsou obsaženy v použitých léčivých přípravcích a jaké jsou jejich mechanismy účinku?

', answer: '

Durogesic obsahuje fentanyl, opioidní analgetikum působící především na µ receptory.

Vendal Retard obsahuje morfin, opioidní analgetikum působící především na µ receptory a v menším rozsahu i na k receptorech.

Neurontin obsahuje gabapentin, antiepileptikum, které se s vysokou afinitou váže na podjednotku α2δ napěťově řízených kalciových kanálů.

Prozac obsahuje fluoxetin, tedy antidepresivum ze skupiny SSRI.

Trittico AC 150 obsahuje antidepresivum, anxiolytikum, hypnosedativum trazodon, který je antagonistou serotoninových 5-HT2 receptorů a inhibitorem zpětného vychytávání serotoninu.

Neurol 0,5 obsahuje alprazolam, tedy benzodiazepin s krátko až střednědobým účinkem.

' }, { task: '

Přiřaďte ke zmíněným přípravkům pravděpodobné indikace, pro které je pacientka užívala.

', answer: '

Durogesic- analgetikum

Vendal Retard- analgetikum

Neurontin- adjuvantní analgetikum

Prozac- antidepresivum

Tritico AC 150- anxiolytikum s pozitivním vlivem na spánek

Neurol 0,5- sedativum, anxiolytikum

' }, { task: '

Jaké jsou normální hodnoty albuminu, draslíku a hořčíku v krvi?

', answer: '

Normální hladiny jsou: albumin 3,5 – 5,3 g/dl (5-7,7 μmol/l), draslík 3,5 – 5,0 mmol/l (13,7-19,6 mg/dl), hořčík 1,7 – 2,2 mg/dl (0,7-0,9 mmol/l).

' } ] }, { text: '

Při příjmu pacientka užívala přibližně 300 mg p.o. opioidů denně. Dávky opioidů byly přepočítané pomocí perorálního morfinového ekvivalentu (OME). Pacientce kromě vysazení Prozacu byla ponechána její medikace beze změn. Navíc jí byla přidána antibiotika a to cefalexin a levofloxacin.

', qs: [ { task: '

Do kterých skupin patří antibiotika použitá u pacienta a jaký je jejich mechanismus účinku a spektrum působení?

', answer: '

cefalexin - cefalosporinové antibiotikum. Působí baktericidně na G+ i G- bakterie díky tomu, že naruší finální část peptidoglykanové syntézy buněčné stěny baktérií.

levofloxacin - fluorochinolové antibiotikum působící na některé G+ i G- bakterie, které inhibuje DNA-gyrázu (topoizomerázu II) i topoizomerázu IV, čímž dochází k inhibici syntézy nukleových kyselin.

' }, { task: '

Jaké HVLP s obsahem cefalexinu a levofloxacinu mohla pacientka užívat?

', answer: '

Přípravky s cefalexinem jsou v ČR registrovány pouze pro veterinární užití, např. Kefavet Vet. Levofloxacin je dostupný např. v přípravcích Levofloxacin KABI, Tavanic a Oftaquix.

' } ] }, { text: '

Den po přijetí si pacientka stále stěžuje na silnou bolest a nauzeu. Při rozhovoru je pacientka klidná, nejeví žádné známky rozrušení, myoklonu nebo asterixis (flapping tremoru). Její analgetická medikace byla změněna a zahrnovala intermitentní bolusové dávky morfinu 4 mg/hod. Pacientce byl také nasazen Duphalac.

', qs: [ { task: '

Jaká účinná látka je obsažena v přípravku Duphalac a proč byl pacientce podán? Jaké dávkování byste pacientce doporučili?

', answer: '

Duphalac obsahuje laxativum laktulózu, která byla dána preventivně pacientce kvůli obstipačnímu působení morfinu. Doporučené dávkování je 15-45 ml denně, obvykle v jedné denní dávce.

' } ] }, { text: '

Po 5 dnech se u pacientky rozvinul myoklonus, pacientka byla zmatená a v útlumu. Tyto příznaky se objevily současně se zvyšováním dávky fentanylu (na 175 μg/hod) a při současném zachování analgetické terapie morfinu pro terapii průlomové bolesti (celková OME 500 mg/den). Ostatní medikace byla beze změn, jen na noc byl pacientce nasazen Rivotril (0,25 – 0,5 mg) a pacientka si přestala žádat Neurol. Její biochemické parametry byly beze změny. Při vyšetření byla pacientka nadále dezorientovaná, zpomalená, trpící bludy, neschopna udržet pozornost, otázky musely být opakovány 3-4x než na ně pacientka odpověděla. U pacientky se objevil multifokální myoklonus a při vyšetření na asterixis sotva byla schopna dorziflexe zápěstí bez poklesu horních končetin. Pacientce bylo diagnostikováno hypoaktivní delirium.

', qs: [ { task: '

Proč byl pacientce nasazen Rivotril a jakou účinnou látku tento přípravek obsahuje?

', answer: '

Rivotril obsahuje benzodiazepin klonazepam, který má kromě sedativních a anxiolytických účinků i účinky myorelaxační a antikonvulzivní. Díky těmto účinkům mohla být tato látka pacientce nasazena na terapii myoklonu a asterixis.

' }, { task: '

Jaké jsou obecně platné principy při farmakoterapii chronické bolesti a při terapii silnými opioidy?

', answer: '

Při farmakoterapii chronické bolesti je vždy nutno nejdříve zjistit charakter a typ bolesti (nociceptivní, neuropatická apod.) a také její intenzitu, např. pomocí VAS (vizuální analogové škály). U chronické bolesti postupujeme dle tzv. „step up principu“, to znamená, že postupně navyšujeme dávky analgetik, kombinujeme analgetika a také případně podáváme adjuvantní látky/koanalgetika pro potenciaci analgetického účinku. Vše dle doporučení třístupňového analgetického žebříčku WHO.

Zásady opioidní terapie:

' } ] }, { text: '

Deliriózní stav, do kterého se pacientka dostala, byl přisuzován zvýšeným dávkám opioidů, protože ostatní adjuvantní léčiva byla užívána beze změny. Vzhledem k tomu, že pacientka v předchozím roce užívala celou řadu různých opioidů, další rotace opioidů již nebyla možná. Na základě empirických zkušeností s terapií sedace inhibitory acetylcholinesterázy (iACHE), se lékaři rozhodli podat pacientce i.v. fyzostigmin (18 μg/kg) v krátké 5 minutové infuzi, které předcházela ještě aplikace 0,1 mg glykopyrronia. Po 5 minutové latenci u pacientky začal mizet myoklonus i asterixis, pacientka začala reagovat na otázky, odpovídala rychle a správně, reagovala bez problémů na jednoduché pokyny. Její manžel popsal zlepšení jejího stavu za dramatické až zázračné.

', qs: [ { task: '

Proč bylo před fyzostigminem podáno glykopyrronium a jaký je jeho mechanismus působení?

', answer: '

Glykoppyronium (glykopyrolát) je dlouhodobě působící antagonista muskarinových receptorů. Před fyzostigminem byl podán kvůli zablokování periferních receptorů a tedy zabránění periferního působení fyzostigminu. Došlo tak ke zvýšenému centrálnímu působení fyzostigminu a zabránění případnému vzniku cholinergní krize.

' } ] }, { text: '

Po 75 minutách pacientce opět mentální výkon klesá a také se objevuje asterixis ve střední intenzitě. Pacientce byl nasazen p.o. Aricept (5 mg/den) a znovu Prozac. Následující den měla pacientka zlepšenou pozornost, lépe reagovala na otázky, ale střední asterixis stále přetrvával. Pacientce byl proto zvýšen Aricept na 10 mg/den, díky čemuž se její stav opět zlepšil.

Pacientka pokračovala dále ve své analgetické terapii (OME 500-600 mg/den) a za 5 dní byla propuštěna do svého hospice s doporučenou medikací, Aricept 5 mg 2x denně a 1 mg glykopyrronia 3x denně.

', qs: [ { task: '

Jaká účinná látka je obsažena v přípravku Aricept a jaký je její mechanismus účinku?

', answer: '

Aricept obsahuje donepezil, reverzibilní inhibitor acetylcholinesterázy a to především cholinesterázy v mozku. Přípravek je běžně indikován k symptomatické terapii Alzheimerovy choroby.

' } ] } ], biblio: ["

Slatkin, Neal; Rhiner, Michelle. Treatment of opioid-induced delirium with acetylcholinesterase inhibitors: a case report. Journal of pain and symptom management. 2004; 27(3): 268-273.

http://www.sciencedirect.com/science/article/pii/S0885392403005153

"], editors: '04', index: 68 }, // A { title: 'A1cz', cathegory: 0, keywords: ["antiobezika", "maligní hypertenze", "kofein", "urapidil", "isosorbiddinitrát", "amlodipin", "fentermin", "naltrexon", "buspiron", "orlistat", "metformin", "lorkaserin", "rimonabant", "sibutramin", "vareniklin", "bupropion"], parts: [ { text: '

36letá žena se dostavila na pohotovost s ostrou retrosternální bolestí vyzařující dozadu.

V anamnéze pacientka neudává žádné závažné onemocnění, hypertenze nebyla v minulosti zjištěna. Nepracuje, je doma s dětmi. Udává snížení hmotnosti během posledních 3 měsíců o 20 – 25 kg.

Pacientka kouří krabičku cigaret denně už přes deset let. Tři měsíce užívá doplněk stravy Xenadrine na snížení tělesné hmotnosti, denně bere 18-20 tobolek, což přesahuje doporučené dávkování uvedené na obalu výrobku (2 tobolky denně). Další medikaci a příjem alkoholu neguje.

Obr. Složení Xenadrinu (obsah látek ve dvou kapslích):

[[17_U_cz.jpg]]

', qs: [ { task: '

Prohlédněte si složení Xenadrinu. Která účinná látka je pravděpodobně zodpovědná za vliv tohoto přípravku na tělesnou hmotnost? Jaká je DTS této látky?

', answer: '

Hlavní účinnou látkou je kofein. V každé tobolce je ho přibližně 100 mg. DTS kofeinu je 50‑250 mg.

' }, { task: '

Spočítejte (zhruba) denní příjem této účinné látky u pacientky. Jaký je mechanismus jejího účinku a nežádoucí účinky?

', answer: '

Denní příjem kofeinu u této pacientky byl 1800 – 2000 mg. Kofein je antagonista adenosinových receptorů, které mají inhibiční vliv na CNS. Účinek na kardiovaskulární systém je dán především antagonizací A1 receptorů. Nežádoucí účinky zahrnují úzkost, poruchu spánku, hypertenzi a tachykardii, má diuretický a natriuretický efekt.

' } ] }, { text: '

Stav pacientky: tělesná výška 165 cm, hmotnost 70 kg, TK na pravé paži 220/110, na levé paži 230/118, EKG záznam odhalil inverzi vlny T ve svodech III, aVF a V6.

Laboratorní vyšetření: v krevním obraze mírná leukocytóza. Biochemie séra odhalila kalcium 8.3 mg/dl (norma 8.6–10.3 mg/dl), a anion gap 6 mmol/l (norma 7–17 mmol/l). Ostatní vyšetření včetně hladiny troponinu, biochemie moče a screeningu návykových látek z moče byly negativní.

', qs: [ { task: '

Stanovte diagnózu. Jak budete takového pacienta léčit?

', answer: '

Diagnóza je maligní hypertenze vyvolaná předávkováním kofeinem. Léčba spočívá v razantní terapii vazodilatancii a antihypertenzivy, v případě tachykardie se dává betablokátor i.v.

' } ] }, { text: '

Pacientka byla přijata na jednotku intenzivní péče a dostala intravenózně Ebrantil 50 i.v. 1 ampuli pomalu a Isoket roztok 0,1% naředěný na finální koncentraci 0,02%, nejprve bolus 20 ml a pak v pomalé infuzi 20 ml/hod do poklesu tlaku. K další kontrole krevního tlaku byl dále nasazen Amlodipin Accord 10 mg tablety, který užívá i nadále po propuštění do domácího ošetřování.

', qs: [ { task: '

Charakterizujte použitá léčiva, zařaďte je do terapeutických skupin a vysvětlete mechanismus jejich účinku.

', answer: '

Ebrantil obsahuje urapidil, látku se smíšeným α2 agonistickým a α1 antagonistickým účinkem.

Isoket obsahuje isosorbiddinitrát (ISDN). Nitráty mají přímé vazodilatační působení prostřednictvím NO.

amlodipin - kalciový blokátor dihydropyridinového typu, jeho účinek je také vasodilatační.

' }, { task: '

Jakou medikaci je možno předepsat pro snížení hmotnosti?

', answer: '

V současné době (2015) je možné ze skupiny antiobezitik předepsat tobolky s fenterminem (centrální stimulans), tablety s obsahem kombinace naltrexonu (opioidní antagonista) a buspironu (anxiolytikum - parciální agonista 5-HT1A) a přípravky s obsahem orlistatu (inhibitor střevních lipáz). V některých případech je možné předepsat antidiabetikum metformin (u pacientů, u kterých je obezita doprovázena poruchou glukozové tolerance nebo diabetem II. typu).

' }, { task: '

Jaká další antiobezitika znáte? Proč se dnes nedoporučují a nepředepisují?

', answer: '

Elsinorské prášky (IPLP kombinace efedrinu a kofeinu) působí jako centrální stimulans. Pro riziko kardiovaskulárních příhod a závislosti nejsou doporučované. Z podobných důvodů není v zemích EU doporučován fentermin, nicméně HVLP přípravky s touto látkou jsou registrované. Dříve používaný sibutramin (inhibitor zpětného vychytávání serotoninu a noradrenalinu) byl stažen pro zvýšené riziko závažných kardiovaskulárních příhod. Pro zvýšení rizika depresivní příhody byl také stažen rimonabant, antagonista CB1 receptorů. Na trh v USA se dostal přípravek s obsahem lorkaserinu (agonista 5-HT2C receptorů), u nás zatím registrovaný není.

' }, { task: '

Pro tuto pacientku by bylo také vhodné omezit kouření. Jakou medikací můžete pomoci pacientovi, který se snaží přestat kouřit?

', answer: '

Kromě nikotinových suplement je možné předepsat přípravky s obsahem vareniklinu (parciální agonista neuronálních nikotinových receptorů), bupropionu a buspironu.

' } ] } ], biblio: ["

Ahmed I. Malignant hypertension and acute aortic dissection associated with caffeine-based ephedra-free dietary supplements: a case report. Cases Journal. 2009;2:6612.

http://www.ncbi.nlm.nih.gov/pmc/articles/PMC2827053/

"], editors: '03', index: 30 }, { title: 'A2cz', cathegory: 0, keywords: ["salbutamol", "metoprolol", "intoxikace"], parts: [ { text: '

2-letý chlapec náhodně požil salbutamol sulfát (Venetlin® inhalační, 0,5%, 30 ml / láhev), předepsaný pro jeho staršího bratra. Matka, která byla svědkem nehody, okamžitě vzala pacienta k nedalekému lékaři. Požitý objem inhalačního přípravku byl odhadnut na 15 ml.

', qs: [ { task: '

Do jaké skupiny léčiv salbutamol sulfát patří – jaký je jeho mechanismus účinku a v jaké indikaci se používá?

', answer: '

salbutamol - selektivní β2 adrenergní agonista, který působí na hladkou svalovinu dýchacích cest a způsobuje bronchodilataci. Salbutamol vyvolává rychlou (do pěti minut), krátkou (čtyřhodinovou) bronchodilataci u reverzibilní obstrukce dýchacích cest v důsledku bronchiálního astmatu, chronické bronchitidy a rozedmy plic.

' }, { task: '

Jakým způsobem dělíme β2 selektivní adrenergní agonisty, uveďte zástupce?

', answer: '

β2 selektivní adrenergní agonisty dělíme do 2 skupin:

krátkodobě působící = short-acting beta-adrenoceptor agonist (SABA) – například fenoterol, salbutamol, terbutalin, hexoprenalin

dlouhodobě působící = long-acting beta-adrenoceptor agonist (LABA) – například salmeterol, formoterol, klenbuterol, bambuterol, prokaterol

' }, { task: '

Jaké další skupiny léčiv jsou používány jako bronchodilatancia? Jaké jsou jejich mechanismy účinku?

', answer: '

Parasympatolytika (ipratropium, oxitropium, tiotropium) - mechanismus účinku je anticholinergní, tj. antagonizují účinky acetylcholinu na muskarinové receptory v hladkém svalstvu průdušek, což způsobuje bronchodilataci.

Methylxantiny (teofylin, aminofylin, etofylin) - tyto látky působí jako kompetitivní neselektivní inhibitory fosfodiesterázy i neselektivní antagonisté adenosinových receptorů.

' }, { task: '

Vypočtěte, jakou dávku salbutamol sulfátu (v mg) pacient při předávkování požil (při požití 15ml 0,5% přípravku).

', answer: '

V 15 ml 0,5% roztoku je obsaženo 75 mg salbutamol sulfátu.

' } ] }, { text: '

Předávkování sulfátem salbutamolu a dalšími β2 selektivními bronchodilatancii se projevuje nežádoucími beta-sympatomimetickými účinky, včetně aktivace kardio-selektivních β1 receptorů, periferní vazodilatace, svalových křečí, hyperglykémie a hypokalémie.

Sinusová tachykardie se vyvíjí částečně díky snížení periferního odporu spojeného s reflexním zvýšením srdečního výdeje.

', qs: [ { task: '

Navrhněte léčbu předávkování.

', answer: '

Viz následující Část III

' } ] }, { text: '

Léčba je primárně zaměřena především na odstranění požitého β2 agonisty pomocí výplachu žaludku, diurézy a indukce defekace a, pokud je to nutné, mohou být podávány injekčně kardioselektivní β blokátory jako je metroprolol. Neselektivní β blokátory jsou u pacientů s astmatem kontraindikovány. Doporučuje se monitorovat hladinu draslíku a hladinu glukózy v krvi pro případnou hypokalémii a hyperglykémii, a zajistit přívod tekutin cestou infúze.

Důležitější než řešení následků a léčba náhodného požití inhalačního β2 agonisty jsou preventivní opatření.

', qs: [ { task: '

Navrhněte, jakým způsobem byste edukovali rodiče – návrh prevence + případně upozornění, jak se zachovat v případě opakování situace.

', answer: '

Viz následující Část IV

' } ] }, { text: '

Doporučujeme, aby rodiny s kojenci a dětmi předškolního věku uchovávaly inhalační přípravky mimo přístup dětí v pevně uzavřené nádobě, na vyvýšeném místě, nebo dokonce ve skříňce.

Je třeba také zvážit formu balení léku - menší balení je bezpečnější.

U pacienta by při požití nemělo být vyvoláváno zvracení, protože by to vedlo ke ztrátě tekutin a elektrolytů a zhoršení stavu. Z tohoto důvodu, je pro členy rodiny nejlepší, aby pacienta předali okamžitě do lékařské péče.

', qs: [ ] } ], biblio: ["

MATSUDA, S.-I. A case of accidental ingestion of a salbutamol sulfate inhalant, Venetlin®. Tokai Journal of Experimental and Clinical Medicine. 2009; 34(3): 76 - 79.

http://mj-med-u-tokai.com/pdf/340304.pdf

"], editors: '05', index: 38 }, { title: 'A3cz', cathegory: 0, keywords: ["intoxikace", "heroin", "naloxon", "vitamin C", "manitol", "piracetam", "buprenorfin"], parts: [ { text: '

Muž, 16 let, nalezen matkou doma v posteli v bezvědomí, chrčící. Po příjezdu RLP byl muž nalezen ležící na boku, bradypnoe 5/min, cyanóza, bradykardie 45/min, puls hmatný pouze centrálně na a. carotis, hrubá porucha vědomí, bez reakce na algický podnět, miotické, nereagující zornice. V průběhu neodkladných úkonů zjišťována orientačně anamnéza a pomocné symptomy: suspektní toxikoman cca ½ roku, výška/váha přibližně 180/65, bez pozičních traumat a stop po vpiších.

', qs: [ { task: '

Pokuste se nalézt příčinu stavu a navrhnout další postup.

', answer: '

Suspektní intoxikace opioidy (heroin). Postup při předávkování: udržení základních životních funkcí, kontrolovat dýchání, stav vědomí. U pacientů v bezvědomí se zástavou dýchání se indikuje ventilační podpora, intubace a intravenózní aplikace antagonisty opioidů (např. 0,4 mg naloxonu i.v.). V případě dlouhodobé respirační nedostatečnosti je třeba dávku opakovat 1 až 3krát ve tříminutových intervalech, dokud se rychlost dýchání nevrátí do normálu a pacient neodpoví na bolestivý podnět. Je nutné pečlivé sledování pacienta (minimálně po dobu 24 hodin), protože doba účinku antagonisty opioidů je kratší než působení opioidů samotných, takže lze očekávat recidivu respirační nedostatečnosti.

' } ] }, { text: '

Transportován za monitorování vitálních funkcí a předán na jednotku intenzivní metabolické péče jako intoxikace neznámou látkou s podezřením na opioidy, pravděpodobné hypoxické postižení mozku. Byla provedena urgentní endotracheální intubace (ETI) adekvátním způsobem, bez nutnosti farmakologické přípravy, v přímé laryngoskopii bez známek aspirace. Převeden na umělou plicní ventilaci, záhy pozorováno zrůžovění. Počínající lehká interference s ventilátorem, avšak oslovitelný nadále není, mióza zornic přetrvává. Zajištěn periferní žilní vstup, podán Manitol 20% 100 ml, Oikamid 2 amp., Celaskon 3 amp., fyziologický roztok 250 ml. Dále podán naloxon i.v. 1 ml (4 mg), opakovaně cca ā 7 min. Následuje rychlá obnova fyziologické akce srdeční, TK 130/80. Další průběh a laboratorní vyšetření potvrzuje podezření - silná intoxikace heroinem, navíc budivými aminy, pacient je během 72 hodin extubován a předán na standardní lůžko.

', qs: [ { task: '

Zjistěte, co obsahují uvedené léčivé přípravky za účinné látky, jaký je jejich hlavní mechanismus účinku.

', answer: '

Manitol 20% – mannitolosmotické diuretikum, které v ledvinných tubulech oslabuje reabsorpci vody z glomerulárního filtrátu a vede k diuréze a zvýšení exkrece renálně vylučovaných látek.

Oikamid – piracetamnootropikum, které má větší spektrum indikací. V tomto případě použit kvůli intoxikaci pro obnovení kognitivní funkce mozku.

Celaskon – vitamín C (kyselina askorbová), použit pro okyselení moče a tím pádem zvýšení vylučování opioidů jakožto slabých bází močí. Při kyselém pH je slabá báze v ionizované formě a tím pádem není zpětně reabsorbována.

' }, { task: '

Mohlo by pacientovi (pokud by byl při vědomí) pomoci sublingvální podání buprenorfinu?

', answer: '

Buprenorfin je semisyntetický opioid, který patří mezi parciální opioidní agonisty. Absorpce z GIT je špatná a navíc vykazuje vysoký first-pass efekt. Z toho důvodu je podáván buď transdermálně nebo sublingválně. Má vysokou afinitu k μ receptorům, ale jeho aktivita je v porovnání s plnými agonisty nižší. Současně je antagonistou na κ receptorech. Je indikován k léčbě silné chronické bolesti a substituční léčbě závislosti na opioidech. Princip substituční léčby spočívá v tom, že se z vazby na μ receptory uvolňuje pomaleji a u závislého pacienta po relativně dlouhou dobu minimalizuje potřebu další dávky drogy. Zároveň při současném podání s plnými opioidy buprenorfin působí jako kompetitivní antagonista tím, že blokuje μ receptory pro jejich vazbu. Z toho důvodu by sublingválně podaný buprenorfin mohl vytěsnit pacientem přijatý opioid z vazby na μ receptory a zmírnit příznaky předávkování.

' } ] } ], biblio: ["

Welsh, Christopher; Sherman, Susan G.; Tobin, Karin E. A case of heroin overdose reversed by sublingually administered buprenorphine/naloxone (Suboxone®). Addiction. 2008;103(7): 1226-1228.

http://onlinelibrary.wiley.com/doi/10.1111/j.1360-0443.2008.02244.x/pdf

"], editors: '07', index: 49 }, { title: 'A4cz', cathegory: 0, keywords: ["metoklopramid", "ranitidin", "akatizie", "perindopril", "indapamid", "vildagliptin", "metformin", "atorvastatin", "bisulepin"], parts: [ { text: '

Ambulantní pacient, muž ve věku 32 let se dostavil na chirurgii na plánovanou endoskopickou operaci. Anamnéza: diabetes mellitus 2. typu, chronická sinusitida, hypertenze dyslipidémie, občasný gastroezofageální reflux, obezita. Spočítané aktuální BMI = 34. Pacient udává alergii na penicilin.

Farmakologická anamnéza:

Prestarium neo combi 5 mg/1.25 mg (1-0-0)

Eucreas 50 mg/850 mg (1-0-1)

Torvacard 20 mg (0-0-1)

', qs: [ { task: '

Co je účinnou látkou zmíněných přípravků?

', answer: '

Prestarium neo combi: perindopril arginin + indapamid

Eucreas: vildagliptin 50 mg, metformin 850 mg

Torvacard: atorvastatin

' }, { task: '

Jaký je mechanismus působení účinných látek zmíněných přípravků?

', answer: '

perindopril arginin - inhibitor angiotensin-konvertujícího enzymu (inhibitor ACE), který konvertuje angiotensin I na vazokonstriktor angiotensin II; konvertující enzym současně stimuluje sekreci aldosteronu kůrou nadledvin, a stimuluje degradaci bradykininu, vazodilatátoru, na inaktivní heptapeptidy.

indapamidthiazidové diuretikum. Inhibuje reabsorpci sodíku v kortikálním dilučním segmentu. Zvyšuje vylučování sodíku a chloridů močí a v menší míře i vylučování draslíku a hořčíku, čímž zvyšuje objem moči a má antihypertenzní účinek.

vildagliptin – primárně působí inhibičně na enzym DPP-4, který je zodpovědný za degradaci inkretinových hormonů GLP-1 (glukanon-like-peptid-1) a GIP (glukózo-dependentní inzulinotropní polypeptid). Podání vildagliptinu vede k rychlé a kompletní inhibici účinku DPP-4 s následným zvýšením endogenních hladin inkretinových hormonů GLP-1 a GIP, nalačno i po příjmu potravy. Zvýšením endogenních hladin inkretinových hormonů zvyšuje vildagliptin citlivost β buněk ke glukóze s výsledným zlepšením sekrece glukózo-dependentního inzulinu.

metforminantidiabetikum ze skupiny biguanidů, snižuje jak bazální tak i postprandiálníglykemii. Nestimuluje sekreci inzulinu, a proto nevyvolává hypoglykemii, ani nezvyšuje hmotnost. Metformin může působit na snížení glykemie třemi mechanismy:

1. snížením tvorby glukózy v játrech inhibicí glukoneogeneze a glykogenolýzy;

2. ve svalech, mírným zvýšením citlivosti na inzulin, zlepšením vychytávání a utilizace glukózy v periferních tkáních;3. zpomalením absorpce glukózy ze střeva.

atorvastatin- selektivní a kompetitivní inhibitor HMG-CoA reduktázy. Tento enzym

katalyzuje přeměnu z 3-hydroxy-3-methyl-glutaryl-koenzymu A na mevalonát, což je prekurzor sterolů včetně cholesterolu.

' }, { task: '

Jaké jsou nežádoucí účinky přípravku Eucreas 50/850 mg?

', answer: '

Laktátová acidóza,nauzea, poškození až selhání ledvin, hepatotoxicita, hypoglykémie.

' } ] }, { text: '

Pacient v posledních 7 letech prodělal 3 menší operace v celkové anestezii bez jakýchkoliv komplikací nebo nežádoucích příhod. Předoperační laboratorní vyšetření včetně krevního obrazu neodhalilo žádné abnormality s výjimkou hyperglykemie (6,5 mmol/L). Na základě pacientova BMI a příležitostného GE- refluxu bylo rozhodnuto, že pacient pro prevenci plicní aspirace dostane profylakticky 1cps magistraliter připraveného metoklopramidu a 1 tbl 75 mg Ranisanu.

', qs: [ { task: '

Co je účinnou látkou přípravku Ranisan? Do kterých farmakologických skupin obě látky patří?

', answer: '

Ranisan – ranitidin – H2 anihistaminikum, antiulcerózum, terapie pyrózy.

Metoklopramid – prokinetikum.

' }, { task: '

Jaký je jejich mechanismus účinku?

', answer: '

metoklopramid - dopaminergní antagonista, blokádou dopamiergních receptorů zvyšuje práh chemorecepční spouštěcí zóny a snižuje reakci přilehlého centra pro zvracení. Dále snižuje citlivost viscerálních aferentních nervů na reflexním oblouku zvracení. V horní části gastrointestinálního traktu podporuje normální vyprazdňování žaludku, má prokinetický účinek.

ranitidinH2 antihistaminikum, blokuje H2 receptory, snižuje sekreci HCl do žaludku.

' } ] }, { text: '

Přibližně hodinu po podání metoklopramidu a ranitidinu začal být pacient úzkostný, neklidný, agitovaný, přecházel z místa na místo, měl pocit horka. Obj. – motorický neklid, pacient se ve stoje kolíbal ze strany na stranu, ve stoje nakračoval nohou, jakoby neustále chodil, vsedě neustále hýbal nohama (křížil, dával přes sebe a zase opětovně vedle sebe). Pacient se také začal zvýšeně potit. Nosní kanylou bylo započato podání kyslíku (2 L/min). Krevní tlak a srdeční frekvence se zvýšily (sinusový rytmus 105/min a TK 159/93).

', qs: [ { task: '

Co může být příčinou této nežádoucí příhody?

', answer: '

Extrapyramidová symptomatologie po podání metoklopramidu.

' }, { task: '

Došlo k této nežádoucí příhodě v důsledku kombinace léčiv?

', answer: '

Ne. Nejčastěji k extrapyramidové symptomatologii dochází po vysokých dávkách antipsychotik 1. generace („typických“ antipsychoticích), popř. po jejich dlouhodobém podání.

' }, { task: '

Jaký druhy léčiv mohou nejčastěji působit podobné problémy a proč?

', answer: '

Antidopaminergní léčiva – antagonisté a parciální agonisté dopaminergních receptorů. Typicky antipsychotika 1. generace. (typická antipsychotika).

Proč? V důsledku deplece dopaminu na nigrostriatální dopaminergní dráze. Tato dráha hraje hlavní roli v kontrole a vedení pohybů.

' }, { task: '

Jaká je léčba akathizie a dyskinezí po těchto látkách?

', answer: '

Jaká je léčba akatizie a dyskinezí po těchto léčivech?

  1. Vysadit antidopaminergní medikaci.
  2. H1 antihistaminika.
  3. Benzodiazepiny.
  4. Symptomatická léčba.
' } ] }, { text: '

Po rozhovoru a vyšetření pacienta ordinoval anesteziolog Dithiaden inj. (25 mg intramuskulárně). Přibližně 20 minut po podání Dithiadenu se symptomy výrazně zmírnily. Pacient tvrdil, že se cítí mnohem lépe. Podávání kyslíku bylo přerušeno, pacientův krevní tlak klesl na 146/67 , pulz 81/min. Po dohodě s pacientem byla operace odložena. Pacient byl informován o akatizii po metoklopramidu a instruován, aby se v budoucnu vyvaroval užití metoklopramidu.

', qs: [ { task: '

Jaké léčivo obsahuje přípravek Dithiaden a jaký je jeho mechanismus účinku a indikace?

', answer: '

Bisulepin - intenzivní a poměrně selektivní H1 antihistaminikum. Působí sedativně, nemá antiadrenergní a jen velmi nízké anticholinergní a antiserotoninové účinky.

Indikace: Projevy hypersenzitivity I. typu (zprostředkované IgE protilátkami); akutní alergické stavy, alergická rýma (zvláště sezónní), astma bronchiale I. typu; alergické reakce po bodnutí hmyzem, aplikaci alergenu při hyposenzibilizaci, po podání léků či požití potravin; kopřivka, Quinckeho edém, atopická dermatitida.

' } ] } ], biblio: ["

Moos DD, Hansen DJ. Metoclopramide and extrapyramidal symptoms: a case report.

J Perianesth Nurs. 2008;23(5):292-299.

http://www.ncbi.nlm.nih.gov/pubmed/18926476

"], editors: '01', index: 50 }, { title: 'A5cz', cathegory: 0, keywords: ["organofosfáty", "intoxikace", "atropin", "pralidoxim", "dichlorvos", "olanzapin"], parts: [ { text: '

Pacient 18 let, muž, přivezen na pohotovost po pokusu o sebevraždu - požití 100 ml insekticidního přípravku Fenda 1000.

Pacient se jeví jako otupělý, skleslý a netečný, přesto je při příjmu při vědomí a orientovaný.

Při příjmu: TK110/70 mm Hg, TF 110/min, funkce jater a ledvin bez patologie, elektrolyty v normě. V osobní anamnéze těžké depresivní příhody.

', qs: [ { task: '

Jaká je účinná látka přípravku Fenda?

', answer: '

dichlorvos

' }, { task: '

Kolik účinné látky pacient požil?

', answer: '

V 1 l přípravku je 1000 g dichlorvosu, pacient požil 100 ml přípravku, tudíž 100 g dichlorvosu.

' }, { task: '

Jaké byste navrhli farmakoterapeutické a další opatření prospěšné pro pacienta?

', answer: '

Výplach žaludku, podání aktivního uhlí, atropinu a reaktivátorů acetylcholinesterázy (ACHE). Pokud nejsou k dispozici reaktivátory ACHE, podávají se velké dávky reverzibilních inhibitorů acetylcholinesterázy s cílem ochránit ještě funkční ACHE před navázáním ireverzibilního antagonisty.

' }, { task: '

Jaké symptomy budou pravděpodobně intoxikaci provázet?

', answer: '

Projevy intoxikace vyplývají z inhibice ACHE. Objevuje se mióza, poruchy vidění, bolesti hlavy, nevolnost, zvracení, křeče v GIT, zvýšená sekrece žláz (pocení, slinění), bradykardie, pokles krevního tlaku, svalové záškuby, křeče až neuromuskulární blokáda.

' } ] }, { text: '

Pacient prodělává standardní toxikologickou terapii: výplach žaludku, podávání aktivního uhlí, projímadel. Poté pacient ponechán na pozorování, při kterém dochází ke zvýšení sekrece slinných a potních žláz. Vyšetření zornic zjišťuje bilaterální miózu, proto je pacientovi aplikován atropin a pralidoxim i.v. infuzi. Vyšetření plazmatické cholinesterázy (2,7 U/ml) potvrzuje intoxikaci organofosfáty. Celkově byla pacientovi v průběhu dvou dní aplikována dávka 1600 mg pralidoximu a 32 mg atropinu v i.v. infuzi. V průběhu terapie je pacient agitovaný, zmatený, občas nadává, nesouvisle mumlá, snaží se o útěk. Dále zjištěna hyperpyrexie (38°C) a tachykardie (120/min). Další psychiatrické vyšetření popisuje pacienta jako dezorientovaného, má problémy s pozorností, dlouhá reakční doba a přítomné bludy pronásledování. Dg. delirium.

', qs: [ { task: '

Jaký je mechanismus účinku pralidoximu?

', answer: '

Pralidoxim je reaktivátor acetylcholinesterázy, patří do skupiny tzv. oximů. Mechanismus účinku spočívá v tom, že k oximu má organofosfát vyšší afinitu než k ACHE a tím pádem je schopen acetylcholinesterázu uvolnit z vazby na organofosfát. Oximy musí být podány co nejdříve po intoxikaci, protože jakmile komplex ACHE-organofosfát "zestárne", nejsou oximy schopné enzym reaktivovat.

' }, { task: '

Co způsobilo pacientovi současné symptomy?

', answer: '

Předávkování atropinem.

' } ] }, { text: '

Pacientovi byl vysazen atropin a pralidoxim a předepsána Zyprexa Velotab 10 mg (1-0-0). Do 24 h delirium ustupuje. Z nemocnice propuštěn 7. den po hospitalizaci bez neurologických následků.

', qs: [ { task: '

Jaká je účinná látka přípravku Zyprexa? Zařaďte tuto látku do ATC skupiny, vyjmenujte další látky z této skupiny a jejich indikace.

', answer: '

Účinnou látkou v přípravku Zyprexa je olanzapin, který řadíme mezi atypická antipsychotika, do skupiny multireceptorových antagonistů (MARTA), ATC skupina N05AH03. Další látky z této skupiny jsou quetiapin a klozapin. Jejich hlavní indikací je schizofrenie a bipolární porucha.

' } ] } ], biblio: ["

Orhan, F. Ozlem, et al. Central anticholinergic syndrome associated with atropine: case report. AJCI. 2008; 2(1)):45-46.

http://www.ajcionline.org/index.php/ajci/article/viewFile/91/72

"], editors: '07', index: 51 }, { title: 'A6cz', cathegory: 0, keywords: ["N-acetylcystein", "aktivní uhlí", "penicilin G", "cimetidin", "silymarin", "amatoxin"], parts: [ { text: '

Žena (72 let) a její syn (45 let) byli přijati na pohotovost se zvracením, průjmem a bolestí břicha po konzumaci pokrmu z hub. Houby nasbírala žena blízko svého současného bydliště, připomínaly jí vzhledem houby, které běžně sbírali a konzumovali ve své zemi původu (na Ukrajině). Anamnéza obou pacientů zahrnuje pouze léčenou hypertenzi. Zvracení, průjem a bolest břicha v pravém horním kvadrantu se u pacientů objevily po 16 hod od konzumace hub u ženy, u muže po 14 hod.

', qs: [ { task: '

Jaké další příznaky a výsledky vyšetření byste očekávali u intoxikovaných?

', answer: '

Otrava houbami může mít různé projevy s různou intenzitou a latencí nástupu příznaků. Z popisu houby pacientkou a z latence příznaků 12-16 hod od požití bylo možné usuzovat na pravděpodobnou otravu amatoxiny (toxiny r. Amanita). Ta se projevuje jako silná nauzea, zvracení, průjem a dlouhotrvající bolesti břicha nastupující nejdříve 6 hod, obvykle ale 12 hod nebo až 24 hod po požití houby. Typický bývá krátkodobý mírný ústup potíží a jejich návrat s větší intenzitou. Očekávat můžeme dehydrataci, hypoglykémii, synkopy, nefrotoxicitu a především hepatotoxicitu. V pokročilejších stádiích otravy je možné očekávat metabolický rozvrat, poruchu krevní srážlivosti, poruchy vědomí, křeče. Neléčená otrava zpravidla končí smrtí do 5‒7 dnů. Z laboratorních výsledků očekáváme patologické hodnoty jaterních enzymů a bilirubinu, poruchy elektrolytové rovnováhy, snížení krevní srážlivosti (nárůst INR), později příznaky metabolického rozvratu (pH, dusíkaté látky apod.)

' } ] }, { text: '

Žena měla tělesnou teplotu 36,9 °C, puls 83/min, krevní tlak 163/97 mm Hg, 18 dechů/min a saturaci kyslíkem 99 %. Muž měl 36,6 °C, puls 120/min, tlak 155/97 mm Hg, 18 dechů/min a saturaci 97 %. Krevní obraz a základní metabolické parametry byly v normálu. Z laboratorních vyšetření bylo patologické stanovení jaterních enzymů (Tab. 1, sloupec „pohotovost“), INR bylo v normě (žena 1,2 a muž 1,0).

Tab. 1: Průběh výsledků vyšetření jaterních enzymů u pacientů

[IU/l]

Pohotovost

Transplantační centrum

Pík

Při propuštění

Kontrola po 5 dnech

Žena

AST

152

280

9640

66

-

ALT

155

279

9360

799

-

Muž

AST

52

71

2868

39

27

ALT

78

107

4215

417

165

', qs: [ { task: '

Jaká jsou fyziologická rozmezí hodnot AST, ALT a INR?

', answer: '

AST 15‒35 IU/l, ALT 10‒35 IU/l, INR 0,8‒1,2.

' }, { task: '

Jaký je správný léčebný postup při podezření na otravu houbami?

', answer: '

V domácím prostředí vyvolání zvracení, přivolání pomoci, podání aktivního uhlí a uchování syrových hub (ev. pokrmu, zvratků) k identifikaci. V nemocnici výplach žaludku, podání vysokých dávek aktivního uhlí, oběhová a dechová stabilizace ev. podpora, častá kontrola jaterních a renálních funkcí. Dále také kontrola glykémie, elektrolytové rovnováhy, ev. podání antiemetik, spasmolytik, analgetik, sedativ. Transplantace jater je poslední možností záchrany. Pro otravu amatoxiny není k dispozici specifické antidotum, účinná není ani hemodialýza nebo forsírovaná diuréza. Pro otravu jinými typy houbových toxinů mohou být k dispozici antidota – např. toxin muskarin (parasympatomimetikum) a antidotum atropin (parasympatolytikum).

' } ] }, { text: '

Před převozem do zařízení připraveného na transplantaci jater dostali oba pacienti aktivní uhlí (50 g p.o.) a N-acetylcystein (150 mg/kg i.v.) Po předání na jednotku intenzivní péče transplantačního centra byla jejich léčba následující: druhá dávka aktivního uhlí (pouze muž, žena dávku vyzvracela a byl jí podán ondansetron), N-acetylcystein (50 mg/kg i.v. infuzí 4 hod, 100 mg/kg i.v. infuzí dalších 16 hod.), penicilin G (500 000 jednotek/kg/den i.m.), cimetidin (33 mg/kg i.v. každých 6 hod) a extrakt z ostropestřce (žena 1000 mg 3× denně p.o., muž 1250 mg 3× denně p.o.)

', qs: [ { task: '

Z jakého důvodu se podává aktivní uhlí, N-acetylcystein, penicilin G a cimetidin?

', answer: '

Aktivní uhlí vychytá toxin přítomný v GIT. Vzhledem k typické latenci příznaků je důležitá především blokáda enterohepatálního oběhu amatoxinů. N-acetylcystein se podává ke zmírnění oxidativního poškození hepatocytů, které bylo při intoxikaci prokázáno. N‑acetylcystein vychytává volné kyslíkové radikály, které poškozují biomakromolekuly v jaterních buňkách. Penicilin G pravděpodobně inhibuje průnik amatoxinů do hepatocytů. Cimetidin inhibuje jaterní CYP2E1, 3A4 a 2D6 a bylo prokázáno, že snižuje nekrózu v zasažených játrech.

' }, { task: '

Jaké účinné látky obsahuje ostropestřec mariánský? Jsou v ČR registrovány přípravky s extraktem z této léčivé rostliny? Pokud ano, jaké mají indikace?

', answer: '

Ostropestřec mariánský obsahuje komplex flavonolignanů označovaný jako silymarin. Jeho složkami jsou např. silybiny, silychristin, silydianiny a další látky. Působí hepatoprotektivně, stimuluje růst hepatocytů a jejich proteosyntézu, inhibuje oxidativní reakce a reguluje zánětlivé procesy v jaterní tkáni. V ČR jsou registrovány tyto HVLP: Flavobion, Lagosa, Legalon, Silymarin AL a Simepar k léčbě chronických zánětlivých jaterních onemocnění, jaterní cirhózy a steatózy a toxického poškození jater. Extrakt je dále přítomen také v perorálních kapkách Iberogast určených k léčbě poruch funkce a motility žaludku a střev.

' } ] }, { text: '

Houby byly mykologem identifikovány jako Amanita ocreata (viz obrázek). Tato muchomůrka je typická pro severní Ameriku. U nás je její blízkou příbuznou muchomůrka zelená (Amanita phalloides).

', qs: [ { task: '

S jakými jedlými houbami bývá muchomůrka zelená zaměňována? Jaké toxiny obsahuje a který je nejnebezpečnější?

', answer: '

Muchomůrka zelená je nejčastěji zaměňována za mladé bedly vysoké, dále za holubinku trávozelenou, h. nazelenalou, h. bukovku, čirůvku zelánku a některé jedlé druhy žampionů (pečárek). M. zelená obsahuje dva typy toxinů – falotoxiny (např. faloidin) a amatoxiny (např. amanitiny). Nejtoxičtějším je α-amanitin, který inhibuje RNA-polymerázu II v hepatocytech, inhibuje tak proteosyntézu a způsobuje nekrózu hepatocytů.

' } ] } ], biblio: ["

Ward, Jeanine, et al. Amatoxin poisoning: case reports and review of current therapies. The Journal of emergency medicine. 2013;44(1):116-121.

http://www.sciencedirect.com/science/article/pii/S0736467912002387

"], editors: '06', index: 54 }, { title: 'A7cz', cathegory: 0, keywords: ["alergie", "anafylaxe", "metylprednizolon", "salbutamol", "klemastin", "beklometazon", "adrenalin"], parts: [ { text: '

17 letá dívka byla sledována na alergologickém pracovišti kvůli potravinové alergii, sezónní alergické rýmě a atopickému průduškovému astmatu. Sérologicky vykazovala pozitivitu specifických IgE protilátek proti roztočům, pylu trav a z potravinových alergenů proti máku, česneku a kmínu. Ve věku pěti let byla 2x hospitalizována pro středně těžkou exacerbaci astmatu. V posledních 3 letech měla symptomy perzistujícího astmatu, které ji omezovaly při sportovních aktivitách (inhalační kortikoidy však neužívala). Při sportovní činnosti (zejména běhu) a při akutní respirační infekci se 3x týdně objevovaly příznaky astmatu (kašel, pískot, dušnost). Během pylové sezóny dívka užívala nepravidelně kvůli alergické rýmě cetirizin (denní dávka 10 mg, Zyrtec tbl.). V posledních 3 letech si stěžovala na projevy potravinové anafylaxe (otoky rtů, svědění patra, nevolnost a silné bolesti břicha po požití máku, kmínu, sezamu a lískových ořechů). Po bonbónech s mátou se vyskytnula dušnost. Dívka byla poučena alergologem, že musí z jídelníčku vyloučit alergizující potraviny. Autoinjektor s adrenalinem k dispozici neměla.

Rodinná anamnéza: otec dívky a její babička z otcovy strany se léčí inhalačními kortikosteroidy kvůli atopickému průduškovému astmatu a matka dívky prodělala epizody anafylaxe, které vyvolal Acylpyrin a penicilinová antibiotika.

', qs: [ { task: '

Co je to cetirizin a jaký je mechanismus jeho účinku?

', answer: '

cetirizin – H1 antihistaminikum, látka antagonizující účinek histaminu, antihistaminikum druhé generace, selektivní antagonista periferních H1 receptorů.

' }, { task: '

Jaký je hlavní rozdíl mezi antihistaminiky 1. a 2. generace?

', answer: '

U antihistamink 2. generace je mnohem menší pravděpodobnost jejich průniku hematoencefalickou bariérou, váží se specifičtěji na H1 receptory: nemají sedativní účinek.

' } ] }, { text: '

Dívka si ve školní jídelně (začátkem dubna, před začátkem pylové sezóny trav, na které byla alergická) objednala pokrm, ve kterém byla jako příloha rýže ochucená kari kořením. V té době neužívala žádné léčivé přípravky. Nikdy předtím toto směsné koření nepožila a netušila, že jeho součástí je kmín (který u ní v minulosti anafylaxi způsobil). Ihned po požití prvního sousta začala dívka cítit pálení v ústní dutině (přestože sousto ihned vyplivla na talíř. Mobilním telefonem okamžitě informovala o svých potížích dědu a ten zavolal rychlou záchrannou službu s lékařem. Dívka šla z jídelny naproti lékaři a před jídelnou na chodníku upadala do bezvědomí. Lékař dorazil na místo 5 minut po ohlášení anafylaxe dispečerce. Ihned zahájil oxygenoterapii, zajistil vstup do žíly a intravenózně podal 60 mg metylprednizolonu (Solu-Medrol inj.), Calcium chloratum 10 ml a rychlou infuzi 250 ml fyziologického roztoku (během 10 minut).

', qs: [ { task: '

Z jakého důvodu byly aplikovány glukokortikoidy v akutní fázi anafylaxe?

', answer: '

Význam glukokortikoidů podaných během akutní fáze anafylaktické reakce spočívá v tom, že zabrání rozvoji pozdní fáze, která se může objevit v intervalu 6–8 hod. Riziko pozdní fáze

anafylaxe je také důvod k hospitalizaci nemocného po dobu 24 hod. (i tehdy, jestliže se podaří akutní fázi dobře zvládnout).

' } ] }, { text: '

Při hospitalizaci byla dívka dušná a dezorientovaná. Dech byl zrychlený (25/min.) s distančními pískoty, srdeční frekvence zrychlená (96/min.), krevní tlak při spodní hranici normálu (100/60 mm Hg). Podáním inhalačního salbutamolu byla okamžitě zahájena bronchodilatační léčba (Ventolin dos. aer. 4 x 4 dávky, tj. 4 х 400 mikrogramů po 20 minutách během první hodiny) přes inhalační nástavec Volumatic. Nitrožilně byl podán klemastin (Tavegyl inj. i.v.) v dávce 2 mg, metylprednizolon (Solu-Medrol inj. i.v.) celkem 500 mg a Calcium chloratum 10 ml. Po přijetí byla aplikována infúze 500 ml fyziologického roztoku rychlosti 250 ml/hod. Po 2 hod. příznaky anafylaxe vymizely. Přestože byla dívka dušná, nebylo provedeno i.m. podání adrenalinu.

', qs: [ { task: '

Co je to klemastin a jaký je jeho mechanismus účinku?

', answer: '

klemastin - látka antagonizující účinek histaminu, antihistaminikum první generace, antagonista H1 receptorů.

' }, { task: '

Co je to salbutamol a jaký je jeho mechanismus účinku?

', answer: '

salbutamol - β2-adrenergní sympatomimetikum s vysoce selektivním účinkem na receptory v bronchiální svalovině. V terapeutických dávkách nepůsobí vůbec nebo jen málo na β1- adrenoreceptory. Salbutamol je také vysoce účinný v prevenci uvolňování histaminu, pomalu reagující substance (slow-reacting substance of anafylaxis, SRS-A) a dalších látek indukovaných antigenem v plicích citlivého jedince.

' } ] }, { text: '

Vyšetření za 2 dny: plicní funkce normální, bronchodilatační test negativní. Na základě anamnézy svědčící pro perzistující průduškové astma byla zahájena preventivní terapie beklometazonem (Ecobec Easi-Breathe dos. aer.) v dávce 2 x 250 mikrogramů. Dívka byla poučena o inhalační technice s dechem aktivovaného inhalátoru a o používání autoinjektoru s adrenalinem (Epipen inj. 0,3 mg). Dívce byl poskytnut písemný plán léčby anafylaxe a astmatu (včetně salbutamolu a prednizonu) a dostala pohotovostní balíček. Byla rovněž poučena o nutnosti důsledného vyhýbání se potravinám, které již dříve způsobily spuštění anafylaxe (kmín, mák, sezam, lískové ořechy, máta) a o riziku skrytých potravinových alergenů.

', qs: [ { task: '

Jaké jsou možné nežádoucí účinky inhalačně podávaných glukokortikoidů a jak je možné jim předejít?

', answer: '

Nežádoucí účinky se při tomto způsobu aplikace objevují zřídka. Nejčastěji se jedná o chrapot nebo kandidovou infekci v oblasti úst a hrtanu. Infekcím je možné předejít inhalací před jídlem nebo výplachem úst vodou po aplikaci.

' }, { task: '

Jaké terapeutické chyby se vyskytly během popsaného postupu léčby anafylaxe?

', answer: '

Nebyl použit adrenalin, ačkoliv je u anafylaxe s dušností anebo hypotenzí jednoznačně indikovaná jeho i.m. aplikace. Důvodem byly zřejmě neopodstatněné obavy z nežádoucích účinků, které u dětí a mladých dospělých zahrnují krátkodobé zblednutí, palpitaci, zvracení a třes. Při anafylaxi, která ohrožuje život, neexistuje pro podání adrenalinu kontraindikace (u mladých osob jsou závažná srdeční onemocněná vzácná, opatrnosti je třeba pouze u obstruktivní hypertrofické kardiomyopatie, která se však vyskytuje výjimečně). Při srovnání i.v. a i.m. aplikace má adrenalin podaný i.m. delší účinek a lepší bezpečnostní profil. Rovněž je třeba změnit postoj k používání kalcia, které již nemá při terapii akutních alergických stavů místo.

' } ] } ], biblio: ["

Ettlerová K. Zásady léčby anafylaktické reakce. Dermatologie pro praxi. 2010;4:101-105.

http://www.dermatologiepropraxi.cz/pdfs/der/2010/02/09.pdf

", "

Chládková J. Anafylaxe po požití kmínu. Klinická farmakologie a farmacie. 2011; 25: 199-200.

http://www.klinickafarmakologie.cz/pdfs/far/2011/04/09.pdf

", "

Potěšil J, Kopřiva1 F, Godava M, Svetlíková M. Kortikorezistentní astma – aktuální klasifikace, definice a molekulární mechanizmy glukokortikoidní rezistence na buněčné úrovni. Klinická farmakologie a farmacie. 2014; 28: 31-35.

http://www.klinickafarmakologie.cz/pdfs/far/2014/01/08.pdf

"], editors: '02', index: 57 }, { title: 'A8cz', cathegory: 0, keywords: ["omeprazol", "vředová choroba", "synkopa", "diseminovaná intravaskulární koagulopatie", "cefotaxim", "heparin"], parts: [ { text: '

Na ambulanci I. interní kliniky byl RZP přivezen 51letý muž. Nemocný, dle parere, ráno nastoupil do práce (řidič z povolání), protože se necítil dobře a měl nevolnost a průjem, po několika hodinách vyhledal lékaře. Tam v čekárně náhle došlo k synkopě s hypotenzí a akrální cyanózou, pro které byl odeslán k přijetí na interní kliniku. Po příjezdu na ambulanci byl již velice špatně vyšetřitelný pro celkový neklid, agitovanost a nesouvislé vyjadřování. Udával křečovité bolesti pod mečíkem a zvracení. Z anamnézy se podařilo zjistit, že se léčí s vředovou chorobou a užívá omeprazol. Další onemocnění pacient negoval. V objektivním nálezu dominovala akrální cyanóza, palpační citlivost pod mečíkem. TK 100/60, kompenzatorní tachykardie 110/min., ostatní fyzikální nález bez pozoruhodností. Byla vyslovena domněnka, že se může jednat o perforaci vředu s krvácením do dutiny břišní a pacient byl odeslán na chirurgii. Po dvou hodinách byl přivezen zpět na internu, po vyloučení náhlé příhody břišní. V tuto dobu byl nemocný stále hypotenzní, došlo k progresi psychomotorického neklidu, nebyl schopen komunikace, měl febrilie.

', qs: [ { task: '

Co je to omeprazol a jaký je mechanismus jeho účinku?

', answer: '

omeprazol - specifický inhibitor protonové pumpy v parietálních buňkách. Omeprazol je slabá báze a v silně kyselém prostředí intracelulárních kanálků parietálních buněk je koncentrován a konvergován na aktivní formu, která tam inhibuje enzym H+/K+-ATPázu (protonovou pumpu). Tento účinek na konečný krok procesu tvorby žaludeční kyseliny je závislý na dávce a poskytuje vysoce účinnou inhibici jak bazální, tak stimulované sekrece kyseliny, bez ohledu na podnět.

' }, { task: '

Jaké existují další možnosti terapie vředové choroby?

', answer: '

Použití antagonistů H2 receptorů (ranitidin, cimetidin, famotidin). Snižují sekreci kyseliny chlorovodíkové parietálními buňkami žaludeční sliznice. Dále je možné použít antacida – léčiva snižující kyselost žaludečního obsahu. Jsou to bazické látky (podávané perorálně), které vážou protony, např.: NaHCO3, CaCO3, Al(OH)3, Mg(OH)2.

' } ] }, { text: '

V průběhu hospitalizace se stav pacienta neodvratitelně zhoršoval. Provedená základní laboratorní vyšetření ukazovala na poškození parenchymatózních orgánů. Klinický obraz, několikahodinová anamnéza febrilií až 40°C, bez nálezu leukocytózy, nás vedly ke stanovení pracovní diagnózy virového onemocnění. Sugestivní nález v krevním obraze a především koagulační vyšetření nás vedly k podezření na DIC, kterou hematolog potvrdil. Dynamiku laboratoří nebylo možno vzhledem k fulminantnímu průběhu sledovat. Celkový stav si vyžádal spolupráci s oddělením ARO a převedení na úplnou plicní ventilaci. Přes maximální terapii vasopresory, cefotaximem, nízkomolekulárním i klasickým heparinem, se však nepodařilo udržet hemodynamicky stabilní oběh a nemocný po 8 hodinách zemřel na multiorgánové selhání se zhroucením oběhu a zástavu srdeční. Resuscitace byla bez efektu. Vzhledem k bleskovému průběhu choroby, DIC a multiorgánovému selhání, jsme v diferenciální diagnostice pomýšleli i na možnost intoxikace, a proto byla provedena pitva na Ústavu soudního lékařství.

', qs: [ { task: '

Uveďte nejběžnější vasopresory a mechanismy jejich účinku.

', answer: '

Adrenalin: agonista α1, α2, β1, β2 receptorů, vazopresin: agonista vaskulárních vazopresinových receptorů subtypu V1.

' }, { task: '

Co je to cefotaxim a jaký je mechanismus jeho účinku?

', answer: '

Cefotaxim je cefalosporinové antibiotikum 3. generace. Cefalosporiny mají stejný mechanismus účinku jako peniciliny. Snižují aktivitu transpeptidáz při výstavbě bakteriální stěny. V dostatečně vysokých koncentracích proto působí baktericidně.

' }, { task: '

Co je to heparin a jaký je mechanismus jeho účinku?

', answer: '

Heparin je látka ovlivňují krevní srážlivost (antikoagulans). Antikoagulační účinek heparinu je dán především aktivací tělu vlastního glykoproteinu antitrombinu III. Antitrombin se ireverzibilně váže na aktivované faktory krevního srážení, které ve svém centru obsahují kyselinu serin (např. faktor Xa a IIa = trombin). Tímto způsobem antitrombin příslušné faktory inaktivuje. Tato reakce za běžných okolností probíhá relativně pomalu a heparin ji značně urychluje tím, že antitrombin aktivuje.

' }, { task: '

Co je to nízkomolekulární heparin?

', answer: '

Nízkomolekulární (frakcionovaný) heparin se připraví štěpením nativního heparinu. Průměrná molekulová hmotnost jeho přípravků je 4000 až 6000 Da. Nízkomolekulární heparin přednostně inaktivuje některé z heparin senzitivních faktorů krevního srážení (např. Xa). Ve srovnání s klasickým heparinem je bezpečnější, má lepší antikoagulační účinek a méně nežádoucích účinků.

' } ] }, { text: '

Intoxikace prokázána nebyla. Bezprostřední příčinou smrti byl (citace z pitevního protokolu) oboustranný splývající vmezeřený zánět plic v souběhu s vmezeřeným zánětem myokardu. Mikroskopický nález prokázal diagnózu masivního virového postižení plic a myokardu. Pro malou výtěžnost odběru vzorků na virologii post mortem nebyl tento odběr proveden, kultivace na bakteriologii byla negativní. Mezi další nálezy patřily četné subpleurální ekchymózy na obou plících, mnohočetná krvácení do kůry i dřeně nadledvin, což potvrdilo diagnózu DIC, akutní venostáza a těžký edém mozku.

', qs: [ ] } ], biblio: ["

Hufová P., Lukl J., Jančíková K., Kolembus P. Diseminovaná intravaskulární koagulopatie vyvolaná virovou infekcí. Interní medicína pro praxi. 2001;3:133-135.

http://www.internimedicina.cz/pdfs/int/2001/03/09.pdf

"], editors: '02', index: 58 }, { title: 'A9cz', cathegory: 0, keywords: ["prednizolon", "metylprednizolon", "pulzní terapie glukokortikoidy", "meropenem", "vankomycin", "kaspofungin", "anafylaktická reakce", "klemastin", "linezolid", "ceftriaxon", "oxacilin", "metronidazol", "metamizol", "fenpiverin", "pitofenon"], parts: [ { text: '​

23 letý muž náhle onemocněl, jeho tělesná teplota vystoupila na 39°C, objevila se bolest v krku a bolest svalů. Vyšetření neodhalilo žádné abnormální nálezy na vnitřních orgánech; pouze abscesy v pěti zubních kořenech. Diagnóza byla nejasná, léčba s přípravky Samixon, Prostaphlin, Efloran a Meropenem nebyla úspěšná. Kvůli podezření na Stillovu chorobu bylo zahájeno perorální podávání prednizolonu v dávce 60 mg/denně a po čtyřech dnech byla přidána intravenózní pulzní terapie metylprednizolonem (v dávce 1,0 g/denně, 3,0 g celkem).

', qs: [ { task: '

Jaké účinné látky obsahují použité přípravky? Popište jejich mechanismy účinku a zařaďte je do farmakoterapeutických skupin.

', answer: '

Samixon - ceftriaxon je antibiotikum používané k léčbě mnoha bakteriálních infekcí. Je to cefalosporin třetí generace. Má širokospektrou aktivitu proti G+ baktériím a ve srovnání s látkami druhé generace rozšířenou aktivitu proti G- baktériím. Inhibuje syntézu bakteriální stěny prostřednictvím vazby na penicilin vázající protein (PBPs). Inhibice PBPs blokuje transpeptidaci v syntéze peptidoglykanu, který je vyžadován pro bakteriální stěnu.

Prostaphlin - oxacilin je isoxazolyl penicilin - β-laktamové antibiotikum rezistentní k peniciláze. Blokuje tvorbu peptidoglykanových řetězců ve stěně bakteriální buňky (vazbou β-laktamového kruhu oxacilinu na transpeptidázu). Transpeptidáza nemůže katalyzovat tvorbu těchto řetězců, což vede k smrti buňky. Používá se k terapii rezistentních stafylokokových infekcí.

Efloran - metronidazol je nitroimidazolové antibiotikum. Váže se na DNA, porušuje její šroubovicovou strukturu a inhibuje syntézu nukleových kyselin, což vede ke smrti bakteriální buňky. Používá se k terapii anaerobních infekcí a smíšených infekcí, k profylaxi při chirurgickém zákroku vyžadujícímu anaerobní krytí, při průjmu a kolitidě způsobené baktérií Clostridium difficile, při infekcích Helicobacter pylori a duodenální vředové chorobě, při bakteriální vaginóze, amébiáze způsobené Entamoeba histolytica a trichomonádových infekcích.

meropenem - karbapenemové antibiotikum. Dobře proniká buněčnou stěnou většin G+ i G- bakterií a dostává se k penicilin vázajícímu proteinu, čímž inhibuje syntézu buněčné stěny. Meropenem je obzvlášť užitečný při terapii infekcí způsobených Escherichia coli, Pseudomonas aeruginosa, Staphylococcus aureus.

prednizolon a metylprednizolon - glukokortikoidy. Význam glukokortikoidů podaných během akutní fáze anafylaktické reakce spočívá v tom, že zabrání rozvoji pozdní fáze, která se může objevit v intervalu 6–8 hod. Glukokortikoidy přestupují přes buněčnou membránu, vážou se na specifický cytoplazmatický receptor, modifikují transkripci a syntézu proteinů. Inhibují fosfolipázu A2 v kaskádě kyseliny arachidonové a tím tvorbu prostaglandinů.

' } ] }, { text: '

Pokračovala terapie meropenemem a vankomycinem. Po 2 dnech léčby se u pacienta rozvinulo selhání ledvin a jater s klinickým obrazem encefalopatie. Objevila se nekrotická faringitida, hemoragický syndrom (jemná petechiální vyrážka na obou předloktích a v oblasti hrudníku) a dechová nedostatečnost. Teplota vzrostla na 38,2°C. Rentgenové vyšetření odhalilo perzistující příznaky pleuritidy, perivaskulární edém a perikarditidu. Vzhledem k tomu, že nebylo možné vyloučit látkovou toxicitu ovlivňující více orgánů, byla ukončena léčba meropenemem (1,0 g 3 x denně, 27 g celkem) a vankomycinem (1,0 g 2 x denně, 14 g celkem), podávání steroidů pokračovalo perorálně i intravenózně v dávce 500 mg/denně (celkem 5 dní). Stav se začal zlepšovat: tělesná teplota pacienta a dýchací funkce se navrátily k normálu. Opakované rentgenové vyšetření ukázalo snižující se intenzitu příznaků pleuritidy a perikarditidy. Funkce ledvin se vrátila k normálu po třech dnech, jaterní funkce se normalizovaly za 12 dnů. Po třech dnech se znovu objevila horečka. Centrální žilní katétr zavedený před 23 dny byl odstraněn. Byla odebrána krev pro kultivaci a vzorek ze špičky katétru. RTG vyšetření hrudníku odhalilo středně velkou ohniskovou infiltraci ve středu levé plíce. Na kůži byla pozorována vezikulární vyrážka. Stav byl vyhodnocen jako plísňová sepse a byla zahájena terapie kaspofunginem. Po 2 dnech (po vyhodnocení bakteriální analýzy krevního vzorku) byla detekována Candida albicans. Kultivace vzorku ze špičky katétru byla pozitivní na Enterococcus faecalis a MRSA citlivý na vankomycin. Bylo rozhodnuto vankomycin podat.

', qs: [ { task: '

Jaký je mechanismus účinku kaspofunginu a vankomycinu?

', answer: '

vankomycin - antibiotikum, které inhibuje tvorbu bakteriální buněčné stěny (interference s transmembránovým transportem strukturálních částí buněčné stěny).

kaspofungin - antimykotikum, první antibiotická látka, která inhibuje syntézu buněčné stěny plísní. Blokuje syntézu beta-(1,3)-D-glukanu (základní složky buněčné stěny druhů Aspergillus a Candida).

' }, { task: '

Jaké je terapeutické použití pro kaspofungin a vankomycin?

', answer: '

vankomycin: po perorální aplikaci se nevstřebává - lokální terapie střevních onemocnění, infekce Clostridium difficile projevující se jako pseudomembranózní enterokolitida, parenterální podání - pouze jako východisko z nouze při těžkých stafylokokových a streptokokových infekcích s rezistencí vůči penicilinu a oxacilinu nebo při alergii na penicilin.

kaspofungin: léčba invazivní kandidózy, léčba invazivní aspergilózy u dospělých nebo pediatrických pacientů, kteří jsou refrakterní vůči amfotericinu B, lipidovým lékovým formám amfotericinu B a/nebo itrakonazolu nebo nesnášejí uvedené látky. Refrakterita je definována jako progrese infekce nebo absence zlepšení po minimálně 7 dnech předcházejících terapeutických dávek účinné antimykotické terapie.

' } ] }, { text: '

Vankomycin (1 g) byl rozpuštěn v 250 ml izotonického roztoku. Infúze byla zahájena rychlostí 10 mg/min. Během první minuty aplikace se u pacienta objevil studený pot, třes, zrudla mu tvář, třes se rozšířil na celé tělo, rozvinula se dyspnoe a tachykardie, tlak krve poklesnul na 80/60 mm Hg, byla pozorována kožní vyrážka.

', qs: [ { task: '

K čemu u pacienta pravděpodobně došlo?

', answer: '

K anafylaktické reakci.

' }, { task: '

Jaká léčiva byste použili pro zvládnutí aktuálního stavu?

', answer: '

Při léčbě akutní anafylaxe je lékem první volby adrenalin, přestože v řadě případů nebývá podán (obavy z nežádoucích účinků). Dále: oxygenoterapie, H1 antihistaminika, dodávka tekutin za účelem udržení oběhu (Ringerův roztok), glukokortikoidy. Pokud má nemocný klinické známky bronchospazmu, je potřebné podání β2 sympatomimetik.

' } ] }, { text: '

Stav byl vyhodnocen jako anafylaktická reakce. Infúze byla zastavena, byl podán klemastin, Analgin, intravenózně prednisolon v dávce 60 mg, byla aplikována terapie kyslíkem a krystaloidní roztok v infúzi (Ringerův roztok, izotonický roztok). Po 20 minutách vymizela dyspnoe a tachykardie, krevní tlak a zarudnutí obličeje se stabilizovaly, třes těla však pokračoval po dobu další 1 hodiny, teplota pacienta stoupla na 39,5°C. Hladina C-reaktivního protein (CRP) byla 69,09 mg/l. Opakovaná kultivace krve neukázala žádné baktérie. Ultrazvukové vyšetření srdce neodhalilo žádné příznaky endokarditidy ani perikardiální tekutiny. Pro další terapii stafylokokové a enterokokální sepse byl aplikován linezolid (600 mg 2 x denně, intravenózně, po dobu celkem 11 dní). Pro terapii fungální sepse pokračovalo podávání kaspofunginu (po dobu celkem 12 dní). Nadále byl perorálně aplikován prednizolon v dávce 40 mg/denně. Pacientovi se normalizovala teplota, zmizely příznaky poškození plic a kožní léze. Hladina CRP se vrátila k normálu. Výsledky biopsie lymfatických uzlin neposkytnuly dostatečné údaje podporující přítomnost T lymfomu nebo Kikuchiho syndromu. Diagnóza Stillovy choroby zůstala nezměněná. Dávka prednizolonu byla postupně snižována na 20 mg/denně a byla vysazena po 4 týdnech. Pacient byl ve zlepšeném stavu propuštěn z nemocnice.

', qs: [ { task: '

Jaký je mechanismus účinku linezolidu a klemastinu?

', answer: '

linezolid - syntetické antibiotikum oxazolidinonové skupiny. Váže se na bakteriální ribozomy a zabraňuje tvorbě funkčního 70S-iniciačního komplexu, čímž inhibuje syntézu proteinu bakteriální stěny. Používá se k terapii infekcí způsobených multirezistentními bakteriemi včetně meticilin-rezistentní baktérie Staphylococcus aureus (MRSA).

klemastin - látka antagonizující účinek histaminu, antihistaminikum první generace, antagonista H1 receptorů.

' }, { task: '

Jaké jsou účinné látky v přípravku Analgin? Jaký jsou jejich mechanismy účinku a terapeutické indikace?

', answer: '

Analgin: kombinované spazmoanalgetikum obsahující neopioidní pyrazolonové analgetikum metamizol, jehož analgetický, protizánětlivý a antipyretický účinek je dán inhibicí COX a tím syntézy prostaglandinů. Spazmolytickou složku přípravku tvoří anticholinergní látky (antagonisté muskarinových receptorů) fenpiverin a pitofenon s přímým účinkem na hladkou svalovinu.

Indikace: k léčbě kolikovitých bolestí žlučových a močových cest, tenezmů močového měchýře, bolestivých spazmů žaludku a střev, spastické dysmenorey a k tlumení spastických bolestí při instrumentálním vyšetření a po něm.

' } ] } ], biblio: ["

Petrů V. Anafylaxe. Postgraduální medicína. 2007; 9: 374-381.

http://zdravi.e15.cz/clanek/postgradualni-medicina/anafylaxe-300317

", "

Kupstaite R, Baranauskaite A, Pileckyte M, Sveikata A, Kadusevicius E, Muckiene G. Severe vancomycin-induced anaphylactic reaction. Medicina (Kaunas). 2010; 46:30-33.

http://medicina.lsmuni.lt/med/1001/1001-05e.pdf

"], editors: '02', index: 62 }, { title: 'A10cz', cathegory: 0, keywords: ["kardiogenní šok", "atropin", "inzulin", "adrenalin", "glukonát vápenatý", "levosimendan", "verapamil"], parts: [ { text: '

Pacient (muž, 60 let) byl hospitalizován s podezřením na kardiogenní šok. RZP byla přivolána poté, co pacient užil 6 tablet přípravku Isoptin 80 mg v průběhu 6 hodin. Pacient si tablety vzal, protože u něj byla již dříve diagnostikována intermitentní supraventrikulární tachykardie a Isoptin mu byl předepsán s doporučením užívání „dle potřeby“.

', qs: [ { task: '

Jaká účinná látka je obsažena v přípravku Isoptin a jaký je její mechanismus účinku?

', answer: '

Účinnou látkou přípravku Isoptin je verapamil. Je to blokátor vápníkových kanálů L-typu. Blokuje influx vápníku do buněk myokardu a hladkých svalů cév, a má tak negativní chronotropní a negativní inotropní efekt. Kromě toho ovlivňuje také vylučování inzulinu z β‑buněk pankreatu, které taktéž vyžaduje influx vápníku do buňky. Verapamil se využívá v léčbě ischemické choroby srdeční (stabilní i nestabilní anginy pectoris, vazospastické anginy pectoris a po infarktu myokardu), v léčbě srdečních arytmií (např. paroxysmální supraventrikulární tachykardie) a v léčbě hypertenze.

' }, { task: '

Jaká je maximální denní dávka účinné látky Isoptinu a jaké množství užil pacient?

', answer: '

Při dlouhodobém užívání by dávka neměla překročit 480 mg/den, krátkodobé překročení je ale možné. Pacient užil 480 mg za 6 hod.

' }, { task: '

Jaké symptomy se kvůli intoxikaci budou u pacienta pravděpodobně rozvíjet?

', answer: '

Mezi symptomy intoxikace blokátory vápníkových kanálů patří především bradykardie a hypotenze, hyperglykemie, dále hrozí srdeční zástava, poruchy vědomí a strnulost. Možný je také rozvoj metabolické acidózy.

' }, { task: '

Jaká je standardní terapie intoxikace léčivy z této skupiny?

', answer: '

Jako u všech intoxikací je žádoucí zajistit dekontaminaci pacienta, je-li to možné (výplach žaludku, ev. podání aktivního uhlí). Poté následuje stabilizace a podpora kardiovaskulárních funkcí – pro úpravu bradykardie je možné podat atropin, střední dávky dopaminu nebo adrenalin, hypotenzi zmírňujeme podáním i.v. tekutin a i.v. suplementací vápníku (glukonát nebo chlorid vápenatý), často je třeba přidat ještě adrenalin. Dále se podává také inzulin a glukóza/dextróza, je třeba monitorovat acidobazickou rovnováhu a hladinu základních elektrolytů.

' } ] }, { text: '

RZP po příjezdu zjistila, že pacient je při vědomí, krevní tlak měl 91/51 mm Hg a puls 46/min. Při transportu do nemocnice mu byl aplikován i.v. roztok krystaloidů a 1 mg atropinu, ovšem bez efektu na tepovou frekvenci. Při příjmu na pohotovosti byl krevní tlak pacienta 98/79 mm Hg, puls 41/min, dechová frekvence 20/min a tělesná teplota 36,1 °C. EKG vyšetření odhalilo SA blok a široký QRS komplex. Pacient ihned dostal infuzi krystaloidů, 1 g glukonátu vápenatého a 70 U inzulinu v dextróze, přesto krevní tlak i tepová frekvence klesly na 70/32 mm Hg a 35 tepů/min a pacient upadl do soporu.

', qs: [ { task: '

Jaká další léčiva bychom mohli pacientovi podat?

', answer: '

Vzhledem k tomu, že pacientovi již byly aplikovány krystaloidy, atropin, inzulin, dextróza a glukonát vápenatý bez úpravy stavu, bude třeba upravit bradykardii a hypotenzi podáním např. adrenalinu.

' } ] }, { text: '

Protože se stav pacienta nadále zhoršoval, byl podán i.v. bolus adrenalinu následovaný infuzí. Pacient byl přemístěn na JIP. Bradykardie, hypotenze, změny na EKG a sopor přetrvávaly i přes podání glukonátu vápenatého, inzulinu, dextrózy a adrenalinu. Protože tato standardní terapie selhala, bylo rozhodnuto o experimentální aplikaci přípravku Simdax s úvodní dávkou 24 μg/kg následovanou infuzí s dávkou 0,1 μg/kg/min.

', qs: [ { task: '

Jaká je účinná látka přípravku Simdax a jaký má mechanismus účinku? Zhodnoťte, jak uvedené dávky korespondují s SPC.

', answer: '

Přípravek Simdax obsahuje levosimendan. Tato látka se využívá v krátkodobé terapii akutně dekompenzovaného chronického srdečního selhání a v případech, kdy je vhodné použít inotropní podporu. Levosimendan je vápníkový senzitizér, zvyšuje citlivost aktinu a myozinu k vápenatým kationtům a oddaluje jejich disociaci vazbou na troponin, v souhrnu tedy vykazuje pozitivní inotropní efekt. Působí ale také vazodilatačně přes ATP-senzitivní draslíkové kanály hladkých svalů cév.

Dle SPC by terapie měla být zahájena úvodní dávkou 6-12 µg/kg i.v. infuzí trvající více než 10 minut a po ní by měla následovat kontinuální infuze o rychlosti 0,1 µg/kg/min. V případě našeho pacienta byla zvolena vyšší úvodní dávka, která může být provázena přechodným zvýšením výskytu nežádoucích účinků. Ty však u pacienta nebyly pozorovány, naopak odezvou byla žádoucí výrazná hemodynamická odpověď.

' } ] }, { text: '

V průběhu několika minut došlo k normalizaci tepové frekvence (60 tepů/min) a QRS komplexu (Obr. 1). Pacient nabyl vědomí, ale zůstal hypotenzní. Levosimendan byl aplikován 24 hod, dokud nedošlo ke stabilizaci krevního tlaku.

Obrázek 1: Tepová frekvence a krevní tlak pacienta v průběhu léčby intoxikace. ABP: arterial blood pressure, NBP: non-invasive blood pressure.

', qs: [ ] } ], biblio: ["

Osthoff, Mirjam, et al. Levosimendan as treatment option in severe verapamil intoxication: a case report and review of the literature. Case reports in medicine.2010;2010.

http://www.ncbi.nlm.nih.gov/pmc/articles/PMC2931406/

"], editors: '06', index: 63 }, { title: 'A11cz', cathegory: 0, keywords: ["klenbuterol", "metoprolol", "diazepam", "metoprolol", "nikotin"], parts: [ { text: '

23letý pacient se dostavil do nemocnice s palpitacemi, úzkostí, svíráním na hrudi a dušností. Pacient uvedl, že před 2 hodinami si aplikoval klenbuterol, který zapil 750-1000 ml vína. Před příjezdem do nemocnice zvracel. Pacient popřel požívání jiných látek nebo sebepoškozování, nedokázal však racionálně vysvětlit velikost dávky nebo důvod, proč lék zapíjel alkoholem. Pacientova osobní anamnéza nezahrnovala žádnou medikaci, jednalo se však o kuřáka.

Při přijetí byl pacient afebrilní, diaforetický, TF 160/min, TK 115/55 mmHg a slabě se třásl.

Biochemické vyšetření v den hospitalizace: draslík 2,0 mmol/l, hořčík 0,62 mmol/l, glukóza 12,7 mmol/l, laktát 9,4 mmol/l, troponin I 0,43 µg/l, kreatinkináza 1,6 µkat/l, Hb 152 g/l, kreatinin 94 µmol/l, TSH 2,17 mU/l.

Pacient byl negativní na testy na kokain a barbituráty a jeho hladina ethanolu byla 39 mmol/l. Úvodní EKG odhalilo sinusovou tachykardii s mírnou inferolaterální ST depresí a nespecifické změny repolarizace.

', qs: [ { task: '

Do jaké farmakoterapeutické skupiny byste klenbuterol zařadili a jaký je jeho mechanismus účinku?

', answer: '

klenbuterol - bronchodilatační úlevové antiastmatikum ze skupiny selektivních agonistů b2 receptorů.

' }, { task: '

Jaké jsou indikace klenbuterolu a v jakých dávkách se běžně používá?

', answer: '

Klenbuterol se využívá k léčbě astmatu a obdobných stavů s reverzibilním zúžením dýchacích cest, díky svým bronchodilatačním účinkům. Běžná dávka k terapii astmatu u dospělého je 20-40 µg 2x denně. V mnoha zemích je vedený pouze jako veterinární léčivo.

' }, { task: '

Zjistěte základní farmakokinetické parametry klenbuterolu a spočítejte, jakou dávku pacient přibližně pozřel, pokud naměřená plazmatická hladina byla 18,4 µg/l ?

', answer: '

F=100 %; Vd=272,5 l; tmax=2-3 hod; Clcelková=110,5 ml/min; t1/2=34 hod

Pro výpočet přibližné dávky klenbuterolu u pacienta lze použít vzorec Vd=D*F/co (pouze pro přibližnou dávku, protože co je počáteční koncentrace léčiva po i.v. aplikaci). Klenbuterol je na trhu v lékových formách I. generace (neobalené tablety a sirup) a lze tedy předpokládat úplnou a rychlou absorpci.

Vd=D*F/co

D=5 mg

' }, { task: '

Proč se stanovovaly u pacienta hladiny troponinu I a kreatinkinázy a jaké jsou jejich normální hladiny?

', answer: '

Troponin I je bílkovina vyskytující se v kosterní a srdeční svalovině. Koncentrace troponinu se využívá jako jeden z ukazatelů srdečního onemocnění (diagnostika infarktu myokardu). Normální hladiny jsou 0,0-0,03 µg/l.

Zjišťování množství kreatinkinázy se využívá v diagnostice svalového postižení. Normální hladiny se liší věkem a pohlavím, ale přibližně 0,2-4,0 µkat/l.

' }, { task: '

Jsou pacientovi výsledky z biochemického vyšetření v normě? Jakou farmakoterapii byste navrhli ke zvládnutí pacientova stavu?

', answer: '

Pacient měl sníženou hladinu draslíku a zvýšené hladiny glukózy, laktátu a troponinu I. Vzhledem k pacientovu stavu a zjištěné tachykardii je nutné přerušit podávání klenbuterolu, doplnit hladiny draslíku a podle typu tachykardie zvolit vhodné antiarytmikum. Vzhledem k tomu, že se u pacienta objevily i mírné křeče, je vhodné použít myorelaxans, např. ze skupiny benzodiazepinů.

' }, { task: '

Za jakým účelem mohl být klenbuterol pacientem zneužit/zneužíván?

', answer: '

Klenbuterol se často zneužívá k redukci váhy (lipolytický účinek) a pro anabolické účinky (retence dusíku a indukce proteosyntézy).

' } ] }, { text: '

Pacientovi byly podány i.v. tekutiny společně s draslíkem. Dále mu byl podán Apaurin a Betaloc Zok (5 mg i.v.). Pacient byl stabilní a převezen na pozorování na koronární jednotku. Opakované EKG byly beze změny a transtorakální echokardiografie odhalila hyperdynamickou levou ventrikulární systolickou funkci. Pacient byl dále léčen p.o. Betaloc Zok (25mg 2x denně), Niquitin Clear (7 mg) a dle potřeby také p.o. draslíkem.

', qs: [ { task: '

Jaké účinné látky jsou obsažené v použitých léčivých přípravcích a jaké jsou jejich mechanismy účinku?

', answer: '

Apaurin obsahuje diazepam, tedy látku ze skupiny benzodiazepinů. Diazepam působí selektivně na specifické vazebné místo na GABAA receptoru.

Betaloc Zok obsahuje b-blokátor metoprolol.

Niquitin Clear náplasti obsahují nikotin a jsou indikovány jako podpora při odvykání kouření. Léčí závislost na tabákových výrobcích prostřednictvím zmírnění abstinenčních příznaků.

' }, { task: '

Kolik promile etanolu měl pacient v krvi při vstupním vyšetření?

', answer: '

ethanol 39 mmol/l = 180 mg/dl = 1,8 g/l = 1,8 promile

' }, { task: '

Na kterou látku aplikovanou u pacienta by se tedy vzhledem k množství etanolu v krvi mělo dát pozor?

', answer: '

Na diazepam.

' } ] }, { text: '

Tachykardie se u pacienta upravila během 48 hod (od poslední dávky klenbuterolu). Troponin vystoupal až na 5,39 µg/l, ale jeho EKG se postupně normalizovalo. Pacient byl propuštěn po třech dnech, s doporučením ještě týden užívat Betaloc Zok (2x denně 50 mg).

', qs: [ ] } ], biblio: ["

Barry, Arden R., et al. Case report and review of clenbuterol cardiac toxicity. Journal of Cardiology Cases. 2013;8(4): 131-133.

http://www.sciencedirect.com/science/article/pii/S1878540913001011

"], editors: '04', index: 66 }, { title: 'A12cz', cathegory: 0, keywords: ["astma", "salbutamol", "adrenalin", "metylprednizolon", "salmeterol", "flutikazon", "klaritromycin"], parts: [ { text: '

36letá žena, která se již od dětství léčí pro astma, si zavolala rychlou záchranou pomoc z důvodu akutní dušnosti (září 2007).

Pacientce byl pravidelně předepisován Seretide Diskus 50/250 a pro případ akutního zhoršení astmatu (při záchvatu) Ventolin Inhaler N.

V minulosti byla pacientka několikrát hospitalizována pro těžké astmatické záchvaty a také byla několikrát upozorněna na to, aby dodržovala správné užívání předepsané medikace. Pacientka špatně spolupracovala a její astma bylo z tohoto důvodu špatně kontrolované. Známé bylo také její špatné užívání a zneužívání přípravku Ventolin Inhaler N, na což byla několikrát upozorňována lékaři i farmaceuty, kteří ji tento přípravek i několikrát odmítli vydat. Pacientčina osobní anamnéza také zahrnovala pravidelnou konzumaci alkoholu a nedávnou abstinenci kouření.

', qs: [ { task: '

Jaké účinné látky jsou obsaženy v použitých léčivých přípravcích a jaké jsou jejich mechanismy účinku?

', answer: '

Seretide Diskus 50/250 je inhalační přípravek, který obsahuje kombinaci glukokortikoidu flutikasonu (inhibitor fosfolipázy A2) a b2 agonisty salmeterolu.

Ventolin Inhaler N obsahuje salbutamol, což je selektivní agonista b2 receptorů.

' }, { task: '

Jaký je rozdíl mezi těmito přípravky z hlediska farmakokinetiky?

', answer: '

Rozdíl je především v délce účinku b2 agonistů. Salmeterol je dlouhodobě působící b2 agonista, působí přibližně 12 hod. (díky dlouhému postrannímu řetězci) a řadíme ho tedy mezi tzv. LABA (long acting beta agonist). Salbutamol je naopak krátkodobě působící b2 agonista, působí přibližně 4-6 hod. a řadíme ho tedy mezi tzv. SABA (short acting beta agonist) a má rychlý nástup účinku (do 5 minut).

' }, { task: '

Proč pacientka zneužívala přípravek Ventolin Inhaler N, tedy proč pacientka užívala jeho vysoké dávky i mezi záchvaty?

', answer: '

Pacientka pravděpodobně zneužívala přípravek Ventolin Inhaler N z toho důvodu, že salbutamol ve vysokých dávkách je schopen vyvolat psychotropní účinky.

' } ] }, { text: '

V červenci roku 2007 byla pacientce diagnostikována akutní bronchitida a předepsán Fromilid a Medrol. V září téhož roku měla pacientka astmatický záchvat, který byl zvládnut bez nutnosti hospitalizace. Nicméně stejný den večer pacientka začala pociťovat akutní dušnost, která byla důvodem pro zavolání rychlé záchranné pomoci, jak již bylo zmíněno v úvodu. Záchranná služba ji našla v bezvědomí a s asystolií.

', qs: [ { task: '

Jaké účinné látky jsou obsaženy v použitých léčivých přípravcích a jaké jsou jejich mechanismy účinku?

', answer: '

Fromilid obsahuje klaritromycin, makrolidové antibiotikum, které se reverzibilně váže na 50s podjednotku ribozomu a tím inhibuje syntézu důležitých proteinů.

Medrol obsahuje metylprednisolon, tedy syntetický glukokortikoid. Glukokortikoidy difundují buněčnou membránou a tvoří komplexy se specifickými cytoplazmatickými receptory. Tyto komplexy poté vstupují do buněčného jádra, vážou se na DNA a stimulují transkripci mRNA s následnou proteosyntézou řady enzymů.

' }, { task: '

Jakou byste pacientce v daném případě poskytli první pomoc a jaké byste použili dávky léčiv v případě farmakoterapie?

', answer: '

Cílem první pomoci by měla být snaha obnovit spontánní cirkulaci, stabilizovat základní funkce a co nejrychleji transportovat pacienta do nejbližšího zdravotnického zařízení. Rozšířená neodkladná resuscitace prováděná profesionály zahrnuje tyto úkony: elektroimpulzoterapie (defibrilace v případě komorové fibrilace či komorové tachykardie bez hmatného pulzu), monitorace elektrické činnosti srdce (EKG), zajištění průchodnosti dýchacích cest pomocí orotracheální intubace nebo jiné alternativní metody umožňující následnou umělou plicní ventilaci s přívodem kyslíku včetně přístrojové ventilace pacienta a podání léků a infuzních roztoků. Při asystolii se co nejdříve po zajištění přístupu do cévního řečiště podává adrenalin. Dávka pro dospělého je 0,01 mg/kg (1 mg), lze opakovat po 3 – 5 minutách do návratu spontánního oběhu, nebo ukončení resuscitace.

' } ] }, { text: '

U pacientky byla okamžitě zahájena srdeční masáž, provedena intubace a byl podán i.v. adrenalin (v celkové dávce 6 mg). Srdeční funkce pacientky byla obnovena, ale s nestabilními parametry (TF 109-211 bpm, STK 70-75 mm Hg). Krátce po hospitalizaci pacientka prodělala dvě srdeční zástavy. Při druhé zástavě pacientka nereagovala na podání nejdříve 3 mg a poté 2 mg adrenalinu. O něco později byla pacientka prohlášena za mrtvou.

Vzhledem k jejímu rychlému úmrtí nebylo hned po její hospitalizaci provedeno biochemické vyšetření. Nicméně post mortem byl proveden rentgen hrudníku, toxikologické vyšetření krve, moči a histopatologie. Toxikologické vyšetření moči odhalilo vysoké koncentrace metylprednisolonu, stopové množství prednisolonu, triamcinolonu a salbutamolu 587 ng/ml. Pacientka byla negativní na nelegální látky v moči i v krvi včetně stanovení alkoholu. V krvi byla koncentrace salbutamolu 40 ng/ml. Histopatologické vyšetření odhalilo rozsáhlý plicní edém, arytmogenní dysplazii pravé srdeční komory, jaterní léze typicky spojené se srdečním selháním apod. Také se zjistilo, že pacientka byla ve 4 měsíci gestace. Příčina její smrti byla diagnostikována jako předávkování salbutamolem ve spojitosti s arytmogenní dysplazií pravé komory.

', qs: [ { task: '

Jaké jsou terapeutické plazmatické koncentrace salbutamolu?

', answer: '

Terapeutická plazmatická koncentrace salbutamolu je 4-18 ng/ml a toxická koncentrace je okolo 30 ng/ml.

' }, { task: '

Jakým mechanismem mohl salbutamol přispět ke smrti pacientky?

', answer: '

Salbutamol je sice selektivní b2 agonista, ale i při inhalačním (lokálním) podáním se může část dávky vstřebat do systémového řečiště (především u akutního předávkování či chronického zneužívání) a vyvolat tak nežádoucí kardiovaskulární účinky jako např. tachykardie a myokardiální ischemie.

' } ] } ], biblio: ["

Boucher, Alexandra, et al. Salbutamol misuse or abuse with fatal outcome: A case-report. Human & experimental toxicology. 2010; 0960327110388957.

http://het.sagepub.com/content/early/2010/12/01/0960327110388957.abstract

"], editors: '04', index: 69 }, // O { title: 'O1cz', cathegory: 3, keywords: ["azitromycin", "malárie", "meflochin", "chinin", "klindamycin", "virová hepatitida", "chlorochin", "proguanil", "atovaquon"], parts: [ { text: '

59 letý muž byl praktickým lékařem doporučen na infekční kliniku pro 6 dnů trvající horečnaté onemocnění s diagnózou: septický stav nejasného původu. Horečka začala náhle z plného zdraví. Již druhý den v noci dosáhla 40° C a provázely ji zimnice a třesavka. Nemocný udával bolest hlavy, bolesti v zádech, nechutenství, řidší stolici a cítil se jako při chřipce. Lékař pomýšlel na letní virózu a ordinoval azitromycin a antipyretikum. Po 3 dnech však horečka neklesla a dostavovala se každý den nepravidelně v záchvatech trvajících 8 až 12 hodin. Při vzestupu teploty nemocný pociťoval zimnici a při poklesu se nápadně potil. Byl malátný, měl tmavší moč a objevilo se zažloutnutí sklér. Při kontrole stavu lékař doporučil hospitalizaci.

', qs: [ { task: '

Co je to azitromycin a jaký je mechanismus jeho účinku?

', answer: '

Azitromycin je makrolidové antibiotikum patřící do skupiny azalidů. Mechanismus účinku azitromycinu je založen na potlačení bakteriální syntézy proteinů vazbou na 50 S podjednotku a tím inhibicí přesunu peptidů.

' }, { task: '

Souhlasíte s navrženou terapií vzhledem k předpokládané diagnóze? Pokud ne, navrhněte terapeutický postup.

', answer: '

Použití antibiotika při stanovení diagnózy infekce virové etiologie není v souladu s principy racionální farmakoterapie. Terapie běžných virových infekcí je většinou symptomatická a zahrnuje podávání látek ze skupiny nesteroidních antiflogistik a antipyretik. Pokud je infekce spojena s výraznou ztrátou tekutin (zvracení, průjem, zde pocení), je vhodné pacienta dostatečně rehydratovat. Doporučuje se také zvýšený příjem vitamínu C.

' } ] }, { text: '

Při příjmu měl nemocný horečku 38,6° C, měl ikterické skléry. Laboratorní vyšetření: sedimentace erytrocytů 22/48, C-reaktivní protein 195 mg/l, leukocyty 3,5× 109/l bez významnější změny v rozpočtu, trombocytopenie 30× 109/l, biochemické známky poškození jater a ledvin. Při epidemiologické anamnéze bylo zjištěno, že nemocný se 10 dnů před začátkem horečky vrátil z třítýdenního turistického pobytu v Keni. Měl platné očkování proti žluté zimnici aplikované před 2 lety, a proto před odjezdem nenavštívil žádné pracoviště cestovní medicíny. Za pobytu v Keni byl několikrát poštípán komáry. Malarickou chemoprofylaxi neužíval a před napadením komáry se nechránil repelenty ani jiným způsobem. Při diagnostické rozvaze přicházela na prvním místě v úvahu malárie, kterou potvrdilo mikroskopické vyšetření krevního nátěru a tlusté kapky. Při vyšetření provedeném v Národní referenční laboratoři pro diagnostiku tropických parazitóz byly nalezeny trofozoity Plasmodium falciparum a ojediněle i vývojová stádia Plasmodium malariae.

', qs: [ { task: '

Jaké existují možnosti profylaxe malárie?

', answer: '

Chlorochin

(Delagil, Resorchin, Aralen, Nivaquine aj.) užívá se pro léčbu a profylaxi malárie, která není na preparát rezistentní (včetně nekomplikované tropické malárie). Dávkování při profylaxi: V našich podmínkách je běžně dostupný preparát Delagil tbl. (150 mg báze v 1 tabletě). Profylakticky podáváme 300 mg 1x týdně, při tělesné hmotnosti nad 75 kg 450 mg 1x týdně jednorázově nebo rozděleně do 2-3 dávek. Profylaxi zahajujeme zpravidla týden před vstupem do malarické oblasti a pokračujeme po celou dobu pobytu a dále 4 týdny po návratu z malarické oblasti.

Chlorochin + proguanil

V místech s mírnou nebo částečnou rezistencí plazmodií můžeme podávání chlorochinu kombinovat s proguanilem. Dávkování při profylaxi: V kombinaci se chlorochin (Delagil tbl.) podává v dávce 300 mg týdně a proguanil (Paludrine tbl.) v jednorázové dávce 200 mg ráno nebo rozděleně do 2 dávek 100 mg ráno a večer.

Meflochin (Lariam, Mephaquin)

Jedná se o antimalarikum, které je stále ještě nejpoužívanějším a nejefektivnějším preparátem v místech s výskytem Plasmodium falciparum rezistentním na chlorochin. Dávkování při profylaxi: Profylaxi začínáme nejpozději 1 týden před vstupem do malarické oblasti dávkou 250 mg meflochinu týdně, touto dávkou pokračujeme po celou dobu pobytu a pokračujeme 4 týdny po výjezdu z malarické oblasti.

Atovaquon 250 mg + proguanil 100 mg (Malarone)

Malarone je určen především pro léčbu tropické malárie v případech, kdy selhala předchozí léčba onemocnění vyvolaného rezistentními kmeny Plasmodium falciparum. Dávkování pro profylaxi: Pro profylaxi se dávkuje 1 tbl. denně. S profylaxí se začíná 1-2 dny před vstupem do oblasti a končí 7 dní po výjezdu z oblasti výskytu tropické rezistentní malárie. Používáme maximálně po dobu 28 dní a u cestovatelů s minimální tělesnou váhou 40 kg.

Tetracyklin, doxycyklin

Mají nevýhody známé při podávání tetracyklinových antibiotik. Kromě možné nesnášenlivosti nebo trávicích obtíží je hlavní nevýhodou fotosenzibilita, kterou tetracyklinové preparáty obecně vyvolávají. Pro profylaxi lze užít samostatně, v terapii malárie vždy v kombinaci. Nepodáváme dětem do 8 let a těhotným.

' }, { task: '

Jaké jsou nejčastější nežádoucí účinky profylakticky podávaných antimalarik?

', answer: '

Nežádoucí účinky chlorochinu

kožní projevy - exantémy, pruritus

kardiovaskulární aparát - pokles TK, deprese ST úseku na EKG, agranulocytóza, trombocytopenie, pancytopenie

gastrointestinální potíže - nauzea, vomitus, průjmy, nechutenství, gastralgie

CNS - závrať, cefalea, neklid až excitace, poruchy spánku.

Nežádoucí účinky chlorochin + proguanilu

Vedlejší účinky současného podávání obou preparátů zhruba odpovídají výše popsaným účinkům chlorochinu.

Nežádoucí účinky meflochinu

Poruchy koncentrace a spánku, nezvyklé sny, pocit slabosti, závratě, bolesti hlavy, poruchy rovnováhy, nevolnost, nauzea, vomitus.

Nežádoucí účinky Malaronu

Bolest břicha, bolest hlavy, anorexie, nevolnost, zvracení, průjem.

' } ] }, { text: '

Ihned byla zahájena léčba meflochinem, ale pacient po podání tablet zvracel. Za 20 hodin po přijetí pacient zvracel opakovaně, málo močil a byl bradypsychický až somnolentní. Laboratorní nálezy svědčily pro maligní formu tropické malárie s anémií, nedostatečností ledvin a poškozením jater. Nemocný byl umístěn na jednotku intenzivní péče, kde byla zavedena parenterální léčba chininem s klindamycinem. Třetí den pobytu se průběh onemocnění komplikoval počínajícím syndromem dechové tísně s dušností, hypoxémií a rentgenovým nálezem připomínajícím šokovou plíci. Po nasazení chininu došlo během 4 dnů k normalizaci teploty, vymizení parazitémie a k postupné úpravě hodnot renálních funkcí. Nemocný byl přemístěn zpět na oddělení běžné péče. Druhý den po vysazení antimalarik se však znovu objevila horečka. Tentokráte nepřesáhla 38,5° C a neprovázely ji zimnice ani třesavka. Lékaři soudili na časný relaps malarické infekce podmíněný sníženou citlivostí plasmodií na podaná antimalarika. Negativní výsledek opakovaného vyšetření krevního nátěru a tlusté kapky však relaps malárie vyloučil. Horečka po třech dnech vymizela, ale subikterus se změnil v intenzivní žloutenku, kterou provázely biochemické známky výrazného poškození jaterního parenchymu. Vyšetření sérologických markerů virových hepatitid svědčilo pro probíhající virovou hepatitidu typu A. Průběh onemocnění hepatitidou A byl protrahovaný a k normalizaci jaterních testů došlo až koncem 4. měsíce rekonvalescence.

', qs: [ { task: '

Co je to meflochin a jaký je mechanismus jeho účinku?

', answer: '

meflochin - antimalarikum působící na krevní nepohlavní formy (schizonty) malarických plasmodií: Plasmodium falciparum, Plasmodium vivax, Plasmodium malariae a Plasmodium ovale. Na základě vysokého gradientu pH prostupuje membránou vakuol a velmi silně se kumuluje v kyselém prostředí trávících vakuol schizontů. Schizonty tráví ve vakuolách hemoglobin napadených erytrocytů. Jako nestravitelný produkt metabolizmu vzniká hem, který je pro parazita jedovatý. Hem je parazitem polymerizován a vzniká makromolekula, hemazoin, která je pro něj nejedovatá. Meflochin tuto přeměnu inhibuje.

' }, { task: '

Co je to chinin a jaký je mechanismus jeho účinku?

', answer: '

Jako u meflochinu.

' }, { task: '

Co je to klindamycin a jaký je mechanismus jeho účinku?

', answer: '

klindamycin - makrolidové antibiotikum ze skupiny linkosamidů. Klindamycin se váže na 50S podjednotku ribozomu a inhibuje časné stádium syntézy proteinů. Mikrobiální spektrum zahrnuje G+ i G- aerobní bakterie a anaerobní bakterie.

' }, { task: '

V jaké jiné indikaci mohou být použita antimalarika?

', answer: '

Další indikací antimalarik je revmatoidní artritida. Antimalarika jsou poměrně slabě účinná DMARDs s dlouhým nástupem účinku, jsou však velmi dobře to­lerována. Efekt nastupuje po 4–6 měsících, účinnost antimalarik je potvrzena u 60–80 % pacientů. Mechanis­mus účinku antimalarik u autoimunitních onemocnění není přesně znám, je však pravděpodobně dán kumula­cí antimalarik v lysozomech leukocytů, fibroblastů a po­lymorfonukleárů. Zástupci jsou chlorochin a hydroxychlorochin. Nežádoucí účin­ky: gastrointestinální obtíže s nauzeou, průjmy, exanté­my, okulotoxicita.

' } ] } ], biblio: ["

Rutsch J. Profylaxe malárie. Postgraduální medicína. 2005;7:193-196.

http://zdravi.e15.cz/clanek/postgradualni-medicina/profylaxe-malarie-166071

", "

Suchý D. Revmatoidní artritida – diagnóza a léčba. Interní medicína pro praxi. 2003;7:342-347.

http://www.internimedicina.cz/pdfs/int/2003/07/05.pdf

", "

Vaništa J. Importovaná nákaza malárií a virovou hepatitidou. Interní medicína pro praxi. 2005; 7-8:368-369.

http://www.internimedicina.cz/pdfs/int/2005/07/12.pdf

"], editors: '02', index: 60 }, ]